0% found this document useful (0 votes)
591 views

Physics QP

This document discusses scalars and vectors. Scalars are physical quantities that have only magnitude, such as mass, time, and temperature. Vectors are physical quantities that have both magnitude and direction, such as displacement, velocity, and force. The document provides examples of scalars and vectors and explains how to describe them. It also discusses position vectors, displacement vectors, and how to add and represent vectors graphically.

Uploaded by

Gowri Shankar
Copyright
© © All Rights Reserved
We take content rights seriously. If you suspect this is your content, claim it here.
Available Formats
Download as DOCX, PDF, TXT or read online on Scribd
0% found this document useful (0 votes)
591 views

Physics QP

This document discusses scalars and vectors. Scalars are physical quantities that have only magnitude, such as mass, time, and temperature. Vectors are physical quantities that have both magnitude and direction, such as displacement, velocity, and force. The document provides examples of scalars and vectors and explains how to describe them. It also discusses position vectors, displacement vectors, and how to add and represent vectors graphically.

Uploaded by

Gowri Shankar
Copyright
© © All Rights Reserved
We take content rights seriously. If you suspect this is your content, claim it here.
Available Formats
Download as DOCX, PDF, TXT or read online on Scribd
You are on page 1/ 92

SCALARS AND VECTORS

All measurable quantities are called physical quantities. Most of the physical
Quantities are classified into ‘Scalars’ and ‘Vectors’.
Scalar:- Physical quantities having only magnitude are called Scalars.

Eg: Length, time, volume, density, temperature, mass, work, energy, electric charge,
electric current, potential, resistance, capacity, etc…..

 To describe a scalar quantity we require


(a) The specific unit of that quantity
(b) The number of times that unit is contained in that quantity

E.g: A bag contain 100 kg of sugar. Here kg is the unit and 100 is the number of units of sugar present in the
bag.

Note: Unit is not a compulsion to represent a scalar


Eg: Specific gravity, Refractive index

Note: Mathematical operations of scalar quantities yield scalar quantities and these quantities are
manipulated by ordinary algebraic rules.

Vector : - Physical quantities having both magnitude and direction and that obeys laws of vector addition are
called vectors(or)
The vector, as a mathematical object, is defined as a directed line segment.
It should also obey the laws of vector addition.

Eg: Displacement, velocity, acceleration, force, momentum, angular momentum, moment of force. Torque,
magnetic moment, magnetic induction field. Intensity of electric field, etc….
 To describe a vector quantity we require.
(a) The specific unit of that quantity.
(b) The number of times that unit is contained in that quantity.
(c) The orientation of that quantity.
Eg: A plane is flying from west to east with a velocity of 50ms-1. Here ms-1 is the unit, 50 is the number of
units of velocity and west to east is the direction.
Note : A physical quantity having magnitude and direction but not obeying laws of vector addition is
treated as a scalar
Eg: Electric current is a scalar quantity
Electric current is always associated with direction, but it is not a vector
quantity. It does not obey law of vector addition for its addition.

The resultant of i1 and i2 is (i1 + i2) by Kirchhoff’s current law. The resultant does not depend on angle
between currents i1 and i2.

Note: Equations in vector form indicate both mathematical and geometrical relationships
among the quantities. Physical laws in vector form are very compact and independent of
choice of coordinate system.

POSITION AND DISPLACEMENT VECTORS

To described the position of an object moving in a plane, we need to choose a


convenient point, say O as origin. Let P and P1 be the positions of the object at time
t and t1, respectively as shown in Fig. We join O and P by a straight line. Then, OP
is the position vector of the object at time t. An arrow is marked at the head of this line.
It is represented by a symbol r, i.e. OP = r. P1 is represented by another position vector,
OP1 denoted by r1. The length of the vector r represents the magnitude of the vector
and its direction is the direction in which P lies as seen from O. If the object moves
from P to P1, the vector PP1 (with tail at P and tip at P1) is called the displacement vector corresponding to
motion from point P (at time t) to point P1(at time t1).

It is important to note that displacement vector is the straight line joining the
initial and final positions and does not depend on the actual path undertaken by the object between the two
positions.
EQUALITY OF VECTORS

Two vectors A and B are said to be equal if, and only if, they represent the same
physical quantity with same magnitude and the same direction.
Figure (a) shows two equal vectors A and B. We can easily check their equality.
Shift B parallel to itself until its tail Q coincides with the of A, i.e Q coincides with O.
Then, since their tips S and P also coincide, the two vectors are said to be equal. In general, equality
is indicated as A = B.
Note that in Fig. (b). vectors A’ and B’ have the same magnitude but they are not equal because they
have different directions. Even if we shift B’ parallel to itself so that its tail Q’ coincides with the tail O’ of A’,
the tip S’ of B’ does not coincide with the tip P’ of A’

Fig(a) Two equal vectors A and B. (b) Two vectors A’ and B’ are unequal thought they are of the same
length.

Note: Two or more vectors (representing same physical quantity) are called equal if their magnitudes and
directions are same.

Eg: Suppose two trains are running on parallel tracks with same speed and direction.
Then their velocity vectors are equal vectors.

Note: If a vector is displaced parallel to itself its magnitude and direction does not change.

TYPES OF VECTORS

(a) Polar Vectors : The vector whose direction does not change even though the
coordinate system in which it is defined changes is called polar vector or Real vector
Eg : Force, momentum, Acceleration.

(b) Axial Vectors : The vectors whose direction changes with the coordinate system in
which it is defined change is called axial vector or Pseudo vector
Eg: Angular velocity, torque, angular momentum

(c) Like Vectors (or) parallel Vectors: Two or more vectors (representing same
physical quantity) are called like vectors if they are parallel to each other, however
their magnitudes may be different.

(d) Unlike Vectors (or) anti parallel Vectors : Two vectors (representing same
physical quantity) are called unlike vectors if they act in opposite direction however their magnitudes can be
different.

(e) Negative Vector : A vector having the same magnitude and opposite in direction to that of a given
vector is called negative vector of the given vector

(f) Co-initial Vectors : The vectors having same initial point are called co-initial vectors.

(g) Colinear Vectors : Two or more vectors are said to be collinear when they act along the same line
however their magnitudes may be different.
Eg : Two vectors ⃗A and ⃗B as shown are collinear vectors.

(h) Coplanar Vectors: A number of vectors are said to be coplanar if they are in the same plane or
parallel to the same plane. However their magnitudes may be different.

(i) Unit Vector: A vector whose magnitude equals one and used to specify a convenient direction is called a
unit vector.

A unit vector has no units and dimensions. Its purpose is to specify the direction of given vector.
In Cartesian coordinate system, unit vectors along positive x, y and z axis are symbolized as î , ĵ and k̂
respectively. These three units vectors are mutually perpendicular and their magnitudes |î| = |ĵ| = |k̂| = 1.
⃗ is a non zero vector, then the unit vector in the direction of A
If A ⃗ is given by

⃗⃗
A
̂=
A ⃗⃗ |
|A
(j) Null Vector (OR) Zero Vector

A vector whose magnitude is equal to zero is called a null vector. Its origin coincides with
terminus and its direction is indeterminate.

Examples of zero vector:

1. The velocity of a particle at rest


2. The acceleration of a particle moving at uniform velocity.
3. The displacement of a stationary object over any arbitrary interval of time.
4. The position vector of a particle at the origin

Note : ⃗A + ⃗O
⃗ = ⃗A, ⃗A × ⃗O
⃗ = ⃗O

Note : In our study, vectors do not have fixed locations. So displacing a vector parallel to itself
leaves the vector unchanged. Such vectors are called free vectors. However, in some physical applications,
location or line of application of a vector is important. Such vectors are called localized vectors.

MULTIPLICATION OF VECTORS BY REAL NUMBERS

Multiplying a vector A with a positive number  gives a vector whose magnitude is changed by the
factor  but the direction is the same as that of A |A| =  |A| if  > 0.

For example, if A is multiplied by 2, the resultant vector 2A is in the same direction as A and has a
magnitude twice of |A| as shown in Fig. (a).

Multiplying a vector A by a negative number  gives a vector  A whose direction is opposite to the
direction of A and whose magnitude is -  times |A|.

Multiplying a given vector A by negative numbers, say -1 and -1.5, gives vectors as shown in Fig (b).

Fig. (a) Vector A and the resultant vector after multiplying A by a positive number 2.
(b) Vector A and resultant vectors after multiplying it by a negative number -1 and -1.5.
The factor  by which a vector A is multiplied could be a scalar having its own physical dimension.
Then, the dimension of  A is the product of the dimensions of  and A. For example, if we multiply a
constant velocity vector by duration (of time), we get a displacement vector.

REPRESENTATION OF ANGLE BETWEEN THE TWO VECTORS

The angle between two vectors is represented by the smaller of the two angles between the
vectors when they are placed tail to tail by displacing either of the vectors parallel to it self.

⃗ is correctly represented in the following figures.


Eg: The angle between 𝐴 and 𝐵
(a) If the angle between 𝐴 and 𝐵⃗ is 𝜃, then the angle between 𝐴 and K𝐵
⃗ is also 𝜃.
Where ‘K’ is a positive constant.

(b) ⃗ is 𝜃, then the angle between 𝐴 and - K𝐵


If the angle between 𝐴 and 𝐵 ⃗ is (180 – 𝜃). Where K is a
positive constant.

(c) Angle between collinear vectors is always zero or 1800C

ADDITION OF VECTORS (GRAPHICAL METHOD)

1. To add two vectors geometrically represent the vectors by arrow head lines using the same suitable
scale, with their proper directions in the chosen coordinate system.
2. Join the initial point of the second vector with the final point of the first vector by moving parallel to
itself
3. Now, draw an arrow from the initial point of the first vector to the final point of the second vector.
This arrow represents the resultant of the two vectors.

Cases:

1. When two vectors are acting in the same direction.


⃗ be acting in the same direction.
Let the two vectors 𝑃⃗ and 𝑄

2. When two vectors are acting at some angle:


⃗ with the final point of 𝑃⃗ and then, to find the resultant of these two,
First join the initial point of 𝑄
⃗ . This single vector 𝑅⃗ is the resultant of
draw a vector 𝑅⃗ from the initial point of 𝑃⃗ to the final point of 𝑄
⃗.
vectors 𝑃⃗ and 𝑄

⃗ both in magnitude and direction. So, 𝑅


𝑅⃗ represents the resultant of 𝑃⃗ and 𝑄 ⃗ .
⃗⃗⃗ = 𝑃⃗ + 𝑄

LAWS OF VECTOR ADDITION

(a) Vector addition obeys commutative law.


⃗ are two
Addition of two vectors is independent of the order of the vectors in which they are added. If 𝑃⃗ and 𝑄
⃗ =𝑄
vectors then 𝑃⃗ + 𝑄 ⃗ + 𝑃⃗ as shown in fig

(b) Vector addition obeys associative law.


While adding more than two vectors, the resultant is independent of the order in which they are added.
⃗ and 𝑅⃗ are three vectors, then (𝑃⃗ + 𝑄
If 𝑃⃗, 𝑄 ⃗ ) + 𝑅⃗ = 𝑃⃗+ (𝑄
⃗ + 𝑅⃗ as shown in fig.

(c) Vector addition obeys distributive law.


If k, k1, k2 are scalars then
K (𝑃⃗ + 𝑄⃗ ) = k 𝑃⃗ + k𝑄
⃗ and 𝑃⃗(k1 + k2) = k1𝑃⃗ + k2𝑃⃗

Note : Vector addition is possible only between vectors of same kind.

Note : What is the result of adding two equal and opposite vectors? Consider two vectors A and – A
shown in Fig. Their sum is A + (A). Since the magnitudes of the two vectors are the same, but the directions
are opposite, the resultant vector has zero magnitude and is represented by 0 called a null vector or a zero vector

Since the magnitude of a null vector is zero, its direction cannot be specified.
The null vector also results when we multiply a vector A by the number zero.The main properties of
0 are : A + 0 = A
0=0 0A=0
Application-4.1 :
Let the person having 3m displacement towards East and 4m displacement towards North. In vector
form displacement towards East is 3𝑖̂.
Displacement towards North is 4𝑖̂
⃗⃗⃗⃗⃗ = 𝑂𝐴
𝑂𝐵 ⃗⃗⃗⃗⃗ + 𝐴𝑃
⃗⃗⃗⃗⃗ 𝑆 = 3𝑖̂ + 4𝑗̂

Magnitude of displacement |𝑆| = √32 + 42 = 5


4
Angle made by the vector with x-axis is ∆OAB tan𝜃= 3
4
𝜃 = tan-1(3)
4
∴ Vector makes an angle tan-1(3) with +x axis in anti clock with direction.
SUBTRACTION OF VECTORS

The process of subtracting one vector from another is equivalent to adding vectorially the
negative of the vector to be subtracted.

⃗ be the two vectors as shown in figure. We want to find the difference 𝑃⃗ - 𝑄


Let 𝑃⃗ and 𝑄 ⃗ . Let a vector - 𝑄

⃗ ).
be added to the vector 𝑃⃗ by the laws of vector addition. Their resultant gives the value of (𝑃⃗ - 𝑄

LAWS OF VECTOR SUBTRACTION

(a) The vector subtraction does not follow commutative law i.e.
⃗ ≠𝑄
𝑃⃗ - 𝑄 ⃗ - 𝑃⃗ But |𝑃⃗ - 𝑄
⃗ | = |𝑄
⃗ - 𝑃⃗|

(b) ⃗ - 𝑅⃗ ) ≠ (𝑃⃗ - 𝑄
The vector subtraction does not follow associative law ie. 𝑃⃗ - (𝑄 ⃗ ) - 𝑅⃗

(c) ⃗ ) = m𝑃⃗ - m𝑄
Vector subtraction follows distributive law m(𝑃⃗ - 𝑄 ⃗

RESOLUTION OF A VECTOR INTO RECTANGULAR COMPONENTS

Resolution of a vector is the process of obtaining the component vectors which when combined,
according to laws of vector addition, produce the given vector.
Consider a vector 𝑟 in the xy plane and it makes an angle 𝜃 with x axis as shown in figure.
𝑖̂ and 𝑗̂ are unit vectors along X-axis and Y-axis respectively.

From the point R, draw RP and RQ perpendicular to X-axis and Y-axis respectively. From the
parallelogram law of vector addition, it follows that

⃗⃗⃗⃗⃗ = ⃗⃗⃗⃗⃗
𝑂𝑅 𝑂𝑃 + 𝑂𝑄⃗⃗⃗⃗⃗⃗ …….. (1)
If OP = x and OQ = y, then
⃗⃗⃗⃗⃗ = x𝑖̂ and 𝑂𝑄
𝑂𝑃 ⃗⃗⃗⃗⃗⃗ = y𝑗̂
From equation (1) 𝑟̂ = x𝑖̂ + y𝑗̂ …….. (2)
In the above equation x𝑖̂ and y𝑗̂ are called the x-component and the y-component of the
vector 𝑟, and x and y are called magnitudes of the two component vectors.

From the triangle OPR.


𝑃𝑄 𝑥
cos𝜃 = 𝑂𝑅 = 𝑟 ⇒ x = r cos 𝜃 ………(3)
𝑃𝑄 𝑦
and sin 𝜃 = 𝑂𝑅 = 𝑟 ⇒ y = r sin 𝜃 ………(4)
𝑦
∴ from (3) and (4) tan 𝜃 = 𝑥 ……….. (5)

The equation (3) and (4) gives the magnitudes of the rectangular component vectors in terms
of the magnitude of the given vector and its inclination with X-axis.

∴ from equation (2), 𝑟 = (r cos 𝜃) 𝑖̂ + (r sin 𝜃) 𝑗̂

from right angled triangle OPR

OR = √𝑂𝑃2 + 𝑃𝑅 2 or r = √𝑥 2 + 𝑦 2 …… (6)

The equation (6) gives the magnitude of the vector in terms of its rectangular components.

Note : Method involving resolution of vectors into components to find the resultant of the vectors is
known as analytical method.

Note : The components of a vector are independent of each other and can be handled separately.

Note: Theoretically, a given vector can be made the diagonal of infinite number of parallelograms.
Thus there can be infinite number of ways to divide a vector into components.

Let a and b be any two nonzero vectors in a plane with different directions and let A be another
vector in the same plane (Fig.). A can be expressed as a sum of two vectors – one obtained by multiplying b
another real number. To see this, let O and P be the tail and head of the vector A. Then, through O,
draw a straight line parallel to a, and through P, a straight line parallel to b. Let them intersect at Q.
Then, we have
A = OP = OQ + QP

But since OQ is parallel to a, and QP is parallel to b, we can write:

OQ =  a, and QP = 𝜇b

Where  and 𝜇 are real numbers. Therefore, A =  a + 𝜇b

We say that A has been resolved into two component vectors  a and 𝜇b along a and b respectively. Using this
method one can resolve a given into two compo nent vectors along a set of two vectors – all the three lie in the
same plane.

Note :

(a) If the vector 𝐴 is in first quadrant then it can be written as 𝐴 = Ax𝑖̂ + Ay𝑗̂

(b) ⃗ is in second quadrant then 𝐵


If the vector 𝐵 ⃗ = Bx𝑖̂ + By𝑗̂

(c) If the vector 𝐶 is in third quadrant then 𝐶 = Cx𝑖̂ + Cy𝑗̂

(d) ⃗ is in fourth quadrant then 𝐷


If the vector 𝐷 ⃗ = Dx𝑖̂ + Dy𝑗̂

Application 4.2 :

A block is placed on smooth horizontal surface and pulled by a force ‘F’ making an angle‘𝜃’with
horizontal
The component of force along horizontal = Fcos 𝜃.

The component of force along vertical = Fsin 𝜃.

Application – 4.3
A block of mass ‘m’ is placed on inclined plane of angle 𝜃 then the component of
weight parallel to the inclined plane is ‘mg sin 𝜃’, the component of weight perpendicular to the inclined
plane is mgcos 𝜃.

Application-4.4:

A simple pendulum having a bob of mass ‘m’ is suspended from a rigid support and it is pulled by a
horizontal force ‘F’. The string makes an angle 𝜃 with the vertical as shown in figure.

The horizontal component of tension = T sin 𝜃

The vertical component of tension = T cos 𝜃

When the bob is in equilibrium


T sin 𝜃 = F, ………. (1)
T cos 𝜃 = mg ……… (2)

𝑚𝑔 𝑚𝑔𝑙
T = 𝑐𝑜𝑠𝜃 = √12 2
−𝑥
From equations (1) and (2)
𝐹 𝑥
Tan 𝜃 = 𝑚𝑔 ⇒ F = mgTan 𝜃= mg √12
−𝑥 2
T= √𝐹 2 − (𝑚𝑔)2

Note : If a vector is rotated through an angle other than integral multiple of 2𝜋 (or 3600) it changes,
but its magnitude does not change.

Note : If the frame of reference is rotated the vector does not change (though its components may
change).

RESOLUTION OF A VECTOR INTO THREE RECTANGULAR COMPONENTS

⃗⃗⃗⃗⃗ as shown in fig. With O as origin, construct a


Let us consider a vector 𝐴 be represented by 𝑂𝑃
rectangular parallelepiped with three edges along the three rectangular axes which meet at O.
𝐴 be-comes the diagonal of the parallelepiped.𝐴 ⃗⃗⃗⃗𝑥 , ⃗⃗⃗⃗
𝐴𝑦 and ⃗⃗⃗⃗
𝐴𝑧 are three vector intercepts along x, y
and z axes respectively. These are the three rectangular components of 𝐴.
Applying triangle law of vectors,
⃗⃗⃗⃗⃗
𝑂𝑃 = ⃗⃗⃗⃗⃗⃗
𝑂𝐾 + ⃗⃗⃗⃗⃗
𝐾𝑃 …………….. (1)
Applying parallelogram law of vectors,

⃗⃗⃗⃗⃗⃗
𝑂𝐾 = 𝑂𝑇⃗⃗⃗⃗⃗ + 𝑂𝑄
⃗⃗⃗⃗⃗⃗ ………………. (2)
From (1) and (2) ∴ ⃗⃗⃗⃗⃗ ⃗⃗⃗⃗⃗ + 𝑂𝑄
𝑂𝑃 = 𝑂𝑇 ⃗⃗⃗⃗⃗⃗ + ⃗⃗⃗⃗⃗
𝐾𝑃

𝐾𝑃 = ⃗⃗⃗⃗⃗
But ⃗⃗⃗⃗⃗ 𝑂𝑆, ∴ ⃗⃗⃗⃗⃗ ⃗⃗⃗⃗⃗ + 𝑂𝑄
𝑂𝑃 = 𝑂𝑇 ⃗⃗⃗⃗⃗⃗ + ⃗⃗⃗⃗⃗
𝑂𝑆
⃗⃗⃗⃗ ⃗⃗⃗⃗ ⃗⃗⃗⃗
𝐴 = 𝐴𝑧 + 𝐴𝑥 + 𝐴𝑦 ,or 𝐴 = Ax𝑖̂ + Ay𝑗̂ + Az𝑘̂

Again, OP2 = OK2 + KP2 = OQ2 + OT2 + KP2


or A2 =𝐴2𝑥 +𝐴2𝑧 +𝐴2𝑦 [∴ KP = OS = Ay]
or A = √𝐴2𝑥 + 𝐴𝑦2 + 𝐴𝑧2
This gives the magnitude of 𝐴 in terms of the magnitudes of components ⃗⃗⃗⃗
𝐴𝑥 , ⃗⃗⃗⃗
𝐴𝑦 and ⃗⃗⃗⃗
𝐴𝑧 .

DIRECTION COSINES

The direction cosines ,m and n of a vector are the cosines of the angles 𝛼, 𝛽 and 𝛾 which a given
vector makes with x-axis, y-axis and z-axis respectively.

If 𝛼, 𝛽,𝛾 are angle made by 𝐴 with x, y, z axis and cos 𝛼, cos 𝛽, cos 𝛾 are called direction cosines.
𝐴 𝐴𝑦
∴ = cos 𝛼 = 𝐴𝑥 , m = cos 𝛽 = 𝐴 , n = cos 𝛾 =
Squaring and adding
cos2 𝛼 + cos2 𝛽 + cos2 =

𝐴2𝑥 𝐴2𝑦 𝐴2 𝐴2𝑥 +𝐴2𝑦 +𝐴2𝑧 𝐴2


+ 𝐴2 + 𝐴𝑧2 = = 𝐴2:
𝐴2 𝐴2

i⇒ cos2 𝛼 + cos2 𝛽 + cos2 𝛾 = 1

and sin2 𝛼 + sin2 𝛽 + sin2 𝛾 = 2

VECTOR ADDITION ANALYTICAL METHOD


⃗ in terms of their components.
Let us consider the addition of two vectors 𝑃⃗ and 𝑄

⃗ = Qx𝑖̂ + Qy𝑗̂
We have 𝑃⃗ = Px𝑖̂ + Py𝑗̂, 𝑄

Let 𝑅⃗ be the resultant vector with component. Rx and Ry along x and y axis respectively.

Then 𝑅⃗ x = Rx 𝑖̂ and 𝑅⃗ y = Ry 𝑖̂.

From the diagram Rx = Px + Qx and Ry = Py + Qy

∴ 𝑅⃗ x = (Px + Qx) 𝑖,̂ 𝑅⃗y = (Py + Qy)

∴ 𝑅⃗ = Rx 𝑖̂ + Ry 𝑗̂; 𝑅⃗ = (Px + Qx) 𝑖̂ + (Py + Qy)

𝟐
|𝑅⃗ | = √(𝑷𝒙 + 𝑸𝒙 )𝟐 + (𝑷𝒚 + 𝑸𝒚 )

𝑅𝑦 𝑃𝑦 +𝑄
∴ Tan𝛼 = , Tan𝛼
𝑅𝑥 𝑃𝑥 +𝑄

𝑃𝑦 +𝑄𝑦
∴ 𝛼 = Tan-1(𝑃 )
𝑥 +𝑄𝑥

PARALLELOGRAM LAW OF VECTORS

Statement: If two vectors are drawn from a point so as to represent the adjacent sides of
parallelogram both in magnitude and the direction, the diagonal of the parallelogram drawn from the same
point represents the resultant of the two vectors both in magnitude and direction.

⃗ , inclined at angle 𝜃 be acting on a particle at the same time. Let they be represented
Let the two vectors 𝑃⃗ and 𝑄
in magnitude and direction by two adjacent sides 𝑂𝐴⃗⃗⃗⃗⃗ and 𝑂𝐵
⃗⃗⃗⃗⃗ of parallegram OACB drawn from a point O

According to parallelogram law of vectors, their resultant vector 𝑅⃗ will be represented by the diagonal
⃗⃗⃗⃗⃗ of the parallelogram
𝑂𝐶

Magnitude of resultant:

The line OA is extended up to point N and draw a perpendicular from point C on to the extended
line as shown in fig.
From the parallelogram OACB

DBOA = DCAN = 𝜃 and

OA = P, OB = AC = Q,

OC = R and

In DCNA,

AN = AC cos 𝜃 = Q cos 𝜃 .........(1)

CN = AC sin 𝜃 = Q sin .....(2)

From triangle ON C

OC2 = ON2 + NC2

(OC)2 =( OA+ AN)2 +( NC)2

R2 = (OA)2 + 2(OA)(AN) + (AN)2 + (NC)2

R2 = P2 + 2 PQCos 𝜃 Q2

R = √𝑃2 + 𝑄 2 + 2𝑃𝑄 cos 𝜃 ....................(3)

⃗ and also on the


The magnitude of the resultant 𝑅⃗ depends on the magnitudes of the vectors 𝑃⃗ and 𝑄
angle between them.

Direction of resultant:

If the line of action of the vector 𝑃⃗ is taken as reference line, the resultant 𝑅⃗ makes an angle 𝛼 with it.
This angle indicates the direction of 𝑅⃗ . Then from right angled triangle ONC,
𝐶𝑁 𝐶𝑁 𝑄 sin 𝜃
tan 𝛼 = 𝑂𝑁 = 𝑂𝐴+𝐴𝑁 = 𝑃+𝑄𝑐𝑜𝑠𝜃 ..........(4)

⃗ , then
Note 4.17: If 𝛽 is the angle between the resultant 𝑅⃗ and the vector 𝑄
𝑃 𝑠𝑖𝑛𝜃
𝛽 = tan-1 (𝑄+𝑃 cos 𝜃),

Special Cases:

(a) If the magnitude of P > Q then 𝛼 < . i.e., the resultant is closer to the vector of larger magnitude.

(b) When angle between two vectors increases, the magnitude of their resultant decreases.

(c) When two vectors are in the same direction (parallel).


then 𝜃 = 00, cos 00 = 1 and sin 00 = 0

from eq.(3), R = √𝑃2 + 𝑄 2 + 2𝑝𝑄(1)

= √(𝑃 + 𝑄)2 = (P + Q)
𝑄×0
from eq.(4), tan 𝛼 = 𝑃+𝑄(1) = 0 or 𝛼 = 00

Thus for two vectors acting in the same direction, the magnitude of the resultant vector is equal

to the sum of the magnitudes of two vectors and acts along the direction of 𝑃⃗ and 𝑄

(d) When two vectors are acting in opposite directions (antiparallel).

then 𝜃 = 1800, cos 1800 = -1 and sin 1800 = 0.

From eq. (3), R = √𝑃2 + 𝑄 2 + 2𝑝𝑄(−1)

=√(𝑃 − 𝑄)2 = (P - Q) or (Q – P)
𝑄×0
From eq. (4), tan 𝛼 = 𝑃+𝑄(−1) = 0

or 𝛼 = 00 or 1800

Thus for two vectors acting in opposite directions, the magnitude of the resultant vector is equal to the
difference of the magnitudes of the two vectors and its direction is along the vector of larger magnitude.

(e) When two vectors are perpendicular to each other,

𝜃 = 900, sin 900 = 1

and cos 900 = 0

From eq(3).

R = √𝑃2 + 𝑄 2 + 2𝑝𝑄(0 = √𝑃2 + 𝑄 2


𝑄(1) 𝑄
From eq(4), tan 𝛼 = 𝑃+𝑄(0) = 𝑃

𝑄
or 𝛼 = tan-1(𝑃 )

(f) If the resultant of two vectors is perpendicular to any one of the vectors, then. The angle between
the two vectors is greater than 900
(g) The resultant vector is perpendicular to the vector having smaller magnitude.
𝑄𝑠𝑖𝑛𝜃
we know tan 𝛼 = 𝑃+𝑄 𝑐𝑜𝑠𝜃
𝑄𝑠𝑖𝑛𝜃
tan 900 = 𝑃+𝑄 𝑐𝑜𝑠𝜃
P + Q cos𝜃 = 0
−𝑃
cos𝜃 = 𝑄 and
R = √𝑄 2 − 𝑃2 ,
𝑃 𝑅 𝑃
sin ∅ = 𝑄, cos ∅ = 𝑄, tan ∅ = 𝑅.
The angle between 𝑃⃗ and 𝑄 ⃗ is 𝜃 = 900 + ∅
𝑅𝑚𝑎𝑥 𝑃+𝑄 𝑃 𝑅 +𝑅
(h) = 𝑃−𝑄 and 𝑄 = 𝑅𝑚𝑎𝑥 −𝑅𝑚𝑖𝑛
𝑅 𝑚𝑖𝑛 𝑚𝑎𝑥 𝑚𝑖𝑛
(i) If magnitudes of P and Q are equal and the angle between them is 𝜃 then their resultant is

R = √𝑃2 + 𝑄 2 + 2𝑝𝑄𝑐𝑜𝑠

R = √𝑃2 + 𝑄 2 + 2𝑃2 𝑐𝑜𝑠 𝜃[𝑃 = 𝑄]

R = √2𝑃2 + 2𝑃2 𝑐𝑜𝑠𝜃, R = √2𝑃2 (1 + 𝑐𝑜𝑠𝜃

𝜃 𝜃
R = √2𝑃2 (2𝑐𝑜𝑠 2 2), ∴R = 2P cos 2

and the resultant makes an angle ‘𝛼’ with P is


𝑄 𝑠𝑖𝑛𝜃 𝑃 𝑠𝑖𝑛𝜃
∴ tan 𝛼 = 𝑃+𝑄 𝑐𝑜𝑠𝜃 = 𝑃+𝑃 𝑐𝑜𝑠𝜃

𝑃 𝑠𝑖𝑛𝜃 𝑠𝑖𝑛𝜃
tan 𝛼 = 𝑃(1+𝑐𝑜𝑠𝜃 = 2𝑐𝑜𝑠2 𝜃/2

𝜃 𝜃
2 𝑠𝑖𝑛 𝑐𝑜𝑠 𝜃
2 2
∴ tan 𝛼 = = tan 2
2𝑐𝑜𝑠2 𝜃/2

𝜃 𝜃
∴ tan 𝛼 = tan 2, ∴𝛼=2

(a) If 𝜃 = 600, then R = √3 P and 𝛼 =300

(b) If 𝜃 = 900, then R = √2 P and 𝛼 =450

(c) If 𝜃 = 1200, then R = P and 𝛼 = 600



(J) ⃗ is 𝑛̂ = 𝑃⃗+𝑄
The unit vector parallel to the resultant of two vectors 𝑃⃗ & 𝑄 |𝑃⃗ ⃗ |
+𝑄

Application – 4.5

Magnitude of difference of two vectors is

⃗ | is
The magnitude of |𝑃⃗ − 𝑄

S = = √𝑃2 + 𝑄 2 − 2𝑃𝑄 𝑐𝑜𝑠 𝜃 and


𝑄 sin(1800 −𝜃) 𝑄 sin 𝜃
tan 𝛼 = 𝑃+𝑄 𝑐𝑜𝑠(1800 −𝜃) = 𝑃−𝑄 𝑐𝑜𝑠𝜃

⃗ | = 2P sin 𝜃
⃗ | then |𝑃⃗ − 𝑄
Note: If |𝑃⃗| = |𝑄
2

𝜃
∴ S = 2P sin 2

⃗ have same magnitude and 𝜃 is the angle between them


Note 4.19: When two vectors 𝑃⃗ and 𝑄

𝜃 Resultant 2P cos 𝜃/2 Difference 2P sin 𝜃/2


00 2P 0
600 √3 P P
900 √2 P √2 P
1200 P √3 P
1800 0 2P
Application-4.6

If 𝑣𝑖 is initial velocity of a particle and 𝑣𝑓 is its final velocity. Then change in its velocity is givenby ⃗⃗⃗⃗
∆𝑣 = 𝑣𝑓 -𝑣𝑖 .

⃗⃗⃗⃗ | = √𝑣𝑖2 + 𝑣𝑓2 − 2𝑣𝑖 𝑣𝑓 𝑐𝑜𝑠𝜃


|∆𝑣

Where ‘𝜃’ is the angle between initial and final velocities.

Application:

When a particle is performing uniform circular motion with a constant speed v, the magnitude of
change in velocity when it describes as angle 𝜃 at the centre is
𝜃
f∆𝑣 = 2vsin(2)

Application:

If velocity of a particle changes from 𝑣𝑖 to 𝑣𝑓 in time t then the acceleration of the particle is
⃗ 𝑓 −𝑣
𝑣 ⃗𝑖
given by 𝑎 =
𝑡

Application-4.9

⃗ are two sides and 𝑅⃗ ,𝑆 are two diagonals of a parallelogram then R2 + S2 = 2(P2 + Q2)
𝑃⃗ , 𝑄


𝑅⃗ = 𝑃⃗ + 𝑄

Magnitude of 𝑅⃗ is R = √𝑃2 + 𝑄 2 + 2𝑃𝑄𝑐𝑜𝑠𝜃

R2 = P2 + Q2 + 2PQcos 𝜃 …….(1)

𝑆 = 𝑃⃗ - 𝑄

Magnitude of 𝑆 is

S = √𝑃2 + 𝑄 2 + 2𝑃𝑄𝑐𝑜𝑠(𝜋 − 𝜃)

S = 𝑃2 + 𝑄 2 + 2𝑃𝑄𝑐𝑜𝑠 𝜃 ……..(2)

From (1) and (2) 𝑅 2 + 𝑆 2 = 2 (𝑃2 + 𝑄 2)

Note 4.20 : We can add a vector to another vector of same kind but we can’t add a vector quantity to a scalar
quantity.

TRIANGLE LAW OF VECTORS

“If two vectors are represented in magnitude and direction by the two sides of a triangle taken in one
order, their resultant vector is represented in magnitude and direction by the third side of the triangle taken in
reverse order”.

⃗ , inclined at an angle 𝜃 be acting on a particle at the same time. Let


Let the two vectors 𝑃⃗ and 𝑄
they be represented in magnitude and direction by two sides 𝑂𝐴 ⃗⃗⃗⃗⃗ and 𝐴𝐵
⃗⃗⃗⃗⃗ of triangle OAB, taken in the same
order shown in fig.

⃗⃗⃗⃗⃗
Then, according triangle law of vector addition, the resultant 𝑅⃗ is represented by the third side 𝑂𝐵
of triangle, taken in opposite order.

Magnitude of resultant 𝑅⃗ :

R = √𝑃2 + 𝑄 2 + 2𝑃𝑄𝑐𝑜𝑠𝜃

Direction of resultant R:
𝑄 𝑠𝑖𝑛𝜃
Tan 𝛼 = 𝑃+𝑄 cos 𝜃

Statement of triangle law when three forces keep a particle in equilibrium:

When three forces acting at a point can keep a particle in equilibrium the three forces can be
represented as the sides of a triangle taken in order both in magnitude and direction.

Suppose three force 𝐹1 , 𝐹2 , 𝐹3 are simultaneously acting at point O and the point is in equilibrium. Then the
three forces can be represented as three sides of a triangle. The triangle PQR is constructed by drawing parallel
lines to the directions in which the forces are applied. The magnitudes of the forces and the corresponding
sides of the triangle have equal ratio i.e.,
𝐹1 𝐹 𝐹
= 𝑃𝑅2 = 𝑄𝑃3
𝑃𝑄

POLYGON LAW OF VECTORS

Statement: If number of vectors are represented by the sides of a polygon both in magnitude and
direction taken in order, their resultant is represented by the closing side of the polygon taken in reverse
order in magnitude and direction.

(or)

If a number of coplanar forces are acting simultaneously at a point keep the particle in equilibrium, these
forces can be represented as the sides of a polygon taken in order both in magnitude and direction.

∴ 𝐹1 +𝐹2 +𝐹3 +𝐹4 +𝐹5 = 0

Note: If x is the side of a hexagon

ABCDEF as shown in figure

AB = x, AC = √3𝑥, AD = 2x

AE = √3𝑥, AF = x

EQUILIBRIANT

A vector having same magnitude and opposite in direction to that of the resultant of a number of
vectors is called the equilibriant. (or)

Negative vector of the resultant of a number of vectors is called the equilibriant (E).

If 𝐹1 +𝐹2 &𝐹3 are the three forces acting on a body, then their resultant force is

𝐹𝑅 = 𝐹1 +𝐹2 +𝐹3

∴ 𝐸⃗ = 𝐹𝑅 , 𝐸⃗ = -(𝐹1 +𝐹2 +𝐹3


Note:

(i) Single force cannot keep the particle in equilibrium#

(ii) Minimum number of equal coplanar forces required to keep the particle in equilibrium is two

(iii) Minimum number of unequal coplanar forces required to keep the particle in equilibrium is three

(iv) Minimum number of equal or unequal non coplanar forces required to keep the particle in equilibrium
is four.

Application-4.10 : Lami’s Theorem

“If three coplanar forces acting at a point keeps it in equilibrium, then each force is proportional to the
sine of the angle between the other two forces”

If F1, F2, F3 are the magnitudes of three forces, and 𝛼, 𝛽, 𝛾 are the angle between forces𝐹2 and
⃗⃗⃗𝐹3 , 𝐹3 and 𝐹1 and 𝐹1 and 𝐹2 respectively, as shown in fig. Then according to lami’s theorem

POSITION VECTOR AND DISPLACEMENT

Position vector of any point, with respect to an arbitrarily chosen origin, is defined as the vector
which connects the origin and the point and is directed towards the point.

Position vector of a point helps in locating the position of the point. Its magnitude gives the
distance between origin and the point. Consider a point ‘P’ having coordinates (x,y,z) as shown in
⃗⃗⃗⃗⃗ is called the position vector (𝑟), 𝑟 = x𝑖̂ + y𝑗̂+z𝑘̂
figure. If ‘O’ is the origin, then 𝑂𝑃
Magnitude of 𝑟 is |𝑟| = √𝑥 2 + 𝑦 2 + 𝑧 2
The unit vector along 𝑟 is given by
𝑟 ̂
𝑥𝑖̂+𝑦𝑗̂ +𝑧𝑘
𝑟̂ = |𝑟| =
√𝑥 2 +𝑦2 +𝑧 2
Displacement vector :

It 𝑟1 is initial position vector of the particle and 𝑟2 is final position vector of the particle then the
displacement of the particle is given by 𝑠 = 𝑟2 - 𝑟1 .

𝑠 = (x2 – x1) 𝑖̂ + (y2 – y1) 𝑗̂ + (z2 – z1) 𝑘̂

The magnitude of the displacement vector is


|𝑠| = AB = √(𝑥2 − 𝑥1 )2 + (𝑦2 − 𝑦1 )2 + (𝑧2 − 𝑧2 )2
Note 4.23: The position vector r of a particle P located in a plane with reference to the origin of an x-y
reference frame Fig. is given by r = x𝑖̂ + y𝑗̂; where x and y are components of r along x-, and y-axes or
simply they are the coordinates of the object

Suppose a particle moves along the curve shown by the thick line from P to P1 in time interval ∆t = t1-t.
Then the displacement is ∆r = r’- r and is directed from P to P1

We can write Eq. in a component form


∆r = (x’𝑖̂ +y’𝑗̂) - (x𝑖̂ +y𝑗̂) = 𝑖̂∆x + 𝑗̂∆y
Where ∆x = x’-x, ∆y = y’-y
MULTIPLICATION OF VECTORS

THE SCALAR PRODUCT

The scalar product or dot product of any two vectors 𝐴 and 𝐵 ⃗ , denoted as 𝐴 . 𝐵

⃗ ) is defined as the product of their magnitude with cosine of angle
(read 𝐴 dot 𝐵
between them. Thus. 𝐴 . 𝐵 ⃗ = AB cos 𝜃 {here 𝜃 is the angle between the vectors}

PROPERTIES:
It is always a scalar which is positive if angle between the vectors is acute (i.e. < 900)
and negative if angle between them is obtuse (i.e. 900 < 𝜃 ≤ 1800)

It is commutative, i.e., 𝐴 . 𝐵 ⃗ =𝐵 ⃗.𝐴


⃗ + 𝐶) = 𝐴 . 𝐵
It is distributive, i.e. 𝐴. (𝐵 ⃗ +𝐴.𝐶

As by definition 𝐴 . 𝐵 ⃗ = AB cos 𝜃. The angle between the vectors 𝜃 = cos-1[𝐴.𝐵]
𝐴𝐵
𝐴.𝐵 ⃗ = A (B cos 𝜃) = B (A cos 𝜃)
⃗ onto 𝐴 and A cos 𝜃 is the projection
Geometrically, B cos 𝜃 is the projection of 𝐵
of 𝐴 onto 𝐵⃗ as shown.
⃗ is the product of the magnitude of 𝐴 and the component of 𝐵
So 𝐴 . 𝐵 ⃗ along 𝐴 and vice versa.


⃗ along 𝐴 = B cos𝜃 = 𝐴.𝐵 = 𝐴̂. 𝐵
Component of 𝐵 ⃗
𝐴


⃗ = A cos𝜃 = 𝐴.𝐵 = 𝐴. 𝐵̂
Component of𝐴 along 𝐵 𝐵

Scalar product of two vectors will be maximum when cos 𝜃 = max = 1, i.e., 𝜃 = 00,
i.e., vectors are parallel  (𝐴 . 𝐵⃗ )max = AB
If the scalar product of two nonzero vectors vanishes then the vectors are perpendicular.
The scalar product of a vector by itself is termed as self dot product and is given by
2
(𝐴) = 𝐴. 𝐴 = A A cos 𝜃 = AAcos00 = A2  A = √𝐴. 𝐴
In case of unit vector 𝑛̂,
𝑛̂. 𝑛̂ = 1 × 1 x cos 00 = 1  𝑛̂. 𝑛̂ = 𝑖̂. 𝑖̂ = 𝑗̂. 𝑗̂ = 𝑘̂. 𝑘̂ = 1
In case of orthogonal unit vectors 𝑖̂. 𝑗̂ and 𝑘̂; 𝑖̂. 𝑗̂ = 𝑗̂. 𝑘̂ = 𝑘̂. 𝑖̂ = 0
= (𝑖̂ 𝐴𝑥 + 𝑗̂ 𝐴𝑦 + 𝑘̂ 𝐴𝑧 ). (𝑖̂ 𝐵𝑥 + 𝑗̂ 𝐵𝑦 + 𝑘̂ 𝐵𝑧 ) = [𝐴𝑥 𝐵𝑥 + 𝐴𝑦 𝐵𝑦 +𝐴𝑧 𝐵𝑧 ]
Example 70. If the Vectors 𝑃⃗ = a 𝑖̂ + a 𝑗̂ + 3 𝑘̂ and 𝑄 ⃗ = a 𝑖̂ - 2 𝑗̂ - 𝑘̂ are perpendicular to each other Find the
value of a?
Solution. If vector 𝑃⃗ and 𝑄 ⃗ are perpendicular
 𝑃⃗ . 𝑄⃗ = 0  (a 𝑖̂ + a 𝑗̂ + 3 𝑘̂). (a 𝑖̂ - 2 𝑗̂ - 𝑘̂) = 0
 a – 2a – 3 = 0
2
 a2 – 3a + a – 3 = 0
 a(a-3) + 1 (a-3) = 0  a = -1, 3
Example 71. Find the component of 3𝑖̂ + 4𝑗̂ along 𝑖̂ + 𝑗̂?

Solution. Componant of 𝐴 along 𝐵 ⃗ is given by 𝐴.𝐵 hence required component
𝐵
(3𝑖̂+4𝑗̂ ).(𝑖̂.𝑗) 7
= =
√2 √2
Example 72. ⃗ = 12𝑗 + 5𝑗 ?
Find angle between 𝐴 = 3𝑗̂ + 4𝑗̂ and 𝐵

𝐴.𝐵 (3𝑖̂+4𝑗̂ ).(12𝑖̂.5𝑗)
Solution. We have cos 𝜃 = =
𝐵 √32 +42 √122 +52
36+20 56 56
cos 𝜃 = = 65 𝜃 = cos-1 65
5 ×13

VECTOR PRODUCT
The vector product or cross product of any two vectors 𝐴 and 𝐵 ⃗ , denoted as
𝐴×𝐵 ⃗ (read 𝐴 𝑐𝑟𝑜𝑠𝑠 𝐵⃗ ) is defined as:
𝐴×𝐵 ⃗ = AB sin 𝜃 𝑛̂
Here 𝜃 is the angle between the vectors and the direction 𝑛̂ is given by the right-hand-thumb rule.

Right-Hand-Thumb Rule:

To find the direction of 𝑛̂, draw the two vectors 𝐴 and 𝐵 ⃗ with both the tails
coinciding. Now place your stretched right palm perpendicular to the plane
of 𝐴 and 𝐵⃗ in such a way that the fingers are along the vector 𝐴 and when the
figure are closed they to towards 𝐵⃗ . The direction of the
thumb gives the direction of 𝑛̂.

PROPERTIES:
Vector product of two vectors is always a vector perpendicular to the plane containing the two
⃗ , though the vectors 𝐴 and 𝐵
vectors i.e. orthogonal to both the vectors 𝐴 and 𝐵 ⃗ may or may not
be orthogonal.

Vector Product of two vectors is not commutative i.e. 𝐴 × 𝐵 ⃗ ≠𝐵 ⃗ × 𝐴.


But |𝐴 × 𝐵 ⃗ | =|𝐵
⃗ × 𝐴| = AB sin 𝜃
The vector product is distributive when the order of the vectors is strictly maintained i.e.
𝐴 × (𝐵 ⃗ +𝐶 ) = 𝐴 × 𝐵 ⃗ + 𝐴 × 𝐶.
The magnitude of vector product of two vectors will be maximum when sin 𝜃 = max = 1, i.e., 𝜃=900
|𝐴 × 𝐵 ⃗ | 𝑚𝑎𝑥 = AB
i.e., magnitude of vector product is maximum if the vectors are orthogonal.
The magnitude of vector product of two non-zero vectors will be minimum when |sin 𝜃| =
minimum = 0,i.e.,
𝜃 = 00 or 1800 and |𝐴 × 𝐵 ⃗ | 𝑚𝑖𝑛 = 0 i.e., if the vector product of two non-zero vectors vanishes,
the vectors are collinear
Note: When 𝜃 = 00 then vectors may be called as like vector or parallel vectors and when 𝜃 = 1800
then vectors may be called as unlike vectors or antiparallel vectors.
The self cross product i.e. product of a vector by itself vanishes i.e. is a null vector.
Note: Null Vector or zero vector : A vector of zero magnitude is called zero vector. The direction of a
zero vector is in determinate (unspecified).
𝐴 × 𝐴 = AA sin 00 𝑛̂ = ⃗0.
In case of unit vector 𝑛̂, 𝑛̂ × 𝑛̂ = ⃗0  𝑖̂ × 𝑖̂ = 𝑗̂ × 𝑗̂ = 𝑘̂ × 𝑘̂ = ⃗0

In case of orthogonal unit vectors 𝑖̂, 𝑗̂ and 𝑘̂


in accordance with right-hand-thumb-rule,
𝑖̂ × 𝑗̂ = 𝑘̂ 𝑗̂ × 𝑘̂ = 𝑖̂ 𝑘̂ × 𝑖̂ = 𝑗̂

𝑖̂ 𝑗̂ 𝑘̂
𝐴𝑦 𝐴𝑧 𝐴 𝐴𝑧 𝐴𝑥 𝐴𝑦
In terms of components 𝐴 ⃗ = |𝐴𝑥
×𝐵 𝐴𝑦 𝐴𝑧 | = 𝑖̂ | | -𝑗̂ | 𝑥 | + 𝑘̂ | |
𝐵𝑦 𝐵𝑧 𝐵𝑥 𝐵𝑧 𝐵𝑥 𝐵𝑦
𝐵𝑥 𝐵𝑦 𝐵𝑧

𝐴×𝐵 ⃗ = 𝑖̂ (𝐴𝑦 𝐵𝑧 - 𝐴𝑧 𝐵𝑦 ) + 𝑗̂(𝐴𝑧 𝐵𝑥 - 𝐴𝑥 𝐵𝑧 ) + 𝑘̂(𝐴𝑥 𝐵𝑦 -𝐴𝑦 𝐵𝑥 )


⃗ equal to |𝐴 × 𝐵
The magnitude of area of the parallelogram formed by the adjacent sides of vectors 𝐴 and 𝐵 ⃗|

SOLVED EXAMPLES
⃗ is downwards. Find the direction of 𝐴 × 𝐵
Example 73. 𝐴 is Eastwards and 𝐵 ⃗?
Solution. ⃗ is along North.
Applying right hand thumb rule we find that 𝐴 × 𝐵

⃗ = |𝐴 × 𝐵
Example 74. If 𝐴 . 𝐵 ⃗ |, find angle between 𝐴 𝑎𝑛𝑑 𝐵

Solution. 𝐴.𝐵 ⃗ = |𝐴 × 𝐵⃗ | AB cos 𝜃 = AB sin 𝜃 tan 𝜃 = 1  𝜃 = 450

Example 75. Two vectors 𝐴 𝑎𝑛𝑑 𝐵 ⃗ are inclined to each other at an angle . Find a unit vector which is
perpendicular to both
𝐴 𝑎𝑛𝑑 𝐵 ⃗
Solution. 𝐴 × 𝐵 ⃗ = AB sin 𝜃 𝑛̂

𝐴×𝐵
 𝑛̂ = here 𝑛̂ is perpendicular to both 𝐴 𝑎𝑛𝑑 𝐵
𝐴𝐵 𝑠𝑖𝑛𝜃
⃗.
Example 76. Find 𝐴 × 𝐵⃗ if 𝐴 = 𝑖̂ - 2 𝑗̂ + 4 𝑘̂ and 𝐵
⃗ = 2 𝑖̂ - 𝑗̂ + 2 𝑘̂.
𝑖̂ 𝑗̂ 𝑘̂
Solution. 𝐴 × 𝐵⃗ = |1 −2 4| = 𝑖̂(-4-(-4)) - 𝑖̂ (2-12) + 𝑘̂(-1 -6)) = 10 𝑗̂ + 5 𝑘̂
3 −1 2

Additional Problems
Problem 1. Find the value of
(a) sin (-𝜃) (b) cos ( - 𝜃) (c) tan(-𝜃)
𝜋 𝜋 𝜋
(d) cos ( 2 – 𝜃) (e) sin ( 2 + 𝜃) (f) cos ( 2 + 𝜃)
3𝜋
(g) sin (𝜋 – 𝜃) (h) cos (𝜋 − 𝜃) (i) sin ( – 𝜃)
2
3𝜋 3𝜋 3𝜋
(j) cos ( – 𝜃) (k) sin ( + 𝜃) (l) cos ( + 𝜃)
2 2 2
𝜋 𝜋
(m) tan ( 2 – 𝜃) (n) cot ( 2 – 𝜃)
Answer : (a) – sin 𝜃 (b) cos 𝜃 (c) – tan 𝜃 (d) sin 𝜃 (e) cos 𝜃 (f) - sin 𝜃
(g) sin 𝜃 (h) - cos 𝜃 (i) – cos 𝜃 (j) - sin 𝜃 (k) - cos 𝜃 (l) sin 𝜃
(m) cot 𝜃 (n) tan 𝜃
Problem 2. (i) For what value of m the vector 𝐴 = 2𝑖̂ + 3𝑗̂ - 6𝑘̂ is perpendicular to 𝐵⃗ = 3𝑖̂ - m𝑗̂ + 6𝑘̂
(ii) Find the components of vector 𝐴 = 2𝑖̂ + 3𝑗̂ along the direction of 𝑖̂ + 𝑗̂ ?
Answers: (i) m = -10
5
(ii) .
√2
Problem 3. (i) 𝐴 is North – East and 𝐵 ⃗ is down wards, find the direction of 𝐴 × 𝐵
⃗.
(ii) Find 𝐵 ⃗ × 𝐴 if 𝐴 = 3𝑖̂ - 2𝑗̂ + 6𝑘̂ and 𝐵
⃗ = 𝑖̂ - 𝑗̂ + 𝑘̂.
Answers: (i) North-West
(ii) -4𝑖̂ - 3𝑗̂ + 𝑘̂.

1. A person runs along a circular path of radius 5 m. If he completes half of the circle find the
magnitude of the displacement vector, How far the person ran?
(a) 10 m, 5𝜋 m (b) 5 𝜋m, 10 m (c) 5𝜋 m, 19m (d) 14 m , 10 𝜋m
2. A car makes a displacement of 100 m towards east and then 200 m towards north.
Find the magnitude and direction of the resultant.
(a) 223.7m, tan-1(2), N of E (b) 223.7m, tan-1(2), E of N
-1
(c) 300m, tan (2), N of E (d) 100 m, tan-1(2), N of E
3. If 𝐴 = 3𝑖̂ - 4𝑗̂ and 𝐵 ⃗ = -𝑖̂ - 4𝑗̂, calculate the direction of 𝐴 + 𝐵 ⃗.
-1
(a) tan (4) with + x- axis in clock wise
(b) tan-1(4) with - x- axis in clock wise
(c) tan-1(4) with + x- axis in anti clock wise
(d) tan-1(4) with - x- axis in anti clock wise
4. Two vectors are given by 𝑎 = -2𝑖̂ + 𝑗̂ - 3𝑘̂ and 𝑏⃗ = 5𝑖̂ + 3𝑗̂ - 2𝑘̂. If 3𝑎+2𝑏⃗ - 𝑐 = 0 then third vector 𝑐 is
(a) 4𝑖̂ + 9𝑗̂ - 13𝑘̂ (b) -4𝑖̂ - 9𝑗̂ + 13𝑘̂ (c) 4𝑖̂ - 9𝑗̂ - 13𝑘̂ (d) 2𝑖̂ - 3𝑗̂ - 13𝑘̂
-1 -1
5. One of the rectangular components of a velocity of 20 ms is 10 ms . Find the other component.
(a) 10√3 ms-1 (b) 20√3 ms-1 (c) 5√3 ms-1 (d) 35√3 ms-1
0
6. A car weighing 100kg is on a slope that makes an angle 30 with the horizontal. The component
of car’s weight parallel to the slope is
(g = 10ms-2)
(a) 500 N (b) 1000 N (c) 15,000 N (d) 20,000 N
7. To go from town A to town B a plane must fly about 1780 km at an angl of 300 West of North. How far
north of A is B?
(a) 1542 km (b) 1452 km (c) 1254 km (d) 11 km
8. Two vectors a and b have equal magnitudes of 12 units. These vectors are making angles 300 and 1200
with the x axis respectively. Their sum is 𝑟. Find the x and y components of 𝑟.
(a) (6√3-6), (6 + 6√3) (b) (6√3 + 6), (6 + 6√3)
(c) (6√3-6), (6 - 6√3) (d) (6√3 + 6), (6 - 6√3)
9. Angle (in rad) made by the vector √3𝑖̂ + 𝑗̂ with the x-axis is
(a) 𝜋/6 (b) 𝜋/4 (c) 𝜋 / 3 (d) 𝜋 /5
10. A bird moves in such a way that it has a displacement of 12 m towards east, 5 m towards north and 9 m
vertically upwards. Find the magnitude of its displacement.
(a) 5√2 m (b) 5√10 m (c) 5√5 m (d) 5 m
4 1 3
11. The direction cosines of a vector 𝐴 are cos 𝛼 = 5√2, cos 𝛽 = and 𝛾 = 5√2 , then the vector 𝐴 is
√2
(a) 4𝑖̂ + 𝑗̂ + 3𝑘̂ (b) 4𝑖̂ + 5𝑗̂ + 3𝑘̂ (c) 4𝑖̂ - 5𝑗̂ - 3𝑘̂ (d) 𝑖̂ + 𝑗̂ - 𝑘̂
12. If 𝑃⃗ = 𝑖̂ + 2𝑗̂ + 6𝑘̂, its direction cosines are
1 2 6 1 2 6
(a) 41, 41 and 41 (b) , and
√41 √41 √41
3 8 7
(c) , , (d) 1,2 and 6
√41 √41 √41
13. If 𝐴 = 2𝑖̂ - 3𝑗̂ + 4𝑘̂ its components in yz plane and zx plane are respectively
(a) √13 and 5 (b) 5 and 2√5
(c) 2√5 and √13 (d) √13 and √29
14. If vectors 𝐴 and 𝐵 are 3𝑖̂ - 4𝑗̂ + 5𝑘 and 2𝑖̂ + 3𝑗̂ - 4𝑘̂ respectively find the unit vector parallel to 𝐴 + 𝐵
⃗ ̂ ⃗.
(5𝑖 −𝑗 + 𝑘) (5𝑖+𝑗 + 𝑘) (5𝑖+𝑗 + 𝑘) (5𝑖−𝑗− 𝑘)
(a) (b) (c) (d) 27
√27 √27 27
15. A vector 3𝑖̂ + 4𝑗̂ rotates about its tail through an angle 370 in anticlockwise direction then the new
vector is
(a) -3𝑖̂ + 4𝑗̂ (b) 3𝑖̂ - 4𝑗̂ (c) 5𝑗̂ (d) 5𝑖̂
16. Maximum and minimum magnitudes of the resultant of two vectors of magnitudes P and Q are
found to be in the ratio 3 : 1. Which of the following relations is true?
(a) P = Q (b) P = 2Q (c) P = 4Q (d) P = Q/3
17. The resultant of two equal forces is 141.4N when they are mutually perpendicular.When they are
inclined at an angle 1200, then the resultant force will be
(a) 100 N (b) 141.4 N (c) 196 N (d) Zero
18. ⃗ ⃗
The magnitudes of two vectors 𝑃 and 𝑄 differ by 1. The magnitude of thei resultant makes and angle
of tan-1(3/4) with P. The angle between P and Q is
(a) 450 (b) 00 (c) 1800 (d) 900
19. Given a + b + c + d = 0 which of the following statement is incorrect
(a) a,b,c and d must each be a null vector.
(b) The magnitude of (a+c) equals the magnitude of (b+d)
(c) The magnitude of a can never be greater than the sum of the magnitudes of b,c and d.
(d) b + c must lie in the plane of a and b if a and d are not collinear, and in the line of a and d, if
they are collinear
20. The resultant of two forces 2P and √2 P is √10 P. The angle between the forces is
(a) 300 (b) 600 (c) 450 (d) 900
21. Eleven forces each equal to 5N act on a particle simultaneously. If each force makes an angle 300
with the next one, the resultant of all forces is
(a) 15N (b) 55 N (c) 5 N (d) zero
22. Which of the following sets of forces acting simultaneously on a particle keep it in equilibrium?
(a) 3N, 5N, 10N (b) 4N, 7N, 12N (c) 2N, 6N, 5N (d) 5N, 8N, 1N
23. The resultant of two forces 1 and P is perpendicular to ‘1’ and equal to 1. What is the
value of ‘P’ and angle between the forces
(a) √2 N, 1350 (b) √2 N, 1500 (c) 2 N , 1200 (d) 2N, 1500
24. If the sum of two unit vectors is also a vector of unit magnitude, the magnitude of the difference
of the two unit vectors is
(a) 1 unit (b) 2 units (c) √3 units (d) Zero
25. ⃗ ⃗ ⃗ ⃗ ⃗ 2 2
If 𝑃 + 𝑄 = 𝑅 and 𝑃 - 𝑄 = 𝑆, then R + S is equal to
(a) P2 + Q2 (b) 2 (P2 - Q2) (c) 2 (P2 + Q2) (d) 4PQ
26. The greater and least resultant of two forces are 7 N and 3 N respectively. If each of the force is
increased by 3N and applied at 600. The magnitude of the resultant is
(a) 7 N (b) 3 N (c) 10 N (d) √129 N
27 Two forces are such that the sum of their magnitudes is 18 N, the resultant is √228 when they
are at 1200. Then the magnitude of the forces are
(a) 12N, 6N (b) 13N, 5N (c) 10N, 9N (d) 16N, 2N

28. Figure shows three vectors 𝑎, 𝑏⃗ and 𝑐, where R is the midpoint of PQ. Then which of the
following relations is correct?
(a) 𝑎+ 𝑏⃗ = 2 𝑐 (b) 𝑎+ 𝑏⃗ = 𝑐
(c) 𝑎 - 𝑏⃗ = 2 𝑐 (d) 𝑎 - 𝑏⃗ = 𝑐

29. ⃗ is 𝑅⃗ , If the magnitude of 𝑄


The resultant of two vectors 𝑃⃗ and 𝑄 ⃗ is doubled, the new
resultant becomes perpendicular to 𝑃⃗, then the magnitude of 𝑅⃗ is
𝑃 2 −𝑄 𝑃+𝑄 𝑃
(a) 2𝑃𝑄
(b) 𝑃−𝑄 (c) Q (d) 𝑄
30. If ‘O’ is at equilibrium then the values of the tension T1 and T2 are, x, y, if 20 N is vertically
down. Then x, y are

(a) 20 N 30 N
(b) 20√3N, 20 N
(c) 20√3N, 20√3N
(d) 10 N, 30 N
31. A body of mass √3 kg is suspended by a string to a rigid support. The body is pulled
horizontally by a force F until the string makes an angle of 300 with the vertical. The value of F and
tension in the string are
(a) 9.8N, 9.8 N (b) 9.8N, 19.6N
(c) 19.6N, 19.6N (d) 19.6N, 9.8N
32. Two light strings of length 4 cm and 3 cm are tied to a bob of weight 500 gm. The free ends of
the strings are tied to pegs in the same horizontal line and separated by 5 cm. The ratio of tension
in the longer string to that in the shorter string is
(a) 4 : 3 (b) 3 : 4 (c) 4 : 5 (d) 5 : 4
33. A boy is hanging from a horizontal branch of a tree. The tension in the arms will be maximum
when the angle between the arms is
(a) 00 (b) 300 (c) 600 (d) 1200
34. Resultant of two vectors of magnitudes P and Q is of magnitude ‘Q’. If the magnitude
of 𝑄⃗ is doubled now the angle made by new resultant with 𝑃⃗ is
(a) 300 (b) 900 (c) 600 (d) 1200
35. When forces F1, F2,F3 are acting on a particle of mass m such that F2 and F3 are mutually
perpendicular, then the particle remains stationary, If the force F1 is now removed then acceleration
of the particle is
(a) F1 / m (b) F1 F3 / F1 (c) (F2- F3)/m (d) F2 /m
36. The square of the resultant of two forces 4N and 3N exceeds the square of the
resultant of the two forces by 12 when they are mutually perpendicular. The angle
between the vectors is
(a) 300 (b) 600 (c) 900 (d) 1200
37. A person moving on a motor cycle in a ground takes a turn through 600 on his left after every 50m.
Then find the magnitude of displacement suffered by him after 9th turn
(a) 100m

(b) 50 m

(c) 50 √3m

(d) 20 m
38. If four forces act at a point ‘O’ as shown in the figure and if O is in equilibrium then the value
of ‘𝜃’ & ‘P’ are
(a) 150, 10√2 N
(b) 450, 10 N
(c) 750, 10√2 N
(d) 900, 20 N

39. The rectangular components of a vector lying in xy plane are (n+1) and 1. If coordinate
system is turned by 600. They are n & 3 respectively the value of ‘n’
(a) 2 (b) 3 (c) 2.5 (d) 3.5
40. A mass M kg is suspended by a weightless string. The horizontal force required to hold the mass
at 600 with the vertical is
𝑀𝐺
(a) Mg (b) Mg√3 (c) Mg(√3 + 1) (d)
√3
TRIGONOMETRY

MEASUREMENT OF ANGLE AND RELATIONSHIP BETWEEN DEGREES AND RADIAN

In navigation and astronomy, angles are measured in degrees, but in calculus it is best to
use units called radians because of the way they simplify later calculations. 
Let ACB be a central angle in a circle of radius r, as in figure.
Then the angle ACB or θ is defined in radius as 
𝐴𝑟𝑐 𝑙𝑒𝑛𝑔𝑡ℎ ̂
𝐴𝐵 A
𝜃 = 𝑅𝑎𝑑𝑖𝑢𝑠  𝜃= 𝑟
r
C

If r = 1 then 𝜃 = AB

The radian measure for a circle of unit radius of angle ACB is defined to be the length of the
circular arc AB. Since the circumference of the circle is 2𝜋 and one complete revolution of a circle is
3600, the relation between radians and degrees is given by : 𝜋 radians = 1800

Angle Conversion formulas


𝜋
𝜋 Degrees to radians : multiply by 180
1 degree = 180 (  0.02) radian

1 radian 57 degrees Radians to degrees : multiply by


180
𝜋
Solved Examples

Example 5. (i) Convert 450 to radians.


𝜋
(ii) Convert rad to degrees.
6
𝜋 𝜋
Solution : (i) 45. 180 = 4 rad

𝜋 180
(ii) 6 . = 300
𝜋

Example 6. Convert 300 to radians.


𝜋 𝜋
Solution : 300 × 180 = 6 rad

𝜋
Example 7. Convert 3 rad to degrees.

𝜋 180
Solution: × =600
3 𝜋

Standard values
𝜋 𝜋 𝜋
(1) 300 = 6 rad (2) 450 = 4 rad (3) 600 = 3 rad
𝜋 2𝜋 3𝜋
(4) 900 = 2 rad (5) 1200 = rad (6) 1350 = rad
3 4
5𝜋
(7) 1500 = 6 rad (8) 1800 = 𝜋 rad (9) 3600 = 2𝜋 rad
(Check these values yourself to see that the satisfy the conversion formulae)
MEASUREMENT OF POSITIVE AND NEGATIVE ANGLES

An angle in the xy-plane is said to be in standard position if its vertex lies at the origin and its initial ray
lies along the positive x-axis (Fig.). Angles measured counterclockwise from the positive x-axis are
assigned positive measures ; angles measured clockwise are assigned negative measures.

y y y y
5

2
x x x x
9 3
3 
4 4

2.3 SIX BASIC TRIGONOMETRIC FUNCTIONS


The trigonometric function of a general angle 𝜃 are defined in terms
of x, y, and r.
𝑜𝑝𝑝 𝑦 ℎ𝑦𝑝 𝑟
Sine: sin 𝜃 = ℎ𝑦𝑝 = 𝑟 Cosecant : cosec 𝜃 =𝑜𝑝𝑝 = 𝑦
𝑎𝑑𝑗 𝑥 ℎ𝑦𝑝 𝑟
Cosine : cos 𝜃 = ℎ𝑦𝑝 = 𝑟 Secant : sec 𝜃 = 𝑎𝑑𝑗 = 𝑥
𝑜𝑝𝑝 𝑦 𝑎𝑑𝑗 𝑥
Tangent : tan 𝜃 = 𝑎𝑑𝑗 = 𝑥 Cotangent : cot𝜃 = 𝑜𝑝𝑝 = 𝑦

VALUES OF TRIGONOMETRIC FUNCTIONS


If the circle in (Fig.above) has radius r = 1, the equations defining sin 𝜃 and cos 𝜃 become
cos 𝜃 = x, sin 𝜃 = y
We can then calculate the values of the cosine and sine directly from the coordinates of P.

Solved Examples

Example 8. Find the six trigonometric ratios from given figure

5
4

3

𝑜𝑝𝑝 4 ℎ𝑦𝑝 3
Solution: sin 𝜃 = ℎ𝑦𝑝 = 5 cos 𝜃 =𝑜𝑝𝑝 = 5
𝑎𝑑𝑗 4 ℎ𝑦𝑝 5
tan 𝜃 = ℎ𝑦𝑝 = 3 cosec 𝜃 = 𝑎𝑑𝑗 = 4
𝑜𝑝𝑝 5 𝑎𝑑𝑗 3
sec 𝜃 = 𝑎𝑑𝑗 = 3 cot𝜃 = 𝑜𝑝𝑝 = 4
Example 9. Find the sine and cosine of angle 𝜃 shown in the unit circle if coordinate of point p are as shown.
1  1 3 y
Solution: cos 𝜃 = x-coordinate of P = - 2   ,  P
 2 2 
√3
sin 𝜃 = y-coordinate of P = 3 
2 1
2
1
2

RULES FOR FINDING TRIGONOMETRIC RATIO OF ANGLES GREATERTHAN 90o


Step 1 → Identify the quadrant in which angle lies.
Step 2 →
(a) If angle = (n𝜋 ± 𝜃) where n is an integer. Then trigonometric function of (n𝜋 ± 𝜃)
= same trigonometric function of 𝜃 and sign will be decided by CAST Rule.
y
I quadrant II quadrant
THE CAST RULE S A
sin positive all positive

x
A useful rule for remembering when the basic T C
tan positive cos positive
trigonometric functions are positive and negative
is the CAST rule. If you are not very enthusiastic III quadrant IV quadrant
about CAST. You can remember it as ASTC
(After school to college)
𝜋
(b) If angle [(2𝑛 + 1) 2 ± 𝜃] where n is an integer. Then
𝜋
Trigonometric function of [(2𝑛 + 1) 2 ± 𝜃] = complimentary trigonometric function of 𝜃
and sign will be decided by CAST Rule.

Values of sin 𝜽, cos 𝜽 and tan 𝜽 for some standard angles.


Degree 0 30 37 45 53 60 90 120 135 180
Radians 0 𝜋/6 37𝜋/180 𝜋/4 53 𝜋/180 𝜋/3 𝜋/2 2 𝜋/3 3 𝜋/4 𝜋
sin 𝜃 0 1/2 3/5 1/√2 4/5 √3/2 1 √3/2 1/√2 0
cos 𝜃 1 √3/2 4/5 1/√2 3/5 1/2 0 -1/2 -1/√2 -1
tan 𝜃 0 1/√3 3/4 1 4/3 √3 ∞ -√3 -1 0

Solved Examples
Example 10. Evaluate sin 1200
√3
Solution: sin 1200 = sin (900 + 300) = cos 300 = 2
√3
Aliter sin 1200 = sin (1800-600) = sin 600 = 2
Example 11. Evaluate cos 1350
1
Solution: cos 1350 = cos (900 + 450) = -sin 450 = −
√2
Example 12. Evaluate cos 2100
√3
Solution: cos 2100 = cos (1800 + 300) = - cos 300 = - 2
Example 13. Evaluate tan 2100
1
Solution: tan 2100 = tan (1800 + 300)= tan 300 =
√3
GENERAL TRIGONOMETRIC FORMULAS:

1. cos2 𝜃 + sin2 𝜃 = 1 2. Cos (A + B) = cos A cos B – sin A sin B


1 + tan2 𝜃 = sec2 𝜃. Sin (A + B) = sin A cos B + cos A sin B
tan 𝐴+𝑡𝑎𝑛𝐵
1 + cot 𝜃 = cosec2 𝜃 tan (A + B) = 1−tan 𝐴 tan 𝐵
3. sin 2𝜃 = 2 sin 𝜃cos 𝜃 ; cos2 𝜃 = cos2 𝜃- sin2 𝜃 = 2cos2 𝜃 - 1 = 1 – 2 sin2 𝜃
1+𝑐𝑜𝑠2𝜃 1+𝑐𝑜𝑠2𝜃
cos2 𝜃 = 2 ; sin2 𝜃 = 2
4. sine rule for triangles 5. cosine rule for triangles


a c
a c
 
b 
b

𝑠𝑖𝑛𝛼 𝑠𝑖𝑛𝛽 𝑠𝑖𝑛𝛾


= = c2 = a2 + b2 – 2ab cos 𝜃
𝑎 𝑎 𝑐
DIFFERENTIATION

FINITE DIFFERENCE
The finite difference between two values of a physical quantity is represented by ∆ notation.
For example:
Difference in two values of y is written as ∆y as given in the table below

y2 100 100 100


y1 50 99 99.5
∆y = y2- y1 50 1 0.5

INFINITELY SMALL DIFFERENCE:


The infinitely small difference means very-very small difference. And this difference is
represented by ‘d’ notation instead of ∆
For example infinitely small difference in the values of y is written as ‘dy’
If y2 = 100 and y1 = 99.99999999……
then dy = 0.000000…………00001

DEFINITION OF DIFFERENTIATION
Another name for differentiation is derivative. Suppose y is a function of x or y = f(x)
Differentiation of y with respect to x is denoted by symbol f ’(x)
𝑑𝑦
Where f ’(x) = 𝑑𝑥
dx is very small change in x and dy is corresponding very small change in y.

NOTATION: There are many ways to denote the derivative of a function y = f(x),
Besides f’(x), the most common notations are these:
Nice and brief but does not name the independent
y “y prime” or “y dash”
variable
𝑑𝑦
“dy by dx”for Names the variables and uses d derivative
𝑑𝑥
𝑑𝑓
“df by dx of f” Emphasizes the function’s name
𝑑𝑥
𝑑 Emphasizes the idea that differentiation is an operation
f(x) “d by dx of f”
𝑑𝑥 performed on f.
Dx f “dx of f” A common operator notation
One of Newton’s notations, now common for time
Y “y dot” derivatives
𝑑𝑦
i.e. 𝑑𝑡 .
Most common notation, it names independent variable
f '(x) f dash xthe and
Emphasize the function’s name

SLOPE OF A LINE
It is the tan of angle made by a line with the positive direction of x-axis, measured in
anticlockwise direction.
Slope = tan 𝜃 (In 1st quadrant tan 𝜃 is +ve & 2nd quadrant tan 𝜃 is –ve)
In Figure – 1 slope is positive In Figure – 2 slope is negative
𝜃 < 900 (1st quadrant) 𝜃 > 900 (2nd quadrant)
y

 
x x

Figure  1 Figure  2

AVERAGE RATES OF CHANGE:


Given an arbitrary function y = f(x) we calculate the average rate
of change of y with respect to x over the interval (x, x + ∆x) by
dividing the change in value of y, i.e. ∆y = f(x + ∆x) – f(x),
by length of interval ∆x over which the change occurred.
The average rate of change of y with respect to x over the
∆y 𝑓(𝑥−∆x)−f(x)
interval [x, x + ∆x] = ∆x = ∆x
∆y 𝑄𝑅
Geometrically ∆x = 𝑃𝑅 = tan 𝜃 = Slope of the line PQ therefore
we can say that average rate of change of y with respect to x is
equal to slope of the line joining P & Q.

THE DERIVATIVE OF A FUNCTION


∆y 𝑓(𝑥−∆x)−f(x)
We know that, average rate of change of y w.r.t is ∆x = ∆x
If the limit of this ratio exists as ∆x → 0, then it is called the derivative of given function
f(x) and is denoted as
𝑑y 𝑓(𝑥+∆x)−f(x)
f '(x) = 𝑑x= lim
x 0 ∆x

GEOMETRICAL MEANING OF DIFFERENTIATION


The geometrical meaning of differentiation is very much useful in the analysis of graphs in
physics. To understand the geometrical meaning of derivatives we should have knowledge of
secant and tangent to a curve.
Secant and tangent to a curve
Secant: A secant to a curve is a straight line, which intersects the curve at any two points

Tangent:-
A tangent is a straight line, which touches the curve at a particular point. Tangent is a limiting
case of secant which intersects the curve at two overlapping points.
In the figure-I shown, if value of ∆x is gradually reduced then the point
Q will move nearer to point P. If the Process is continuously repeated
(Figure - 2) value of ∆x will be infinitely small and secant PQ to
the given curve will become a tangent at point P .
 y  𝑑y
Therefore x 0   = = tan 𝜃
 x  𝑑x
𝑑y
We can say that differentiation of y with respect to x,i.e. (𝑑x) is
𝑑y
Equal to slope of the tangent at point P(x,y) or tan 𝜃 = 𝑑x
(From fig. 1, the average rate of change of y from x to
x +∆x is identical with the slope of secant PQ.)

RULES FOR DIFFERENTATION

RULE NO.1: DERIVATIVE OF A CONSTANT

The first rule of differentiation is that the derivative of every constant function is zero.
𝑑
If c is constant, then 𝑑x c = 0.

𝑑 𝑑 1 𝑑
Example 14 (8) = 0, (− 2) = 0, (√3) = 0
𝑑x 𝑑x 𝑑x

RULE NO. 2: POWER RULE


𝑑
If n is a real number, then 𝑑x xn = nxn-1
To apply the power Rule, we subtract 1 from the original exponent (n) and multiply the result by n.

Example 15. F x x2 x3 x4 …..

f’ 1 2x 3x2 4x3 …..

Example 16
𝑑 1 𝑑 1
(i) 𝑑x (𝑥) =𝑑x (x-1) = (-1) x-2 = 𝑥 2
𝑑 4 𝑑 12
(ii) 𝑑x (𝑥 3 ) =4𝑑x (x-3) = 4(-3) x- 4 = 𝑥 4

𝑑 1 1
Example 17. (a) (x1/2) = 2 x-1/2 =2
𝑑x √𝑥

Function defined for x ≥ 0 derivative defined only for x > 0


𝑑 1
(b) 𝑑x(x1/5) = 5 x- 4/5

Function defined for x ≥ 0 derivative not defined at x = 0

RULE NO. 3 : THE CONSTANT MULTIPLE RULE


𝑑 𝑑𝑢
If u is a differentiable function of x, and c is a constant, then 𝑑x (cu) = c 𝑑x

𝑑
In particular, if n is a positive integer, then (cxn) = cn xn-1
𝑑x
Example 18 The derivative formula
𝑑
(3x2) = 3(2x) = 6x
𝑑x
Says that if we rescale the graph of y = x2 by multiplying each
y-coordinate by 3, then we multiply the slope at each point by 3.

Example 19 A useful special case


The derivative of the negative of a differentiable function is the negative of the function’s
derivative.
Rule 3 with c = -1 gives.
𝑑 𝑑 𝑑 𝑑
(-u) = 𝑑x(-1.u) = -1. 𝑑x(u) = 𝑑x(u)
𝑑x

RULE NO. 4 : THE SUM RULE

The derivative of the sum of two differentiable functions is the sum of their derivatives.

If u and v are differentiable functions of x, then their sum u + v is differentiable at every point
where u and v are both differentiable functions is their derivatives.
𝑑 𝑑 𝑑𝑣 𝑑𝑢 𝑑𝑣
(u – v) = [u + (-1)v] = = -
𝑑x 𝑑x 𝑑x 𝑑x 𝑑x
The Sum Rule also extends to sums of more than two functions, as long as there are only
finitely many functions in the sum, If u1, u2, …. un are differentiable at x, then so is u1+u2+ …. +
𝑑 𝑑𝑢 𝑑𝑢 𝑑𝑢
un, and 𝑑x(u1+u2+ …. + un) = 𝑑x1+ 𝑑x2 + …..+ 𝑑x𝑛

Example 20
4
(a) y = x4 + 12x (b) y = x3 + 3 x2 – 5x + 1

𝑑𝑦 𝑑 𝑑 𝑑𝑦 𝑑 𝑑 4 𝑑 𝑑
= 𝑑x (x4) + 𝑑x+(12x) = 𝑑x (x3) + 𝑑x+(3 𝑥 2 ) - 𝑑x(5x)+ 𝑑x(1)
𝑑x 𝑑x
4 8
= 4x3+ 12 = 3x2 + 3 . 2x – 5 + 0 = 3x2 + 3 x – 5
Notice that we can differentiate any polynomial term by term, the way we differentiated the
polynomials in above example.

RULE NO. 5: THE PRODUCT RULE


𝑑 𝑑𝑣 𝑑𝑢
If u and v are differentiable at x, then so is their product uv, and 𝑑x (uv) = u 𝑑x + 𝑣 𝑑x .
The derivative of the product uv is u times the derivative of v plus v times the derivative of u.
In prime notation (uv)’= uv’ + vu’.
While the derivative of the sum of two functions is the sum of their derivatives, the derivative
of the product of two functions is not the product of their derivatives. For instance,
𝑑 𝑑 2 𝑑 𝑑
(x.x) = (x ) = 2x, while (x). (x) = 1.1 = 1.
𝑑x 𝑑x 𝑑x 𝑑x
2 3
Example 21 Find the derivatives of y = (x +1) (x + 3).
Solution: From the product Rule with u = x2 + 1 and v = x3 + 3, we find
𝑑
[(x2 + 1) (x3+3)] = (x2 + 1) (3x2)+ (x3+3) (2x)
𝑑x
= 3x4 + 3x2 + 2x4 + 6x = 5x4 + 3x2 + 6x.
Example can be done as well (perhaps better) by multiplying out the original expression for y
and differentiating the resulting polynomial.
We now check : y = (x2 + 1) (x3+3) = x5 + x3 + 3x2 + 3
𝑑𝑦
= 5x4 + 3x2 + 6x.
𝑑x
This is in agreement with our first calculation.
There are times, however, when the product Rule must be used. In the following examples. We
have only numerical values to work with.
Example 22 Let y = uv be the product of the functions u and v. Find y’(2) if u (2)= 3, u’(2) = -4, v(2) = 1, and
v’(2) = 2.
Solution : From the Product Rule, in the form
y’ = (uv)’ = uv’ + vu’ ,
we have y’(2) = u(2) v’(2) + v)2) u’(2) = (3) (2) + (1) (-4) = 6 - 4=2

RULE NO. 6: THE QUOTIENT RULE


If u and v are differentiable at x, and v(x) = 0, then the quotient u/v is differentiable at x,
𝑑𝑢 𝑑𝑣
𝑑 𝑢 𝑣 𝑢
and 𝑑x (𝑣 ) = 𝑑𝑣𝑣2 𝑑𝑥
Just as the derivative of the product of two differentiable functions is not the product of their
derivatives, the derivative of the quotient of two functions is not the quotient of their derivatives.
𝑡 2 −1
Example 23 Find the derivative of y = 𝑡 2 +1
Solution: We apply the Quotient Rule with u = 𝑡 2 − 1 and v = 𝑡 2 + 1:
𝑑𝑢
𝑑𝑦 (𝑡 2 +1).2𝑡(𝑡 2 −1).2𝑡 𝑑 𝑢 𝑣( )− 𝑢(𝑑𝑣/𝑑𝑡)
𝑑x
= (𝑡 2 +1)2
 𝑑t 𝑣
( )= 𝑑𝑡
𝑣2

2𝑡 3 +2𝑡−2𝑡 3 +2𝑡 4𝑡
= (𝑡 2 +1)2
= (𝑡 2 +1)2

RULE NO. 7 DERIVATIVE OF SINE FUNCTION


𝑑
(sinx) = cos x
𝑑X

Example 24
𝑑𝑦 𝑑
(a) y = x2 – sinx : = 2x - 𝑑x (sin x) Difference Rule
𝑑x
𝑑𝑦 𝑑
(b) y = x2 – sinx : = x2(sin x) + 2x sin x Product Rule
𝑑x 𝑑x
2
= x cos x + 2x sin x
𝑑
sin 𝑥 𝑑𝑦 𝑥. (sin 𝑥)−sin 𝑥.1
𝑑𝑥
(c) y = : = Quotient Rule
𝑥 𝑑x 𝑥2

𝑥 cos 𝑥−sin 𝑥
= 𝑥2

RULE NO. 8 DERIVATIVE OF COSINE FUNCTION


𝑑
(cos x) = - sinx
𝑑x
EXAMPLE 25 (a) y = 5x + cos x
𝑑𝑦 𝑑 𝑑
= 𝑑x (5x) + 𝑑x (cos x) Sum Rule
𝑑x
= 5 – sin x
(b) y = sinx cosx
𝑑𝑦 𝑑 𝑑
= sin x 𝑑x (cos x) + cos x 𝑑x (sin x) Product Rule
𝑑x
= sinx (- sinx) + cos x (cos x)
= cos2 x – sin2x

RULE NO. 9 : DERIVATES OF OTHER TRIGONOMETRIC FUNCTIONS

Because sin x and cos x are differentiable functions of x, the related functions

sin 𝑥 1
tan x = ; sec x =
cos 𝑥 cos 𝑥

cos 𝑥 1
cot x = sin 𝑥 cosec x = sin 𝑥
are differentiable at every value of x at which they are defined. There derivatives.
Calculated from the Quotient Rule, are given by the following formulas.

𝑑 𝑑
( tan x) = sec2 x ; ( sec x) = sec x tan x
𝑑𝑥 𝑑𝑥

𝑑 𝑑
( cot x) = - cosec2 x ; ( cosec x) = -cosec x cot x
𝑑𝑥 𝑑𝑥

Example 26 Find dy / dx if y = tan x.


𝑑 𝑑
𝑑 𝑑 sin 𝑥 cos 𝑥 (sin 𝑥)−sin 𝑥 (cos 𝑥)
𝑑𝑥 𝑑𝑥
Solution: (tan x) = 𝑑𝑥 (cos 𝑥) =
𝑑𝑥 𝑐𝑜𝑠2

cos 𝑥 cos 𝑥−sin 𝑥 (− sin 𝑥)


= 𝑐𝑜𝑠2 𝑥
𝑐𝑜𝑠2 𝑥+𝑠𝑖𝑛2 𝑥 1
= = 𝑐𝑜𝑠2 𝑥 = sec2x
𝑐𝑜𝑠2 𝑥
𝑑 𝑑
Example 27 (a) (3x + cot x) = 3 + 𝑑𝑥 (cot x) = 3 – cosec2x
𝑑𝑥
𝑑 2 𝑑 𝑑
(b) (sin 𝑥) = (d 𝑥) (2cosec x) = 2d 𝑥 (cosec x)
𝑑𝑥
= 2 (-cosec x cot x) = -2 cosec x cot x

RULE NO. 10 DERIVATIVE OF LOGARITHM AND EXPONENTIAL FUNCTION


𝑑 1 𝑑
d𝑥
(logex) = 𝑥  d 𝑥 (ex) = ex
Example 28 y = ex. loge(x)
𝑑𝑦 𝑑 𝑑 𝑑 𝑒𝑥
d𝑥
= d 𝑥 (ex). log (x) + . d 𝑥 [loge (x)] ex  d 𝑥 = ex. loge (x) + 𝑥

RULE NO. 11 CHAIN RULE OR ‘OUTSIDE INSIDE’RULE

𝑑𝑦 𝑑𝑦 𝑑𝑢
= 𝑑𝑢 , 𝑑𝑥
𝑑𝑥

It sometimes helps to think about the Chain Rule the following way. If y = f(g(x)),
𝑑𝑦
= f ’[g(x)].g’(x)
𝑑𝑥

In words: To find dy/dx, differentiate the “outside” function f and leave the “inside” g(x) alone;
then multiply by the derivative of the inside.

We now know now to differentiate sin x and x2 -4, but how do we differentiate a composite like
sin(x2-4)? The answer is, with the Chain Rule, which says that the derivative of the composite
of two differentiable functions is the product of their derivatives evaluated at appropriate points.
The Chain Rule is probably the most widely used differentiation rule in mathematics.
This section describes the rule and how to use it. We begin with examples.

SOLVED EXAMPLES

Example 29 The function y = 6x – 10 = 2(3x-5) is the composite of the functions y = 2 and u = 3x-5.
How are the derivatives of these three functions related?
𝑑𝑦 𝑑𝑦 𝑑𝑢
Solution: We have 𝑑𝑥
= 6, 𝑑𝑢 = 2, 𝑑𝑥 = 3
𝑑𝑦 𝑑𝑦 𝑑𝑢
Since 6 = 2. 3, = . 𝑑𝑥
𝑑𝑥 𝑑𝑢
𝑑𝑦 𝑑𝑦 𝑑𝑢
Is it an accident that = . ?
𝑑𝑥 𝑑𝑢 𝑑𝑥
If we think of the derivative as a rate of change, our intuition allows us to see that this
relationship is reasonable. For y = f(u) and u = g(x), if y changes twice as fast as u and u
changes three times as fast as x, then we expect y to change six times as fast as x.
Example 30 We sometimes have to use the Chain Rule two or more times to find a derivative.
Here is an example. Find the derivative of g(t) = tan (5- sin 2t)
𝑑
Solution: g’(t) = 𝑑𝑡(tan(5-sin 2t)
Derivative of
tan u with
𝑑
2
= sec (5-sin 2t). 𝑑𝑡 (5- sin 2t) u = 5 – sin 2t

Derivative of
𝑑
= sec (5-sin 2t). (0 – (cos 2t). 𝑑𝑡(2t)
2
tan u with
u = 5 – sin 2t

= sec2 (5-sin 2t). (- cos 2t) . 2


= -2(cos 2t) sec2 (5- sin 2t)
𝑑 1
Example 31 (a) (1-x2)1/4 = 4 = (1-x2)-3/4 (-2x) u = 1 – x2 and n = ¼
𝑑𝑥

Function defined
on [-1, 1]
−𝑥
= 2(1−𝑥 2 )3/4

derivative defined only on (-1, 1)


𝑑
(b) sin 2x
𝑑𝑥
𝑑
= cos 2 x 𝑑𝑥 (2x)
= cos 2x.2
= 2 cos 2x
𝑑
(c) ( A sin (𝜔t + ∅)
𝑑𝑡

𝑑
= A cos (𝜔t + ∅) 𝑑𝑡 (𝜔t + ∅ )

= A cos (𝜔t + ∅). 𝜔.

= A 𝜔 cos (𝜔t + ∅)

RULE NO. 12 : POWER CHAIN RULE

If u(x) is a differentiable function and where n is a Real number, then u” is


𝑑 𝑑𝑢
differentiable and 𝑑𝑥 un = nun-1 𝑑𝑥 , ∀ n ∈ R
𝑑 𝑑
Example 32 (a) sin5x = 5 sin4 x 𝑑𝑥 (sin x)
𝑑𝑥

= 5 sin4 x cos x
𝑑 𝑑
(b) (2x + 1)-3 = -3 (2x + 1)-4 𝑑𝑥 (2x + 1)
𝑑𝑥

= -3(2x + 1)-4- (2) = -6 (2x + 1)-4


𝑑 1 𝑑 𝑑
(c) (3𝑥−2) = (3x -2)-1 = -1 (3x – 2)-2 𝑑𝑥(3x-2)
𝑑𝑥 𝑑𝑥

3
= -1(3x -2)-2 (3) = -
(3𝑥−2)2
In part (c) we could also have found the derivative with the Quotient Rule.
𝑑
Example 33 (a) 𝑑𝑥 (Ax + B)n

𝑑𝑢
Solution: (a) Here u = Ax + B , 𝑑𝑥 = A

𝑑
∴ 𝑑𝑥(Ax + B)n = n (Ax + B)n-1. A

𝑑
(b) 𝑑𝑥 sin (Ax + B) = cos (Ax + B). A

𝑑 1
(c) 𝑑𝑥 log (Ax + B) = 𝐴𝑥+𝐵 . A

𝑑
(d) 𝑑𝑥 tan (Ax + B) = sec2 (Ax + B). A

𝑑
(e) 𝑑𝑥 e(Ax + B) = e(Ax +B) . A

RULE NO. 13 : RADIAN VS. DEGREES


𝑑 𝑑 𝜋𝑥 𝜋 𝜋𝑥 𝜋
sin (x°) = 𝑑𝑥 sin (180) = 180 cos (180) = 180 cos (x°)
𝑑𝑥

DOUBLE DIFFERENTIATION
If f is dittorenttablo function. then its derivative f ‘is also a function, so f ’may have a
derivative of its own, denoted by (f ’ )’= f ” . This new function f ” is called the second
derivative of because it is the derivative of the derivative of f. Using Leibniz notation, we
write the second derivative of y = f(x) as
𝑑 𝑑𝑦 𝑑2 𝑦
( ) = 𝑑𝑥 2
𝑑𝑥 𝑑𝑥
Another notation is f ” (x) = D2 f(x) = D2f(x)
INTERPRETATION OF DOUBLE DERIVATIVE
We can interpret f ” (x) as the slope of the curve y = f ’(x) at the point (x, f ’(x)). In
other words, it is the rate of change of the slope of the original curve y = f(x)
In general, we can interpret a second derivative as a rate of change of a rate of change.
The most familiar example of this is acceleration, which we define as follows.
It s = s(t) is the position function of an object that moves in a straight line, we known
that its first derivative represents the velocity v(t) of the object as a function of time:
𝑑𝑠
v(t) = s’(t) = 𝑑𝑡
The instantaneous rate of change of velocity with respect to time is called the acceleration
a(t) of the object. Thus, the acceleration function is the derivative of the velocity function and is
therefore the second derivative of the position function:
a(t) = v’(t) = s” (t)
𝑑𝑣 𝑑2 𝑠
or in Leibniz notation, a = 𝑑𝑡 = 𝑑𝑡 2

Solved Examples
Example 34 : If f(x) = x cos x, find f ” (x)
Solution: Using the Product Rule, we have
𝑑 𝑑
f '(x) = x 𝑑𝑥 (cos x) + cos x 𝑑𝑥 (x)
= -x sin x + cos x
To find f ” (x) we differentiate f ’ (x)
𝑑
f " (x) = (-x sin x + cos x)
𝑑𝑥
𝑑 𝑑 𝑑
= -x 𝑑𝑥 (sin x) + sin x 𝑑𝑥 (- x) + 𝑑𝑥 (cos x)
= - x cos x – sin x – sin x = - x cos x – 2 sin x
Example 35: The position of a particle is given by the equation
s = f (t) = t3 – 6t2 + 9t
Where t is measured in seconds and s in meters.
Find the acceleration at time t. What is the acceleration after 4 s?
Solution: The velocity function is the derivative of the position function:
s = f(t) = t3 – 6t2 + 9
where t is measured in seconds and s in meters.
Find the acceleration at time t. What is the acceleration after 4 s?
Solution: The velocity function is the derivative of the position function:
s = f(t) = t3-6t2 + 9t
𝑑𝑠
 v(t) = 𝑑𝑡 = 3t2 – 12t + 9
The acceleration is the derivative of the velocity function:

𝑑2 𝑠 𝑑𝑣
a(t) = 𝑑𝑡 2 = 𝑑𝑡 = 6t – 12  a(4) = 6(4) – 12 = m/s2

APPLICATION OF DERIVATIVES
DIFFERENTIATION AS A RATE OF CHANGE
𝑑𝑦
is rate of change of ‘y’ with respect to ‘x’
𝑑𝑥
For examples:
𝑑𝑥
(i) v = 𝑑𝑡 this means velocity ‘v’ is rate of change of displacement ‘x’ with respect to time ‘t’
𝑑𝑦
(ii) a = 𝑑𝑥 this means acceleration ‘a’ is rate of change of velocity ‘v’ with respect to time ‘t’
𝑑𝑝
(iii) F= this means force ‘F’ is rate of change of momentum ‘p’ with respect to time ‘t’
𝑑𝑡
𝑑𝐿
(iv) 𝜏 = 𝑑𝑡 this means torque ‘𝜏′is rate of change of angular momentum ‘L’ with respect to time‘ 𝜏′
𝑑𝑞
(v) I= this means current ‘I’ is rate of flow of charge ‘q’ with respect to time ‘t’
𝑑𝑡
Solved Examples
𝜋
Example 36. The area A of a circle is related to its diameter by the equation A = 4 D2
How fast is the area changing with respect to the diameter when the diameter is 10 m ?
Solution: The (Instantaneous) rate of change of the area with respect to the diameter is
𝑑𝐴 𝜋 𝜋𝐷
= 4 2D = 4
𝑑𝐷
When D = 10 m, the area is changing at rate (𝜋/2) 10 = 5 𝜋 m2/m. This means that a small change
∆D m in the diameter would result in a change of about 5 𝜋∆D m2 in the area of the circle.
t (seconds) s(meters)

Example 37. Expermental and theoretical investigations revealed that the
distance a body released from rest falls in time t is proportional t  0
to the square of the amount of time it has fallen.
We express this by saying that 10
15
1 2 t2 20
s = 2 gt
25
Where s is distance and g is the acceleration due to Earth’s 30
gravity. This equation holds in a vacuum, where there is no 35
air resistance, but it closely models the fall of dense, heavy t3 40
objects in air. Figure shows the free fall of a heavy ball bearing 45
released from rest at time t = 0 sec. A ball bearing falling from rest
(a) How many meters does the ball fall in the first 2 sec?
(b) What is its velocity, speed, and acceleration then?
Solution: (a) The free-fall equation is s = 4.9 t2.
During the first 2 sec. the ball falls
s(2) = 4.9(2)2 = 19.6 m,
(b) At any time t, velocity is derivative of displacement:
𝑑
v(t) = s’(t) = 𝑑𝑡 (4.9t2) = 9.8 t.
At t = 2, the velocity is v(2) = 19.6 m/sec
In the downward (increasing s) direction. The speed at t = 2 is
𝑑2 𝑠
speed = |v(2)| = 19.6 m/sec. a = 𝑑𝑡 2 = 9.8 m/s2

MAXIMA AND MINMIA


y
Suppose a quantity y depends on another quantity
x in a manner shown in the figure. It becomes maximum
at x1 and minimum at x2, At these points the tangent to the
curve is parallel to the x-axis and hence its slope tan 𝜃=0.
Thus, at a maximum or a minimum,
𝑑𝑦
slope = 𝑑𝑥 = 0

MAXIMA
Just before the maximum the slop is positive, at the maximum
𝑑𝑦
it is zero and just after the maximum it is negative. Thus𝑑𝑥
𝑑𝑦
decreases at a maximum and hence the rate of change of 𝑑𝑥 is
𝑑 𝑑𝑦
negative at a maximum i.e. 𝑑𝑥 (𝑑𝑥 ) < 0 at maximum.

𝑑 𝑑𝑦
The quantity 𝑑𝑥 (𝑑𝑥 ) is the rate of change of the slope.
𝑑2 𝑦
It is written as 𝑑𝑥 2
𝑑𝑦 𝑑2 𝑦
Conditions for maxima are :- (a) 𝑑𝑥 = 0 (b) 𝑑𝑥 2 < 0
For min inma, as
MINIMA
Similarly, at a minimum the slope changes from negative
to positive. Hence with the increase of x. the slope is
increasing that means the rate of change of slope with
respect to x is positive
𝑑 𝑑𝑦
hence 𝑑𝑥 (𝑑𝑥 ) > 0.
𝑑𝑦 𝑑2 𝑦
Conditions for minima are : - (a) 𝑑𝑥 = 0 (b) 𝑑𝑥 2 > 0

Quite often it is known from the physical situation whether the quantity is a maximum or a
𝑑2 𝑦
minimum. The test on 𝑑𝑥 2 may then be omitted.

Solved Examples
Example 38 Particle’s position as a function of time is given as x = 5t2 – 9t + 3. Find out the maximum value
of position co-ordinate ? Also, plot the graph.
Solution: x = 5t2 – 9t + 3
𝑑𝑥
= 10 t – 9 = 0
𝑑𝑡
t = 9/10 = 0.9
𝑑2 𝑦
Check, whether maxima or minima exists. = 10 > 0
𝑑𝑥 2
there exists a minima at t = 0.9
Now, Check for the limiting values.
When t = 0 ; x=3
t=∞; x=∞
So, the maximum position co-ordinate does not exist.
Graph

Putting t = 0.9 in the equation


x = 5 (0,9)2- 9(0.9) + 3 = 1.05
NOTE: If the coefficient of t2 is positive, the curve will open upside.

SOLVED EXAMPLES ON APPLICATION OF DERIVATIVE

Example 39 Does the curve y = x4 – 2x2 + 2 have any horizontal tangents ? If so, where
Solution: The horizontal tangents, if any, occur where the slope dy/dx is zero. To find these point. We
𝑑𝑦 𝑑
1. Calculate dy/dx: 𝑑𝑥 = 𝑑𝑥(x4 – 2x2 + 2) = 4x3 – 4x
𝑑𝑦
2. Solve the equation: 𝑑𝑥 = 0 for x : 4x3 – 4x = 0
4x(x2 -1) = 0
x = 0,1, -1
The curve y = x – 2x + 2 has horizontal
4 2

tangents at x = 0, 1, and -1.


The corresponding points on the
curve are (0,2) (1, 1) and (-1, 1).
See figure.

Example 40 A hot air balloon rising straight up from a level field is tracked by a range finder 500 ft from the
lift – off point. At the moment the range finder’s elevation angle is 𝜋/4, the angle is increasing at
the rate of 0.14 rad/min. How fast is the balloon rising at the moment?
Solution: We answer the question in six steps.

Step 1 : Draw a picture and name the variables and constants (Figure) . The variables in
the picture are 𝜃 = the angle the range finder makes with the ground(radians)
y = the height of the balloon (feet).
We let t represent time and assume 𝜃 and y to be differentiable functions of t.
The one constant in the picture is the distance from the range finder to the lift-off point(500 ft).
There is no need to give it a special symbol s.
Step 2 : Write down the additional numerical information
𝑑𝜃
= 0.14 rad/min when 𝜃 = 𝜋/4.
𝑑𝑡
Step 3: Write down what we are asked to find, We want dy/dt when 𝜃 = 𝜋/4.
𝑦
Step 4: Write an equation that relates the variables y and 𝜃. = tan 𝜃 , or y = 500 tan 𝜃
500

Step 5: Differentiate with respect to t using the Chain Rule. The result tells how dy/dt
(which we want) is releated to d 𝜃/dt(which we know).
𝑑𝑦 𝑑𝜃
= 500 sec2 𝜃
𝑑𝑥 𝑑𝑡

Step 6 : Evaluate with 𝜃 = 𝜋/4 and d 𝜃/dt = 0.14 to find dy/dt


𝑑𝑦 2 𝜋
= 500 (√2) (0.14) = (1000) ( 0.14) = 140 (sec 4 = √2)
𝑑𝑡

At the moment in question, the balloon is rising at the rate of 140 ft/min.

Example 41. A police cruiser, approaching a right-angled intersection from the north, is chasing a
speeding car that has turned the corner and is now moving straight east. When the Cruiser
is 0.6 mi north of the intersection and the car is 0.8 mi to the east, the police determine with
radar that the distance between them and the car is increasing at 20 mph. If the cruiser is moving
at 60 mph at the instant of measurement, what is the speed of the car?

Solution: We carry out the steps of the basic strategy.

Step 1 : Picture and variables. We picture the car and cruiser in the coordinate plane, using the positive
x- axis as the eastbound highway and the positive y-axis as the northbound highway (Figure).
We let t represent time and set x = position of car at time t.
y = position of cruiser at time t, s = distance between car and cruiser
at time t.
We assume x, y and s to be differentiable functions of t.
𝑑𝑦 𝑑𝑠
x = 0.8 ,mi, y = 0.6 mi, = - 60 mph , = 20 mph
𝑑𝑡 𝑑𝑡
(dy/dt is negative because y is decreasing.)
𝑑𝑥
Step 2 : To find : 𝑑𝑡
Step 3 : How the variables are related : s2 = x2 + y2 Pythagorean theorem
(The equation s = √𝑥 2 + 𝑦 2 would also work.)
𝑑𝑠 𝑑𝑥 𝑑𝑦
Step 4 : Differentiate with respect to t. 2s 𝑑𝑡 = 2x 𝑑𝑡 + 2y 𝑑𝑡 Chain Rule
𝑑𝑠 1 𝑑𝑥 𝑑𝑦 1 𝑑𝑥 𝑑𝑦
= 𝑠 (𝑥 𝑑𝑡 + 𝑦 𝑑𝑡 ) = (𝑥 𝑑𝑡 + 𝑦 𝑑𝑡 )
𝑑𝑡 √𝑥 2 +𝑦2
Step 5 : Evaluate, with x = 0.8 , y = 0.6 , dy/dt = -60, ds/dt = 20 , and solve of dx/dt.
1 𝑑𝑥
20 = (0.8 𝑑𝑡 + (0.6)(−60))
√(0.8)2 + (0.6)2

𝑑𝑥 𝑑𝑥 20+36
 20 = 0.8 𝑑𝑡 - 36  𝑑𝑡 = 0.8
= 70
INTEGRATION

In mathematics, for each mathematical operation, there has been defined an inverse operation.
For example- Inverse operation of addition is subtraction, inverse operation of multiplication
is division and inverse operation of square root. Similarly there is a inverse operation for differentiation
which is known as integration

ANTIDERIVATIVES OR INDEFINITE INTEGRALS

Definitions :

A function F (x) is an antiderivative of a function f(x) if F’(x) = f(x) for all x in the domain of f.
The set of all antiderivatives of f is the indefinite integral of f with respect to x, denoted by
The function is the integrand.

x is the variable of integration


Integral sign

f ( x)dx
Integral of f

The symbol  is an integral sign. The function f is the integrand of the integral and x is the variable of
integration
For example f(x) = x3 then f’(x) = 3x2
2 3
So the integral of 3x is x
Similarly if f(x) = x3 + 4 then f’(x) = 3x2
2 3
So the integral of 3x is x + 4
there for general integral of 3x2 is x3 + c where c is a constant
One antiderivative F of a function f, the other antiderivatives of f differ from F by a constant. We indicate
this in integral notation in the following way:
 f ( x)dx  F ( x)  C. …………..(i)
The constant C is the constant of integration or arbitrary constant, Equation (1) is read, “The indefinite
integral of f with respect to x is F(x) + C.” When we find F(x) + C, we say that we have integrated f
and evaluated the integral.

Solved Examples
Example 42. Evaluate
an antiderivative of 2x

 2xdx  x C
2
Solution :
the arbitrary constant

The formula x2 + C generates all the antiderivatives of the function 2x.


The function x2 + 1, x2 – 𝜋, and x2+√2 are all antiderivatives of the function 2x, as you can
check by differentiation.
Many of the indefinite integrals needed in scientific work are found by reversing derivative formulas.
INTEGRAL FORMULAS

Indefinite Integral Reversed derivative formula


x n 1 d x n 1

   C, n  1, n rational  xn 
n
1. x dx 
n 1 dx  n  1 

d
 dx  1dx  x  C (special case) dx
(x)  1

 cos(Ax B) d   cos kx) 


2.  sin(Ax  B)dx  A
C 
dx  k
 = sin kx

sin kx d  sin kx) 
3.  cos kxdx  k
C 
dx  k 
 = cos kx

d
 sec xdx  tan x  C tan x  sec 2 x
2
4.
dx
d
 cos ec xdx   cot x + C ( cot x)  cos ec 2 x
2
5.
dx
d
6.  sec x tan x dx = sec x + C dx
sec x  sec x tan x

d
7.  cos ecx cot x dx = - cosec x + C dx
( cos ecx)  cos ec x cot x

1 1 d
8.  (ax b)  a n (ax + b) C
dx
(  cos ecx)  cosec x cot x

Solved Examples
Example 43. Examples based on above formulas:

x6
 x dx C
5
(a) Formula 1 with n = 5
6

1
(b)  x
dx =  x 1/2 dx = 2x1/2 + C = 2√𝑥 + C Formula 1 with n = -1/2

 cos 2x
(c)  sin 2xdx  +C Formula 2 with k = 2
2

x x sin(1 / 2)x x
(d)  cos dx =  cos xdx =  C = sin  C Formula 3 with k = ½
2 2 1/ 2 2

Example 44. Right:  x cosx dx = x sin x + cos x + C

Reason :The derivative of the right-hand side is the integrand:


𝑑𝑥
Check: ( x sin x + cos x + C) = x cos x + sin x – sin x + 0 = x cos x.
𝑑𝑡
Wrong:  x cosx dx = x sin x + C
Reason: The derivative of the right-hand side is not the integrand:
𝑑𝑥
Check: ( x sin x + C) = x cos x + sin x + 0 ≠ x cos x.
𝑑𝑡
RULES FOR INTEGRATION
RULE NO. 1 : CONSTANT MULTIPLE RULE

A function is an antiderivative of a constant multiple kf of a function f if and only


if it is k times an antiderivative of f.

 kf(x)dx = k  f(x)dx ; where k is a constant


Example 45. Rewriting the constant of integration
 5 sec x tan x d x = 5  sec x tan x d x Rule 1
= 5 (sec x + C) Formula 6
= 5 sec x + 5C First form
= sec x + C’ Shorter form, where C’ is 5C
= 5 sec x + C Usual form – no prime. Since 5 times an
arbitrary constant an arbitrary
constant, we rename C’.
What about all the different forms in example? Each one gives all the antiderivatives of
f(x) = 5 sec x tan x.
so each answer is correct. But the least complicated of the three, and the usual choice, is
 5 sec x tan x d x = 5 sec x + C
Just as the Sum and Difference Rule for differentiation enables us to differentiate expressions
term by term,
the Sum and Difference Rule for integration enables us to integrate expressions term by term.
When we do
so, we combine the individual constants of integration into a single arbitrary into a single
arbitrary
constant at the end.

RULE NO. 2: SUM AND DIFFERENCE RULE

A function is an antiderivative of a sum or difference f ± g if and only if it is the sum or


difference of an
antiderivative of f an antiderivative of g.
 [f(x) ± g(x)]dx =  f(x) dx ±  g(x) dx
SOLVED EXAMPLES
Example 46. Term-by-term integration
Evaluate  (x2-2x+5) dx
Solution. If we recognize that (x3/3)-x2 + 5x is an antiderivative of x2 – 2x + 5, we can evaluate
the integral as
antiderivative arbitrary constant
2
x
(x 2  2x 5)dx   x 2  5x C
3

If we do not recognize the antiderivative right away, we can generate it term by term
with the sum and difference Rule.

𝑥3
 (x2 – 2x + 5) dx =  x2 dx -  2xdx +  5dx = 3
+ C1 – x2 + C2 + 5x + C2

This formula is more complicated than it needs to be. If we combine C1, C2 and C3 into a
single constant C = C1 + C2 + C3, the formula simplifies to

𝑥3
- x2 + 5x + C
3
and still gives all the antiderivatives there are. For this reason we recommend that you
go right to the final form even if you elect to integrate term by term. Write
𝑥3
 (x2 – 2x + 5) dx =  x2dx -  2xdx +  5dx = 3
- x2 + 5x + C

Find the simplest antiderivative you can for each part add the constant at the end.

Example 47.

We can sometimes use trigonometric identities to transform integrals we do not know how to
evaluate into integrals we do know how to evaluate. The integral formulas for sin2x and cos2 x
arise frequently in applications.
1−cos 2𝑥 1−cos 2𝑥
(a)  sin2 xdx =  2
dx sin2x = 2
1 1 1
=2  (1-cos2x) dx = 2  dx - 2  cos2xdx
𝑥 1 sin 2𝑥 𝑥 sin 2𝑥
= 2 - (2) +C=2- +C
2 2

1+𝑐𝑜𝑠2𝑥 1+𝑐𝑜𝑠2𝑥
(b)  cox2 xdx =  2
dx cos2x = 2

𝑥 sin 2𝑥
=2+ + C As in part (a), but with a sign change
2

Example 48. If the body is dropped from rest, what will its velocity be t seconds after it is released? The
acceleration of gravity near the surface of the earth is 9.8 m/sec2. This means that the
𝑑𝑣
velocity v of a body falling freely in a vacuum changes at the rate of = 9.8 m/sec2.
𝑑𝑡

Solution. In mathematical terms, we want to solve the initial value problem that consists of
𝑑𝑣
The differential condition: = 9.8
𝑑𝑡

The initial condition: v = 0 when t = 0 (abbreviated as v(0) = 0)

We first solve the differential equation by integrating both sides with respect to t:
𝑑𝑣
= 9.8 The differential equation
𝑑𝑡

𝑑𝑣
 𝑑𝑡
dt =  9.8dt Integrate with respect to t.

v + C1 = 9.8t + C2 Integrals evaluated

v = 9.8t + C. Constants combined as one

This last equation tells us that the body’s velocity t seconds into the fall is 9.8t + C m/sec.
For value of C: What value? We find out from the initial condition:
v = 9.8t + C
0 = 9.8(0) + C v(0) = 0
C = 0.
Conclusion : The body’s velocity t seconds into the fall is
v = 9.8t + 0 = 9.8t m/sec.
The indefinite integral F(x) + C of the function f(x) gives the general solution
y = F(x) + C of the differential equation dy/dx = f(x). The general solution gives
all the solutions of the equation (there are infinitely many, one for each value of C).
We solve the differential equation by finding its general solution. We then solve
the initial value problem by finding the particular solution that satisfies the
initial conditiony(x0) = y0 (y has the value y0 when x = x0.).

RULE NO. 3. RULE OF SUBSTITUTION

 f(g(x)) = g’(x) dx =  f(u)du 1. Substitute u = g(x), du = g’(x)dx.


= F(u) + C 2. Evaluate by finding an antiderivative F(u) of
f(u). (any one will do.)
= F(g(x)) + C 3. Replace u by g(x)

Solved Examples

5
Example 49. Evaluate (x + 2) dx
We can put the integral in the form
 u du
n

by substituting
𝑑
u = x + 2, du = d(x + 2) = 𝑑𝑥 (x + 2). Dx
= 1.dx = dx.
Then  (x + 2)5 dx =  u5du u=x+2 du = dx
u6
= +C Integrate, using rule no. 3 with n = 5.
6
(𝑥+2)6
= +C Replace u by x + 2
6
Example 50. Evaluate  √1 + 𝑦 2 . 2y dy =  u1/2du Let u = 1 + y2, du = 2y dy.

𝑢(1/2)+1
= (1/2) + 1 Integrate, using rule no. 3 with n = ½
2
= u3/2 + C Simpler form
3
2
= 3 (1 + y2)3/2 + C Replace u by 1 + y2
1
Example 51. Evaluate  cos ( 7𝜃 + 5) d 𝜃 =  cos u. 7du Let u = 7 𝜃 + 5, du = 7 d 𝜃, (1/7) du = d 𝜃

1
=7  cosudu With (1/7) out front, the integral
is now in standard form
1
= 7 sin u + C Integrate with respect to u.
1
= 7 sin (7 𝜃+5) + C Replace u by 7 𝜃 + 5.
Example 52. Evaluate  x2 sin(x)3 dx =  sin(x)3. x2 dx
1
=  sinu. 3du Let u = x3, du = 3x2 dx, (1/3) dy = x2dx.
1
= 3  sinudu
1
= 3 (- cos u) + C Integrate with respect to u.
1
= - 3 cos (x3) + C Replace u by x3
1 1
Example 53.  𝑐𝑜𝑠2 2𝜃
d𝜃 =  sec 2 2 𝜃 sec 2 𝜃 = 𝑐𝑜𝑠2𝜃
1
=  sec 2u. 2 du Let u = 2 𝜃, du = 2d 𝜃, d 𝜃 = (1/2)du.
1
=2  sec 2 u du
1
= 2 tan u + C Integrate, using eq. (4)
1
= 2 tan 2 𝜃 + C Replace u by 2 𝜃
𝑑 1 1 𝑑 1 𝑑
Check: 𝑑𝜃 (2 𝑡𝑎𝑛2𝜃 + 𝐶) = 2. 𝑑𝜃 (tan 2𝜃) + 0 = 2. (𝑠𝑒𝑐 2 2𝜃. 𝑑𝜃 2𝜃)Chain Rule
1 1
= 2. sec22𝜃 .2 = 𝑐𝑜𝑠2 2𝜃
Example 54.  sin4tcostdt =  u4du Let u = sin t, du = cos t dt.
𝑢5
= 𝑑𝜃 + C Integrate with respect to u.
𝑠𝑖𝑛5 𝑡
= 5 +C Replace u.
The success of the substitution method depends on finding a substitution that will
change an integral we cannot evaluate directly into one that we can. If the first substitution fails,
we can try to simplify the integrand further with an additional substitution or two.

DEFINITE INTEGRATION OR INTEGRATION WITH LIMITS


Upper limit of integration The function is the integrand

Integral sign

 f(x) dx
b

Lower limit of integration


Integral of f from a to b
b

 f (x)dx  [g(x)]  g(b)  g(a)


b
a
a

Where g(x) is the antiderivative of f(x) i.e.g’(x) = f(x)

Solved Examples
4 4
 3dx  3 dx  3[x]  3[4  (1)]  (3)(5)  15
4
Example 55. 1
1 1
 /2

𝜋
sin xdx  [ cos x]0/2  - cos(2 ) + cos(0) = -0 + 1 = 1
0

APPLICATION OF DEFINITE INTEGRAL : CALCULATION OF AREA OF A CURVE

From graph shown in figure if we divide whole area in


infinitely small strips of dx width.
We take a strip at x position of dx width.
Small area of this strip dA = f(x) dx.
So, the total area between the curve and
b
x- axis = sum of area of all strips =  f (x)dx
a

Let f(x) ≥ 0 be continuous on [a,b]. The area of the region between the graph of f and the x-axis is
b
A  xdx
a

Solved Examples

Example 56. Using an area to evaluate a definite integral


b

 xdx
a
0 < a < b.

Solution. We sketch the region under the curve y = x, a ≤ x ≥ b


(figure) and see that it is a trapezoid with height(b-a)
and bases a and b. The value of the integral is the
area of this trapezoid:
b
𝑎+𝑏 𝑏2 𝑎2
 xdx = (b – a)
a
2
= 2
– 2
.

Notice that x2/2 is an antiderivative of x, further evidence of a connection between


antiderivatives and summation.

VECTOR
In physics we deal with two types of physical quantity one is scalar and other is vector.
Each scalar quantities has magnitude.
Magnitude of a physical quantity means product of numerical value and unit of that physical
quantity.
For example mass = 4 kg
Magnitude of mass = 4 kg
and unit of mass kg
Example of scalar quantities : mass, speed, distance etc

Scalar quantities can be added, subtracted and multiplied by simple laws of algebra.

DEFINITION OF VECTOR

If a physical quantity in addition to magnitude.


(a) has a specified direction.
(b) It should obey commutative law of additions A ⃗ +B⃗ += B ⃗ +A⃗
(c) obeys the law of parallelogram of addition, then and then only it is said to
be a vector. If any of the above conditions is not satisfied the physical quantity cannot be a
vector.
If a physical quantity is a vector it has a direction, but the converse may or may not be true,
i.e.if a physical quantity has a direction, it may or may not a be vector. e.g., time pressure,
surface tension or current etc. have directions but are not vectors because they do not obey
parallelogram, law of addition.

⃗ | of A.
The magnitude of a vector (𝐴) is the absolute value of a vector and is indicated by|A
Example of vector quantity: Displacement, velocity, acceleration, force etc.

Representation of vector:
Geometrically, the vector is represented by a line with an arrow indicating the direction
of vector as
Tail Head
Length
(magnitude)

Mathematically, vector is represented by ⃗A


Sometimes it is represented by bold letter A.

IMPORTANT POINTS:

If a vector is displaced parallel to itself it does not change (see Figure)


C A B C
Transition of a vector
A parallel to itself
B
If a vector is rotated through an angle other than multiple of 2𝜋(or 3600) it changes ( see Figure)
If the frame of reference is translated or rotated the vector does not change (though its
components may change). See Figure).

Two vectors are called equal if their magnitudes and directions are same, and they represent
values of same physical quantity.
Angle between two vectors means smaller of the two angles between the vectors when they are
placed tail to tail by displacing either of the vectors parallel to itself(i.e. 0 ≤ 𝜃 ≤ 𝜋).

5.2 UNIT VECTOR


Unit vector is a vector which has a unit magnitude and points in a particular direction. Any
⃗ ) can be written as the product of unit vector (A
vector(A ̂ )in that direction and magnitude of the
given vector.
⃗⃗
A
⃗A = A 𝐴̂ or 𝐴̂ = 𝐴
A unit vector has no dimensions and unit. Unit vectors
along the positivex-, y- and z-axes of a rectangular
coordinate system are denoted by 𝑖̂, 𝑗̂ and 𝑘̂ respectively such that
|𝑖̂|= |𝑗̂|= |𝑘̂|= 1

Exercise-1

PART – 1 : FUNCTION & DIFFERENTIATION

*Marked Question are having more than one correct option.

SECTION – (A) : FUNCTION

A-1 f(x) = cos x + sin x

Find f(𝜋/2)

A-2 If f(x) = 4x + 3

Find f(f(2))

A–3 f(x) = log x3 and g(x) = log x

Which of the following statement is /are true-

(A) f(x) = g(x) (B) 3f(x) = g(x) (C) f(x) = 3g(x) (D) f(x) = (g(x))

A-4 tan 150 is equilvalent to:


5−√3 5+√3
(A) (2-√3) (B) (5 +√3) (C) ( ) (D) ( )
2 2
2
A -5 sin 𝜃 is equivalent to :
1+𝑐𝑜𝑠𝜃 1+𝑐𝑜𝑠2𝜃 𝑐𝑜𝑠2𝜃−1
(A) ( 2 ) (B) ( 2 ) (C) ( 2 )
A–6 sin A. sin(A+B) is equal to
1
(A) cos2A. cosB + sinA sin2B (B) sin2A. cosB + cos2A. sinB
2
2 1 2
(C) sin A. cosB +2 sin A. sinB (D) sin A. sinB + cosA cos2B
2

A–7 -sin𝜃 is equivalent to:


𝜋 𝜋
(A) cos ( 2 + 𝜃) (B) cos ( 2 − 𝜃) (C) sin (𝜃 − 𝜋) (D) sin (𝜃 + 𝜋)

A–8 If x1 = 8 sin 𝜃 and x2 = 6 cos 𝜃 then


(A) (x1 + x2)max = 10 (B) x1 + x2 = sin(𝜃 + 370)
𝑥 4
(C) x1x2 = 24 sin2 𝜃 (D) 𝑥1 = 3 tan 𝜃
2
A–9 𝜃 is angle between side CA and CB of triangle, shown in the figure then 𝜃 is given by

√5 √5 2
(A) cos 𝜃 (B) sin 𝜃 = (C) tan 𝜃 = (D) tan 𝜃 = 3
3 3

SECTION- (B) : DIFFERENTATION OF ELEMENTRY FUNCTIONS

Find the derivative of given functions w.r.t. corresponding independent variable

B-1 y = x2 + x + 8
B-2 y = tan x + cot x

Find the first derivative & second derivate of given functions w.r.t corresponding independent variable.
B–3 y = sinx + cos x
B – 4. y = nx = ex

SECTION –(C) : DIFFERENTIATION BY PRODUCT RULE

Find derivative of given functions w.r.t the independent variable x.


C–1 y = ex nx
C–2 y = sinx cos x

SECTION- (D) : DIFFERENTIATION BY QUOTIENT RULE

Find derivative of given functions w.r.t the independent variable.


2𝑥+5
D–1 y = 3𝑥−2
nx
D–2 y= 𝑥
D–3 y = (secx + tanx) (sec x – tanx)
D – 4. Suppose u and v are functions of x that are differentiable at x = 0 and that
u(0) =- 5, u’(0) = -3, v(0) = -1 v’(0) = 2
Find the values of the following derivatives at x = 0.
d d 𝑢 d 𝑣 d
(a) 𝑑𝑥 (uv) (b) 𝑑𝑥 (𝑣 ) (c) 𝑑𝑥 (𝑢) (d) 𝑑𝑥 (7v – 2u)

SECTION – (E) : DIFFERENTIATION BY CHAIN RULE

dy
Find as a function of x
𝑑𝑥
E–1 y = sin 5 x
E–2 y = 2 sin (𝜔𝑥 + ∅) where 𝜔 and ∅ constants
E–3 y = (4 – 3x)9

SECTION – (F) : DIFFERENTIATION OF IMPLICIT FUNCTIONS

dy
Find 𝑑𝑥
F–1 (x + y)2 = 4
F–2 x y + xy2 = 6
2

SECTION – (G) : DIFFERENTIATION AS A RATE MEASUREMENT

G–1 Suppose that the radius r and area A = 𝜋r2 of a circle are differentiable functions of t.
Write an equation that relates dA / dt to dr / dt.
G–2 Suppose that the radius r and surface area S = 4𝜋𝑟 2 of sphere are differentiable
𝑑𝑠 𝑑𝑟
functions of t. Write an equation that relates to .
𝑑𝑡 𝑑𝑡

SECTION – (H): MAXIMA & MINIMA

H–1 Particle’s position as a function of time is given by x = -t2 + 4t + 4 find the maximum
value of position co-ordinate of particle.
H–2 Find the values of function 2x3-15 x2 + 36 x + 11at the points of maximum and minimum.

SECTION – (I):MISCELLANEOUS
𝑑𝑦
Given y = f(u) and u = g(x), find 𝑑𝑥
I – 1. y = 2u3, u = 8x – 1
I-2 y – sinu, u = 3x + 1
I–3 y = 6u – 9, u = (1/2)x4
𝑥
I–4 y = cosu, u = - 3

PART – II : INTEGRATION
SECTION – (A) : INTEGRATION FO ELEMENTRY FUNCTIONS
Find integrals of given functions
A – 1 x2 – 2x + 1
1
A-2 √𝑥 + 𝑥

3 1
A-3 √𝑥 + 3 𝑥

A-4 sec2 x
A-5 csc2 x
A-6 sec x tan x
1
A–7 3𝑥

SECTION – (B) : INTEGRATION BY SUBSTITUTION METHOD


Integrate by using the substitution suggested in bracket
B-1. ∫ 𝑥sin(2x2) dx, (use, u = 2x2)

B-2 ∫ 𝑠𝑒𝑐2ttan2t dt, (use, u = 2t)

Integrate by using a suitable substitution


3
B – 3 ∫ (2−𝑥)2 dx
B – 4. ∫ sin(8z – 5)dz
SECTION – (C) : DEFINITE INTEGRATION
1

C – 1.  2 d𝜃
4

5 2

C–2  rdr r dr
2
1
C–3  e dx ex dx
x

SECTION – (D) : CALCULATION OF AREA

Use a definite integral to find the area of the region between the given curve and the x – axis on the
interval [0,b]

D – 1 y = 2x
𝑥
D–2 y=2+1
Use a definite integral to find the area of the region between the given curve and the x – axis
one the interval[0, 𝜋]
D - 3 y = sin x
D - 4 y = sin2x
2
D–5 I  sin
0
sin(𝜃 + ∅).d𝜃 where ∅ is a constant. Then value of I :

(A) may be positive (B) may be negative


(C) may be zero (D) Always zero for any value of ∅
D – 6* If x = 3 sin𝜔t and x = 4 cos𝜔t then
1 2
𝑥 2𝜋
(A) 𝑥1 is independent of t (B) Average value of < 𝑥12 + 𝑥22 > from t = 0 to t = 𝜔 is zero
2

D – 7* I =  sin sin(𝜃 + ∅)d𝜃, where ∅ is non zero constant then the value of I:
0

(A) may be positive (B) may be negative


𝜋
(C) may be zero (D) always zero of ∅ = 4

PART – III : VECTOR

SECTION – (A) : DEFINITION OF VECTOR & ANGLE BETWEEN VECTORS

A – 1 Vectors ⃗A, ⃗B and ⃗C are shown in figure. Find angle between

(i) ⃗A and ⃗B (ii) ⃗A and ⃗C (iii) ⃗B and ⃗C


A – 2 The forces, each numerically equal to 5 N, are acting as shown in the Figure. Find the
angle between forces?

A – 3 Rain is falling vertically downwards with a speed 5 m/s. If unit vector along upward is
defined as 𝑗̂, represent velocity of rain in vector form.
A – 4 The vector joining the points A(1, 1 -1) and B (2, -3,4) & pointing from A to B is –
(A) −𝑖̂ + 4𝑗̂ - 5𝑘̂ (B) 𝑖̂ + 4𝑗̂ + 5𝑘̂ (c) 𝑖̂ - 4𝑗̂ + 5𝑘̂ (d) - 𝑖̂ - 4𝑗̂ - 5𝑘̂

SECTION – (B) : ADDITION OF VECTORS


B – 1 A man walks 40 m North, then 30m East and then 40 m South. Find the displacement
from the starting point?
B – 2 A vector of magnitude 30 and direction eastwards is added with another vector of
magnitude 40 and direction Northwards. Find the magnitude and direction of resultant with the east.
B – 3 Two vectors a⃗ and ⃗b inclined at an angle 𝜃 w.r.t each other have a resultant c which makes and
angle 𝛽 with a⃗ . If the directions of a⃗ and ⃗b are interchanged, then the resultant will have the same
(A) magnitude (B) direction
(C) magnitude as well as direction (D) neither magnitude nor direction
B – 4 Two vectors A and B lie in a plane. Another vector ⃗C lies outside this plane. The
⃗ ⃗
resultant ⃗A + ⃗B+ ⃗C of these three vectors
(A) can be zero (B) cannot be zero
(C) lies in the plane of . 𝐴. & 𝐵 ⃗ (D) lies in the plane . 𝐴 & 𝐴 + 𝐵⃗
B – 5 The vector sum of the forces of 10 N and 6 N can be
(A) 2 N (B) 8 N (C) 18 N (D) 20 N
B – 6 A set of vectors taken in a given order gives a closed polygon. Then the resultant of these vectors
is a
(A) Scalar quantity (B) pseudo vector (C) unit vector (D) null vector
B – 7 The vector sum of two force P and Q is minimum when the angle 𝜃 between their positive directions,
is
𝜋 𝜋 𝜋
(A) 4 (B) 3 (C) 2 (D) 𝜋
B – 8 The vector sum of two vectors 𝐴 and 𝐵 ⃗ is maximum, then the angle 𝜃 between two vectors is –
0 0
(A) 0 (B) 30 (C) 450 (D) 600
B – 9 Given 𝐶 = 𝐴 + 𝐵 ⃗ . Also, the magnitude of 𝐴 ,𝐵⃗ and 𝐶 are 12, 5 and 13 units respectively, The
angle between 𝐴 and 𝐵 ⃗ is
𝜋 𝜋
(A) 00 (B) 4 (C) 2 (D) 𝜋
B – 10 If 𝑃⃗ + 𝑄⃗ = 𝑃⃗ - 𝑄
⃗ and 𝜃 is the angle between 𝑃⃗ and 𝑄 ⃗ , then
0 0
(A) 𝜃 = 0 (B) 𝜃 = 90 (C) P = 0 (D) Q = 0
B – 11 The sum and difference of two perpendicular vectors of equal lengths are
(A) Of equal lengths and have an acute angle between them
(B) of equal length and have an obtuse angle between them
(C) also perpendicular to each other and are of different lengths
(D) also perpendicular to each other and are of equal lengths
SECTION – (C) : RESOLUTION OF VECTORS
y
C – 1. Find the magnitude of 3𝑖̂ + 2𝑗̂ + 𝑘̂?
C – 2. If 𝐴 = 3𝑖̂ + 4𝑗̂ then find 𝐴̂ x
C – 3. What are the x and they y components of a 25 m displacement at an
angle of 2100 with the x- axis (anti clockwise)?
C – 4. One of the rectangular components of a velocity of 60 km h-1 is 30 km h-1. Find other
rectangular component?
C – 5. If 0.5𝑖̂ + 0.8𝑗̂ + C𝑘̂ is a unit vector. Find the value of C
C – 6. The rectangular components of a vector are (2, 2). The corresponding rectangular components
of another vector are (1, √3). Find the angle between the two vectors.
C – 7. The x and y components of a force are 2 N and – 3 N. The force is
(A) 2𝑖̂ - 3𝑗̂ (B) 2𝑖̂ + 3𝑗̂ (C) -2𝑖̂ - 3𝑗̂ (D) 3𝑖̂ + 2𝑗̂

SECTION – (D) : PRODUCTS OF VECTORS


D – 1. If 𝐴 = 𝑖̂ + 𝑗̂ + 𝑘̂ and 𝐵
⃗ = 2𝑖̂ + 𝑗̂ find (a) 𝐴 . 𝐵
⃗ (b) 𝐴 × 𝐵⃗
⃗ | = 3 and 𝜃 = 60 in the figure, Find (a) 𝐴 . 𝐵
D – 2 If |𝐴| = 4, |𝐵 0 ⃗ ⃗|
(b) |𝐴 × 𝐵

⃗ & 𝐶 satisfy the relation 𝐴 . 𝐵


D – 3 Three non zero vectors 𝐴 , 𝐵 ⃗ = 0 𝐴 . 𝐶 = 0. Then 𝐴 can be parallel to :
(A)𝐵⃗ (B) 𝐶 (C) 𝐵⃗ .𝐶 (D) 𝐵⃗ ×𝐶
D -4 The magnitude of scalar product of two vectors is 8 and that of vector product is 8√3. The
angle between them is:
(A) 300 (B) 600 (C) 1200 (D) 1500

EXERCISE – 2

PART – I : FUNCTION & DIFFERENTIATION


𝑥−1
1. If f(x) = 𝑥+1 then find {f(x)}
2𝑥−3
2. y = f(x) = 3𝑥−2
Find f(y)
3. For a triangle shown in the figure, side CA is 10 m, <A and angle < C are equal then:

(A) side a = side c = 10m (B) side a ≠ side c


10√3 10
(C) side a = side c = m (D) side a = side c = m
3 √2
4. If y1 = A sin 𝜃1 and y2 = A sin 𝜃2 then
𝜃 +𝜃 𝜃 −𝜃
(A) y1 + y2 = 2A sin ( 1 2 2 ) cos ( 1 2 2 )(B) y1 + y2 = 2A sin𝜃1 sin𝜃2
𝜃 −𝜃 𝜃 +𝜃 𝜋 𝜋
(C) y1 - y2 = 2A sin ( 1 2 2 ) cos ( 1 2 2 )(D) y1 . y2 = -2A2 cos( 2 + 𝜃1 ). cos ( 2 − 𝜃2 )
5. Which of following are true
(A) sin 370 + cos 370 = sin 530 + cos 530(B) sin 370 - cos 370 = cos 530 - sin 530
(C) tan370 + 1 = tan 530 – 1 (D) tan370 × tan 530 = 1
6. If R2 = A2 + B2 + 2AB cos𝜃, if |A| = |B| then value of magnitude of R is equivalent to:
𝜃 𝜃 𝜃
(A) 2Acos 𝜃 (B) Acos 2 (C) 2Acos 2 (D) 2Bcos 2
Find the first derivative and second derivative of given functions w.r.t the independent variable x.
7. y = nx2 + sin x
7
8. y = √𝑥 + tan x
Find derivative of given functions w.r.t the corresponding independent variable.
1 1
9. y = (𝑥 + 𝑥) (x - 𝑥 + 1)
10. r = ( 1 + sec 𝜃) sin 𝜃
Find derivative of given functions w.r.t the respective independent variable.
cot 𝑥
11. y = 1+cot 𝑥
nx+ex
12. tan 𝑥
𝑑𝑦
Find 𝑑𝑥 as a function of x
13. y = sin3 x + sin 3x
14. sin2(x2 + 1)
𝑑𝑞
15. q = √2𝑟 − 𝑟 2 , find 𝑑𝑟
𝑑𝑦
Find 𝑑𝑥
16. x3 + y3 = 18 xy
17. The radius r and height h of a circular cylinder are related to the cylinder’s volume V by
the formula V = 𝜋r2h.
(a) If height is increasing at a rate of 5 m/s while radius is constant, Find rate of increase
of volume of cylinder
(b) If radius is increasing at a rate of 5 m/s while height is constant, Find rate of increase of volume of
cylinder
(c) If height is increasing at a rate of 5 m/s and radius is increasing at a rate of 5 m/s, Find rate of
increase of volume of cylinder.
18. Find two positive numbers x & y such that x + y = 60 and xy is maximum –
19. A sheet of area 40 m2 in used to make an open tank with a square base, then find the
dimensions of the base such that volume of this tank is maximum.

PART – II : INTEGRATION
Find integrals of given functions

x
3
1. ( x  1)dx

 (1  cot
2
2. x)dx c

3.  cos (tan   sec )d


Integrate by using the substitution suggested in bracket

 12( y  4 y 2  1) 2 ( y 3  2 y )dy, u  y 4  4 y 2  1)
4
4.

dx
5.  5x  8

(a) Using u = 5x + 8 (b) Using u = 5x  8

6.  3  2s ds

 sec (3x  2)
2
7.
     
8.  csc  2
 cot 
  2
 d

6cos t
9.  (2  sin t ) 3
dt

2
10.   d

3 7
11.  0
x 2 dx

 x sin x dx
2
12.
0

1
dx
13.  3x  2
0

Use a definite integral to find the area of the region between the given curve and the x-axis on the interval [0,b],

14. y = 3x2

PART – III VECTOR

SUBJECTIVE QUESTIONS

1. Vector 𝐴 points N – E and its magnitude is 3 kg ms-1 it is multiplied by the scalar  = -4 second.
Find the direction and magnitude of the new vector quantity. Does it represent the same physical
quantity or not?
2. A force of 30 N is inclined at an angle 𝜃 to the horizontal. If its vertical component is 18 N,
find the horizontal component & the value of 𝜃
3. Two vectors acting in the opposite directions have a resultant of 10 units. If they act at
right angies to each other, then the resultant is 50 units. Calculate the magnitude of two vectors.
4. The angle 𝜃 between directions of forces 𝐴 and 𝐵 ⃗ is 900 where A = 8 dyne and B = 6 dyne.
If the resultant 𝑅⃗ makes an angle 𝛼 with 𝐴 then find the value of ‘𝛼’
5. ⃗⃗⃗⃗⃗ , 𝑂𝐵
Find the resultant of the three vectors 𝑂𝐴 ⃗⃗⃗⃗⃗ and 𝑂𝐶
⃗⃗⃗⃗⃗ each of magnitude r as shown in figure?

6. If 𝐴 = 3𝑖̂ ⃗ = 𝑖̂ +𝑗̂ + 2𝑘̂ then find out unit vector along 𝐴 + 𝐵


+4𝑗̂ and 𝐵 ⃗.

7. The x and y components of vector 𝐴 are 4 , amd 6m respectively. The x,y components of
⃗ are 10m and 9m respectively. Find the length of 𝐵
vector 𝐴 + 𝐵 ⃗ and angle that 𝐵
⃗ makes with the x axis.

OBJECTIVE QUESTIONS
8. A Vector is not changed if
(A) it is displaced parallel to itself (B) it is rotated through an arbitrary angle
(C) it is cross-multiplied by a unit vector (D) it is multiplied by an arbitrary scalar.

9. If the angle between tow forces increases, the magnitude of their resultant#
(A) decreases (B) increases
(C) remains unchanged (D) first decrease and then increases
10. A car is moving on a straight road due north with a uniform speed of 50 km h-1 when it turns
left through 900. If the speed remains unchanged after turning, the change in the velocity of the
car in the turning process is
(A) zero (B) 50 2 km h-1 S – W direction

(C) 50 2 km h-1 N – W direction (D) 50 km h-1 due west.

11. Which of the following sets of displacements might be capable of bringing a car to its returning point?
(A) 5, 10, 30 and 50 km (B) 5, 9, 9 and 16 km
(C) 40, 40, 90 and 200 km (D) 10, 20, 40 and 90 km
12. When two vector 𝑎 and 𝑏⃗ are added, the magnitude of the resultant vector is always
(A) greater than (a + b) (B) less than or equal to (a + b)
(C) less than (a + b) (D) equal to (a + b)
13. ⃗ | = |𝐴| = |𝐵
If |𝐴 + 𝐵 ⃗ | , then the angle between 𝐴 and 𝐵
⃗ is
0 0
(A) 0 (B) 60 (C) 900 (D) 1200
14. Given: 𝑎 + 𝑏⃗ + 𝑐 = 0. Out of the three vectors 𝑎 , 𝑏⃗ and 𝑐 two are equal in magnitude.
The magnitude of the third. Vectors is 2 times that of either of the two having equal magnitude. The
angles between the vectors are:

(A) 900 , 1350,1350 (B) 300, 600, 900 (C) 450, 450, 900 (D) 450,600,900
15. Vector 𝐴 is of length 2 cm and is 600 above the x-axis in the first quadrant. Vector 𝐵 ⃗ is of length
0
2 cm and 60 below the x-axis in the fourth quadrant. The sum 𝐴 + 𝐵 ⃗ is a vector of magnitude –
(A) 2 along + y-axis (B) 2 along + x-axis (C) 1 along - x-axis (D) 2 along - x-axis
16. Six forces, 9.81 N each, acting at a point are coplanar. If the angles between neighboring forces
are equal, then the resultant is
(A) 0 N (B) 9.81 N (C) 2 × 9.81 N (D) 3 × 9.81 N.
17. A vector 𝐴 points vertically downward & 𝐵 ⃗ points towards east, then the vector product 𝐴 × 𝐵 ⃗ is
(A) along west (B) along east (C) zero (D) along south

MORE THAN ONE CHOICE TYPE

18*. Which of the arrangement of axes of Fig. can be labeled “right-handed coordinates system”?
As usual, each axis label indicates the positive side of the axis.

19.* Which of the following is a true statement?


(A) A vector cannot be divided by another vector
(B) Angular displacement can either be a scalar or a vector
(C) Since addition of vectors is commutative therefore vector subtraction is also commutative
(D) The resultant of two equal forces of magnitude F acting at a point is F if the angle between
the two forces is 1200.
20*. In the Figure which of the ways indicated for combining the x and y components of vector a are
proper to determine that vector?

|𝑎⃗|
21*. Let 𝑎 and 𝑏⃗ be two non-null vectors such that |𝑎 + 𝑏⃗| = |𝑎 - ⃗⃗⃗⃗
2𝑏|. Then the value of |𝑏⃗| may be:
1 1
(A) 4 (B) (C) 1 (D) 2
8

EXERCISE - 3
PART – I ; MATCH THE COLUMN
1. Match the integrals (given in column – II) with the given functions (in column – I)
Column – I Column – II
𝑐𝑜𝑠𝑒𝑐𝐾𝑥
(A) ∫ sec 𝑥 tan 𝑥 𝑑𝑥 (p) - +C
𝐾
𝑐𝑜𝑠𝑒𝑐𝐾𝑥
(B) ∫ 𝑐𝑜𝑠𝑒𝑐 𝐾𝑥 cot 𝐾𝑥 𝑑𝑥 (q) - +C
𝐾
2
(C) ∫ 𝑐𝑜𝑠𝑒𝑐 𝐾𝑥 𝑑𝑥 (r) sec x + C
𝑠𝑖𝑛𝐾𝑥
(D) ∫ 𝑐𝑜𝑠 𝐾𝑥𝑑𝑥 (s) 𝐾 + C
2. Match the statements given in column – I with statements given in column - II
Column – I Column – II
⃗ ⃗ ⃗
(A) If = |𝐴| = |𝐵| and |𝐴 + 𝐵| = |𝐴| then angle between 𝐴 and 𝐵 is (p) 900
(B) Magnitude of resultant of two forces |𝐹1 | = 8N and |𝐹2 | = 4 N may be (q) 1200
(C) Angle between 𝐴 = 2𝑖̂ + 2𝑗̂ & 𝐵 ⃗ = 3𝑘̂ is (r) 12 N
(D) Magnitude of resultant of vectors 𝐴 = 𝑖̂ + 𝑗̂ & 𝐵 ⃗ = 3𝑘̂ is (s) √14

PART – II : COMPREHENSION
COMPREHENSION – 1
A particle is moving along positive x-axis. Its position varies as x = t3-3t2 + 12t + 20, where x
is in meters and t is in seconds.
3. Initial velocity of the particle is.
(A) 1 m/s (B) 3 m/s (C) 12 m/s (D) 20 m/s
4. Initial acceleration of the particle is.
(A) Zero (B) 1 m/s2 (C) -3 m/s2 (D) -6 m/s2
5. Velocity of the particle when its acceleration zero is
(A) 1 m/s (B) 3 m/s (C) 6 m/s (D) 9 m/s
COMPREHENSION – 2
Two forces 𝐹1 = 2𝑖̂ + 2𝑗̂ N and 𝐹2 = 3𝑗̂ + 4𝑘̂ N are acting on a particle.
6. The resultant force acting on particle is:
(A) 2𝑖̂ +5𝑗̂ +𝑘̂ (B) 2𝑖̂ -5𝑗̂ -𝑘̂ (C) 𝑖̂ -3𝑗̂ -2𝑘̂ (D) 𝑖̂ -𝑗̂ -𝑘̂
7. The angle between 𝐹1 & 𝐹2 is :
3 3 2 √3
(A) 𝜃 = cos-1(2√5) (B) 𝜃 = cos-1(5√2) (C) 𝜃 = cos-1(3√5) (D) 𝜃 = cos-1( 5 )
8. The component of force 𝐹1 along force 𝐹2 is :
5 5 6 5
(A) 6 (B) 3 (C) 5 (D) 2
PART – III : ASSERTION / REASON
9. Statement – 1 : A vector is a quantity that has both magnitude and direction and obeys the
triangle law of addition.
Statement – 2 : The magnitude of the resultant vector of two given vectors can never be less
than the magnitude of any of the given vector.
(A) Statement -1 is True, Statement-2 True; Statement-2 is a correct explanation for Statement-1
(B) Statement -1 is True, Statement-2 True; Statement-2 is NOT a correct explanation for Statement-1
(C) Statement-1 is True, Statement-2 is False
(D) Statement-1 is True, Statement-2 is True
10. Statement-1 : If the rectangular components of a force are 8 N and 6N, then the magnitude of the
force is 10N.
Statement-2 : If |𝐴|=|𝐵⃗ | = then |𝐴 × 𝐵⃗ |2+|𝐴 . 𝐵
⃗ |2 = 1
(A) Statement -1 is True, Statement-2 True; Statement-2 is a correct explanation for Statement-1
(B) Statement -1 is True, Statement-2 True; Statement-2 is NOT a correct explanation for Statement-1
(C) Statement-1 is True, Statement-2 is False
(D) Statement-1 is True, Statement-2 is True
11. Statement-1 : If three vectors 𝐴, 𝐵 ⃗ and 𝐶 satisfy the relation 𝐴, 𝐵
⃗ = 0 & 𝐴, 𝐶 = 0 then the vector
⃗ × 𝐶.
𝐴 is parallel 𝐵
Statement-2: 𝐴 ⊥ 𝐵 ⃗ and 𝐴 ⊥ 𝐶 and 𝐵 ⃗ × 𝐶 ≠ 0 hence𝐴 is perpendicular to plane formed by 𝐵 ⃗ and 𝐶 .
(A) Statement -1 is True, Statement-2 True; Statement-2 is a correct explanation for Statement-1
(B) Statement -1 is True, Statement-2 True; Statement-2 is NOT a correct explanation for Statement-1
(C) Statement-1 is True, Statement-2 is False
(D) Statement-1 is True, Statement-2 is True
12. Statement-1 : The minimum number of non-zero vectors of unequal magnitude required to produce
zero resultant is three.
Statement-2 : Three vectors of unequal magnitude which can be represented by the three sides
of a triangle taken in order, produce zero resultant.
(A) Statement -1 is True, Statement-2 True; Statement-2 is a correct explanation for Statement-1
(B) Statement -1 is True, Statement-2 True; Statement-2 is NOT a correct explanation for Statement-1
(C) Statement-1 is True, Statement-2 is False
(D) Statement-1 is True, Statement-2 is True
𝜋
13. Statement-1 : The angle between the two vectors (𝑖̂+𝑗̂) and (𝑘̂) is 2 radian.

𝐴.𝐵
Statement-2 : Angle between two vectors (𝑖̂+𝑗̂) and (𝑘̂) is given by 𝜃 = cos-1( 𝐴𝐵 )
(A) Statement -1 is True, Statement-2 True; Statement-2 is a correct explanation for Statement-1
(B) Statement -1 is True, Statement-2 True; Statement-2 is NOT a correct explanation for Statement-1
(C) Statement-1 is True, Statement-2 is False
(D) Statement-1 is True, Statement-2 is True
14. Statement-1 : Distance is a scalar quantity.
Statement-2 : Distance is the length of path transversed.
(A) Statement -1 is True, Statement-2 True; Statement-2 is a correct explanation for Statement-1
(B) Statement -1 is True, Statement-2 True; Statement-2 is NOT a correct explanation for Statement-1
(C) Statement-1 is True, Statement-2 is False
(D) Statement-1 is True, Statement-2 is True

PART – IV : TRUE / FALSE


15. State True or False
(i) f(x) = -f’(x) for some function f.
(ii) f(x) = f’(x) for some function f.
(iii) If 𝐴 & 𝐵⃗ are two force vectors then 𝐴 . 𝐵
⃗ = 𝐵⃗.𝐴
(iv) If 𝐴 & 𝐵⃗ are two force vectors then 𝐴 × 𝐵 ⃗ = 𝐵
⃗ ×𝐴
(v) If the vector product of two non-zero vectors vanishes, the vectors are collinear.
PART – V : FILL IN THE BLANKS
16. Fill in the blanks
⃗ and 𝐶 representing the sides of a cube of
(i)The magnitude of sum of three vectors 𝐴 , 𝐵
length A is equal to ……

(ii) If 𝐴 = 3𝑖̂ + 4𝑗̂ and 𝐵⃗ = 7𝑖̂ + 24𝑗̂, then the vector having the same magnitude as 𝐵

and parallel to 𝐴 is …..
⃗ then 𝐴 × 𝐵
(iii) If 𝐴 || 𝐵 ⃗ = ………..
(iv) The magnitude of area of the parallelogram forced by the adjacent sides of vectors 𝐴 = 3𝑖̂ + 2𝑗̂
and 𝐵⃗ = 2𝑖̂ + 4𝑘̂ is ……
(v) Sum of two opposite vector to each other is a ………….. vector.
(vi) The unit vector along vector 𝑖̂ +𝑗̂ +𝑘̂ is ………….
(vii) If 𝐴 is …….. to 𝐵 ⃗ , then 𝐴 . 𝐵
⃗ =0
(viii) The vector 𝐴 = 𝑖̂ + 𝑗̂, where 𝑖̂ and 𝑗̂ are unit vectors along x-axis and y-axis respectively,
makes an angle of …….. degree with x-axis.
(ix) ⃗ +𝐶 =0
If 𝐴 + 𝐵 ⃗ , then 𝐴 . (𝐵⃗ × 𝐶 ) = ……………

EXERCISE # 1

PART – 1 PART - II

SECTION – (A) SECTION – (A)

𝑥3 2√𝑥 3
A -1 1 A-2 47 A – 3 (C) A – 4. (A) A–1 - 𝑥 2 + x + c A – 2. + 2 √𝑥 + c
3 3
3𝑥 4/3 3𝑥 2/3
A – 5 (C) A-6 (C) A – 7. (A), (C), (D) A–3 + +c
4 4
A – 8. (A), (B), (C), (D) A – 9 (A), (B), (C)
A – 4 tan x + c A – 5. - cot x + c
1
SECTION – (B) A – 6 sec x + c A – 7. 3 nx + c

𝑑𝑦
B – 1. = 2x + 1 B – 2 sec2 x – cosec2 x SECTION – (B)
𝑑𝑥
𝑑𝑦 𝑑2 𝑦 1 1
B–3 = cos x –sin x , 𝑑𝑥 2 = - sinx – cos x B – 1 - 4 cos(2x2) + C B–2 sec 2t + C
𝑑𝑥 2
𝑑𝑦 1 𝑑2 𝑦 1 2
B–4 = 𝑥 + ex, 𝑑𝑥 2 = - 𝑥 2 + ex B–3 +C B–4 -
𝑑𝑥 2−𝑥
cos(8𝑧−5)
+C
8
SECTION – (C) SECTION – (C)

ex 3𝜋
C – 1 ex nx + C – 2. cos2 x – sin2 x C–1 C – 2 24 C–3 e-1
𝑥 2

SECTION – (D) SECTION – (D)


−19 1 nx 𝑏
D -1. y’ = (3𝑥−2)2 D – 2 𝑥2 - D – 1 Using n subintervals of length ∆x = and
𝑥2 𝑛
right -
b
endpoint values : Area  2 xdx = b2 units
0
𝑑𝑦 𝑏2 𝑏(4+𝑏)
D–3 =0 D–2 +b= units
𝑑𝑥 4 4
7
D – 4. (a) 13 (b) -7 (c) 25 (d) 20 D – 3 2 units D – 4 𝜋 / 2 units

SECTION – (E) D – 5 (D) D – 6. (B), (C), (D)

E – 1 5 cos 5 x E – 2 2 𝜔cos(𝜔x + ∅) D – 7 (A), (B), (C)


𝑑𝑦
E – 3 𝑑𝑥 = -27(4-3x)8 PART - III
SECTION – (F) SECTION – (A)
𝑑𝑦 −2𝑥𝑦− 𝑦 2
F–1 = -1 F–2 A – 1 (i) 1050, (ii) 1500, (iii) 1050
𝑑𝑥 𝑥 2 +2𝑥𝑦
SECTION – (G)
A – 2 1200 A – 3 𝑉⃗ 𝑅 = 5𝑗̂ A – 4. (C)
𝑑𝐴 𝑑𝑟 𝑑𝑠 𝑑𝑟
G – 1 𝑑𝑡 = 2𝜋r 𝑑𝑡 . G – 2 𝑑𝑡 = 8𝜋r 𝑑𝑡 SECTION – (B)
SECTION – (H) B – 1 30 m East B – 2 50, 530 with
East
H–1 8 H – 2 (i) 39 (ii) 38 B – 3 (A) B–4 (B) B – 5 (B)
B – 6 (D)
SECTION – (I) B – 7 (D) B–8 (A) B – 9 (C)
B – 10 (D)
𝑑𝑦
I – 1. 𝑑𝑥 = 48(8X – 1)2 I – 2 3 cos (3x + 1) B – 11 (D)
𝑑𝑦 1 𝑥
I – 3 12x3. I – 4. = - 3 sin 3 SECTION – (C)
𝑑𝑥
3𝑖̂+4𝑗̂
C – 1. √14 C – 2. 5
C – 3. -25 cos 300 and -25 sin 300
√11
C – 4. 30√3 km h-1. C – 5. ± 10
C – 6 150. C – 7. (A)
2 1
SECTION – (D) 5. [5 √5𝑥 + 8] + C 6. - 3 (3 − 2s)3/2+ C
1 ʋ−𝜋
D-1 (a) 3 (b) - 𝑖̂ + 2𝑗̂ - 𝑘̂ 7. tan (3x + 2) + C 8. -2 csc( )+C
3 2
−3 3𝜋 2
D–2 (a) 6(b) 6 √3 D–3 (D) D – 4 (B), (C) 9. +C 10.
(2+sin 𝑡)2 2
1
7 1 5 5 3
11. 12. 1 13. n 2 = n(2)
3 3
𝑏
EXERCISE # 2 14. Using n subintervals of length ∆x = 𝑛 and right end point
b
PART – I values : Area = 
0
3x2 dx = b3
1
1. -𝑥 2. x 3. (C) 1. No it does not represent the same
physical quantity. 12
4. (A),(C) 5. (A),(B),(D) 6. (C),(D) 2. 24N ; 370 approx
𝑑𝑦 2 𝑑2 𝑦 −2
7. 𝑑𝑥
= 𝑥 + cos x, 𝑑𝑥 2 = 𝑥2
- sin x 3. P = 40 ; Q = 30
6

𝑑𝑦 𝑥 7
8. = + sec2 x, 4. 370 5. r(1 + √2)
𝑑𝑥 7
−13
𝑑2 𝑦 −6 ̂
4𝑖̂+5𝑗̂ +2𝑘 1
= 𝑥 7 + 2tan x sec2 x 6. 7. 3√5, tan-1 2
𝑑𝑥 2 49 √45
𝑑𝑦 2 1
9. = 1 + 2x + 𝑥 3 - 𝑥 2 8. (A) 9. (A) 10. (B) 11.
𝑑𝑥
(B)
𝑑𝑟 −𝑐𝑠𝑐 2 𝑥
10. = cos𝜃+ sec2𝜃 11. 12. (B) 13. (D) 14. (A)
𝑑𝜃 (1+cot 𝑥)2
15. (B)
1
tan 𝑥 (𝑒 𝑥 + )−𝑠𝑒𝑐 2 𝑥(𝑒 𝑥 + nx
𝑥
12. 16. (A) 17. (D) 18. (A), (B), (C)
𝑡𝑎𝑛2 𝑥
2
13. 3sin x cos x + 3cos 3x 19. (A), (B), (D) 20. (A), (B), (C)
14. 4x sin (x2 + 1) cos (x2 + 1) 21. (C), (D)
1−𝑟 𝑑𝑦 18𝑦− 3𝑥 2
15. 16. = 3𝑦 2 − 18𝑥 EXERCISE # 3
√2𝑟−𝑟 2 𝑑𝑥
𝑑𝑉 𝑑ℎ
17. (a) 𝑑𝑡 = 𝜋r2 𝑑𝑡 = 5 𝜋r2 1. (A) → r, (B) → p (C) → q,
(D) → s
𝑑𝑉 𝑑𝑟
(b) 𝑑𝑡 = 2𝜋ℎr 𝑑𝑡 = 10𝜋rh 2. (A) → q, (B) → r (C) → p,
(D) → s
𝑑𝑉 𝑑ℎ 𝑑𝑟
(c) 𝑑𝑡 = 𝜋r2 𝑑𝑡 +2𝜋ℎr 𝑑𝑡 = 5 𝜋r2+ 10𝜋rh 3. (C) 4. (D) 5. (D) 6. (A)
7. (B) 8. (C)
18. y = 30 9. (C) 10. (B) 11. (D) 12. (A) 13. (A) 14.
(A)
40
19. so volume is maximum at x = √ 3 m 15. (i) True (ii) True (iii) True (iv)
False (v) True
PART – II 16. (i) (√3) A (ii) 15𝑖̂ + 20 𝑗̂.
1 1
1. - 𝑥 - 2𝑥 2 + C 2. 2x + cot x + C (iii) Null vector (iv) √224 units
3. - cos 𝜃 + 𝜃 + C 4. (y 4 + 4y 2 + 1)2+ C (v) Negative, Positive or Zero
1 1 1
(vi) 𝑖̂ + 𝑗̂ + 𝑘̂ (vii)
√3 √3 √3
Perpendicular.
(viii) 450 (ix) Zero
1
17. - 3 ( 7- 3y2)3/2 + C
1 𝑥 1 𝑥
ALP Answers 18. sin6(3) + C 19. tan8 2 + C
2 4
6 4
𝑟3 1 𝑟5
PART – I 20. (18 − 1) + C 21. - 2 (7 − 10) + C
3
22. - 4 cos (x4/3-8)+ C 23. -2 csc
ʋ−𝜋
( )+C
2
𝑥 2 +𝑥+4 2
1. g’(x) = (𝑥+0.5)2 24. - 3(cot3y)1/2 + C 25. √𝑠𝑒𝑐 𝑧 +
C
2 1 1
2. (a) -2 (b) 25 (c) - 2 (d) -7 26. - sin ( 𝑡 − 1) + C
−2 𝑐𝑜𝑠𝑒𝑐 𝑡 cot 𝑡 𝑑𝑠 1
3. 4. = cos 𝑡−1 27. Area = 2 square units
(1−𝑐𝑜𝑠𝑒𝑐 𝑡)2 𝑑𝑡
𝑑𝑦 𝑑𝑦 𝑑𝑢
5. With u = sin x, y = u3: 𝑑𝑥 = 𝑑𝑢 𝑑𝑥 = 3u2 cos 28. Area = 2.5 square units
𝜋3
x = 3 sin2 x(cos x) 29. 30. 9b3
24
𝑑𝑦 𝑐𝑜𝑠𝑒𝑐 𝜃
6. = 20 sin x cos-5x 7. PART - III
𝑑𝑥 cot 𝜃+𝑐𝑜𝑠𝑒𝑐 𝜃
𝑠𝑒𝑐 𝜃
8. 1. 2√2 P 2. 3 km in North
sec 𝜃+𝑡𝑎𝑛 𝜃
3. 1500 4. 1130
PART – II 5. When the angle between A & B is 1200; when
it is 600
1
1. (a) – csc x + C (b) 5 csc (5x) + C 6. Hence P : Q : R = √2 : √3 : √2
𝜋𝑥 1
(c) 2 csc ( 2 ) + C 7. 𝐴3 = 2 (-𝑖̂ + √3 𝑗̂)
2. 3x + sin 2x + 4 sin x + C 8. And directed at an angle ∅ = tan-
1 3−2√2
[3√3+2√2−2] with-ve
3
𝑥2 8
3. + 4x1/2 + C 4. 4y2 - 3 y3/4 + C x – direction, as shown in the figure.
3
2
5. x2 + 𝑥 + C 6. 3 cot x + C 9. The magnitude of displacement = |∆𝑟|
𝜃
= 2Rsin . The
2
–tan 𝑥 1
7. +C 8. - 2 csc 𝜃 + C directon
3
2 𝑃𝑄 𝑃𝑄
9. 𝑠𝑒𝑐 𝜃 + C 10. 4 sec x – 2 tan x + C 10. 𝑟 - 𝑟0 + 𝑎̂. Then by putting = n,
5 𝐴 𝐴
we obtain
1 1 1
11. - 2 cot x + 2 csc x + C 12. - 2 cos 2x + cot x + C 𝑟 = 𝑟0 + n𝑎̂

13. sin 2x + cos 3x + C 14. 2y – sin 2y + C 11. |𝑅⃗ | = |𝐴 + 𝐵


⃗ | = √|𝐴|2 + |𝐵
⃗ |2 + 2|𝐴||𝐵
⃗ |cos 𝜃
2
15. tan 𝜃 + C 16. (5s + 4)1/2 + C 12. 𝑖̂ + 𝜋𝑗̂ +2𝑘̂ 13. -5 cos t
5
RECTILINEAR MOTION

Mechanics is the branch of physics which deals with the cause and effects of motion of a particle, rigid
objects and deformable bodies etc. Mechanics is classified under tow streams namely Statics and
dynamics. Dynamics is further divided into Kinematics and Kinetics

Mechanics

Statics Dynamics
Study of forces and their Study of forces and their
effect on objects at rest effect on objects in motion

Kinematics Kinetics
The word kinematics means It is branch of mechanics
'science of motion'. Branch of which is concermed about the
mechanics which deals with causes (i.e. the force , torque)
study of motion without going that cause motion of bodies.
into the cause of motion, i.e.
force, torqueetc.

1 MOTION AND REST

Motion is a combined property of the object and the observer. There is no meaning of rest
or motion without the observer. Nothing is in absolute rest or in absolute motion.
An object is said to be in motion with respect to a observer, if its position changes with respect
to that observer. It may happen by both ways either observer moves or object moves.

RECTILINEAR MOTION

Rectilinear motion is motion, along a straight line or in one dimension. It deals with the
kinematics of a particle in one dimension.

2.1 Position
The position of a particle refers to its location in the space at a certain moment of time. It is concerned
with the question – “Where is the particle at a particular moment of time?”
2.2 Displacement

The change in the position of a moving object is known as


displacement. It is the vector joining the initial position (𝑟1)
of the particle to its final position (𝑟2 ) during an interval of time.
Displacement can be negative positive or zero.

2.3 Distance
The length of the actual path travelled by a particle during a given time interval is called as
distance. The distance travelled is a scalar quantity which is quite different from displacement. In
general, the distance travelled between two points may not be equal to the magnitude of the
displacement between the same points.
Solved Example

Example 1.
Ram takes path 1 (straight line) to go from P to Q and
Shyam takes path 2 (semicircle)
(a) Find the distance travelled by Ram and Shyam?
(b) Find the displacement of Ram and Shayam?
Solution:
(a) Distance travelled by Ram = 100 m
Distance travelled by Shyam = 𝜋(50 m) = 50 𝜋 m
(b) Displacement of Ram = 100 m
Displacement of Shyam = 100 m

2.4 Average Velocity (in an Interval):

The average velocity of a moving particle over a certain time interval is defined as the
displacement divided by the lapsed time.
𝑑𝑖𝑠𝑝𝑙𝑎𝑐𝑒𝑚𝑒𝑛𝑡
Average Velocity = 𝑡𝑖𝑚𝑒 𝑖𝑛𝑡𝑒𝑟𝑣𝑎𝑙

for straight line motion, along x-axis, we have


∆𝑥 𝑥𝑓 −𝑥𝑖
vav= v̅ = <v> = =
∆𝑡 𝑡𝑓 −𝑡𝑖

The dimension of velocity is [LT-1] and its Sl unit is m/s.

The average velocity is a vector in the direction of displacement. For motion in a straight line,
directional aspect of a vector can be taken care of by +ve and –ve sign of the quantity.

2.5 Instantaneous Velocity (at an instant) :

The velocity at a particular instant of time is known as instantaneous velocity.


The term “velocity usually means instantaneous velocity.

 x  𝑑𝑥
Vinst. = lim   = 𝑑𝑡
t 0 t
 

In other words, the instantaneous velocity at a given moment(say, t) is the limiting


value of the average velocity as we let ∆𝑡 approach zero. The limit as ∆𝑡 →0 is written in
calculus notation as dx/dt and is called the derivative of x with respect to t.
Note:
 The magnitude of instantaneous velocity and instantaneous speed are equal.
 The determination of instanteous velocity by using the definition usually
𝑑𝑥
involves calculation of derivative. We can find v = 𝑑𝑡 by using the standard results from
differential calculus.
 Instantaneous velocity is always tangential to the path.
Solved Example
Example 2. A particle starts from a point A and travels along the solid curve shown in figure. Find
approximately the position B of the particle such that the average velocity between the
positions A and B has the same direction as the instantaneous velocity at B.

Answer: x = 5m, y = 3m

Solution: The given curve shows the path of the particle starting at y = 4 m.
𝑑𝑖𝑠𝑝𝑙𝑎𝑐𝑒𝑚𝑒𝑛𝑡
Average velocity = 𝑡𝑖𝑚𝑒 𝑖𝑛𝑡𝑒𝑟𝑣𝑎𝑙 ; where displacement is straight line distance between
points Instantaneous velocity at any point is the tangent drawn to the curve at that point

Now, as shown in the graph, line AB shows displacement as well as the tangent to
the given curve. Hence, point B is the point at which direction of AB shows average as well as
instantaneous velocity.
2.6 Average Speed ( In an interval)

Average speed is defined as the total path length travelled divided by the total time interval
during which the motion has taken place. It helps in describing the motion along the actual path.
𝑑𝑖𝑠𝑝𝑙𝑎𝑐𝑒𝑚𝑒𝑛𝑡
Average Speed = 𝑡𝑖𝑚𝑒 𝑖𝑛𝑡𝑒𝑟𝑣𝑎𝑙

The dimension of velocity is [LT-1] and its Sl unit is m/s.

Note :

 Average speed is always positive in contrast to average velocity which being a vector, can be
positive or negative.
 If the motion of a particle is along a straight line and in same direction then, average
velocity = average speed.
 Average speed is, in general, greater than the magnitude of average velocity.#

Solved Example
Example 3. In the example 1, if Ram takes 4 seconds and Shyam takes 5 seconds to go from P to Q,
find
100
Solution: (a) Average speed of Ram = m/s = 25 m/s
4
50𝜋
Average speed of Shyam = m/s = 10 𝜋 m/s
5
100
(b) Average velocity of Ram = m/s = 25 m/s (From P to Q)
4
100
Average velocity of Shyam = 5 m/s = 20 m/s (From P to Q)
Example 4. Average travels half of total distance with speed v1 and next half with speed v2 along a
straight line. Find out the average speed of the particle?
Solution : Let total distance travelled by the particle be 2s.
𝑆
Time taken to travel first half = 𝑉
1
𝑆
Time taken to travel next half = 𝑉
2
𝑇𝑜𝑡𝑎𝑙 𝑑𝑖𝑠𝑡𝑎𝑛𝑐𝑒 𝑐𝑜𝑣𝑒𝑟𝑒𝑑 2𝑆 2𝑉 𝑉
1 2
Average Speed = = 𝑆 𝑆 = 𝑉 +𝑉 (harmonic progression)
𝑇𝑜𝑡𝑎𝑙 𝑡𝑖𝑚𝑒 𝑡𝑎𝑘𝑒𝑛 + 1 2
𝑉1 𝑉2
Example 5. A person travelling on a straight line moves with a uniform velocity v1 for some time
and with uniform velocity v2 for the next equal time, The average velocity v is given by
𝑉 +𝑉
Answer: v = 1 2 2 (Arithmetic progression)

Solution:

As shown, the person travels from A to B through a distance S, where first part, S1
is travelled in time t/2 and next S2 also in time t/2.
𝑆1 𝑆2
So, according to the condition: v1 = 𝑡/2 and V2 = 𝑡/2
𝑉1 𝑡 𝑉2 𝑡
𝑇𝑜𝑡𝑎𝑙 𝑑𝑖𝑠𝑝𝑙𝑐𝑒𝑚𝑒𝑛𝑡 𝑆1 +𝑆2 + 𝑉 +𝑉
Average velocity = 𝑇𝑜𝑡𝑎𝑙 𝑡𝑖𝑚𝑒 𝑡𝑎𝑘𝑒𝑛 = 𝑡 = 𝑡 = 1 2 2 2 2

2.6 Average acceleration (In an Interval):


The average acceleration for a finite time interval is defined as :
𝑐ℎ𝑎𝑛𝑔𝑒 𝑖𝑛 𝑣𝑒𝑙𝑜𝑖𝑐𝑡𝑦
Average acceleration = 𝑡𝑖𝑚𝑒 𝑖𝑛𝑡𝑒𝑟𝑣𝑎𝑙
Average acceleration is a vector quantity whose direction is same as that of the change in
velocity.

∆𝑣 ⃗ −𝑣
𝑣 ⃗
𝑎𝑎𝑣 = ∆𝑡 = 𝑓∆𝑡 𝑖
Since for a straight line motion the velocities are along a line, therefore
∆𝑣 𝑣𝑓 −𝑣𝑖
aav = ∆𝑡 =
𝑡𝑓 −𝑡𝑖

(where one has to substitute vt and vi with proper signs in one dimensional motion)

2.8 Instantaneous Acceleration (at an instant):

The instantaneous acceleration of a particle is its acceleration at a particular instant of time. It is


defined as the derivative (rate of change) of velocity with respect to time. We usually mean
instantaneous acceleration when we say “acceleration”. For straight motion we define
instantaneous acceleration as:

𝑑𝑣  v  ⃗
𝑑𝑣  v 
a = 𝑑𝑡 = lim   and in general 𝑎 = = lim  
t 0 t t  0  t
  𝑑𝑡
 

The dimension of acceleration is [LT-2] and its Sl unit is m/s2

3. GRAPHICAL INTERPRETATION OF SOME QUANTITIES

3.1 Average Velocity

If a particle passes a point P(X1) at time t = t1 and reaches Q(X1) at a later time instant
∆𝑥 𝑥𝑓 − 𝑥𝑖
t = tf, its average velocity in the interval PQ is Vav = =
∆𝑡 𝑡𝑓 −𝑡𝑖

x x
P Q

This expression suggests that the average velocity is equal


to the slope of the line (chord) joining the points corresponding
to P and Q on the x-t graph.

3.2 Instantaneous Velocity

Consider the motion of the particle between the two points P and Q on the x-t graph shown. As
the point Q is brought closer and closer to the point, P, the time interval between PQ (∆𝑡, ∆𝑡′,
∆𝑡",.....) get progressively smaller. The average velocity for each time interval is the slope of the
appropriate dotted line (PQ, PQ’,PQ”.......)

As the point Q approaches P, the time interval approaches


zero, but at the same time the slope of the dotted line
approaches that of the tangent to the curve at the point P.
As ∆𝑡 → 0, Vav (=∆𝑥/∆𝑡) →Vinst.
Geometrically, as ∆𝑡 → 0, chord PQ → tangent at P.

Hence the instantaneous velocity at P is the slope of the


tangent at P in the x – t graph. When the slope of the x – t
graph is positive, v is positive (as at the point A in figure).
At C, v is negative because the tangent has negative slope.
The instantaneous velocity at point B (turning point) is zero as the slope is zero.

Instantaneous Acceleration :
The derivative of velocity with respect to time is the slope of the tangent in velocity time (v – t)
graph.

Solved Example
Example 6. Position of a particle as a function of time is given as x = 5t2 + 4t + 3. Find the
velocity and acceleration of the particle at t= 2 s?
𝑑𝑥
Solution: Velocity v = = 10 t + 4
𝑑𝑡
At t=2s
v = 10(2) + 4
v = 24 m/s
𝑑2 𝑥
Acceleration ; a = 𝑑𝑡 2 = 10
Acceleration is constant, so at t = 2 s a = 10 m/s2
4. MOTION WITH UNIFORM VELOCITY
Consider a particle moving along x – axis with uniform velocity u starting from the point
x = xi at t= 0.
Equations of x, v, a are : x(1) = xi + ut ; v(t) = u ; a (t) = 0
 x – t graph is a straight line of slope u through xi.
 as velocity is constant, v – t graph is a horizontal line.
 a – t graph coincides with time axis because a = 0 at all time instants.
x
xi slo
p e=
xi u
u is positive u is negative
O t t
O

Positive velocity t
u O a
negative velocity
t u

. UNIFORMLY ACCELERATED MOTION:

If a particle is accelerated with constant acceleration in an interval of time, then the motion is termed as
uniformly accelerated motion in that interval of time.
For uniformly accelerated motion along a straight line (x – axis) during a time interval of t seconds, the
following important results can be used.

𝑣−𝑢
(a) a= 𝑡

𝑣+𝑢
(b) Vav = 2

(c) S = (Vav)t
𝑣+𝑢
(d) S=( )t
2

(e) v = u + at

(f) s = ut + 1/2 at2

s = vt – 1/2 at2

xf = xi + ut + ½ at2

(g) v2 = u2 + 2as

(h) sn = u + a/2 (2n – 1)

u= initial velocity (at the beginning of interval)

a= acceleration

v= final velocity (at the end of interval)

s= displacement (xf – xi)

xf = final coordinate (position)

xi = initial coordinate (position)

Sn = displacement during the nth sec


6. DIRECTIONS OF VECTORS IN STRAIGHT LINE MOTION

In straight line motion, all the vectors (position, displacement, velocity & acceleration) will have only
one component (along the line of motion) and there will be only two possible directions for each vector.

 For example, if a particle is moving in a horizontal line (x – axis), the two directions are
right and left. Any vector directed towards right can be represented by a positive number and
towards left can be represented by a negative number.
 For vertical or inclined motion, upward direction
can be taken +ve and downwards as –ve

 For objects moving vertically near the surface of the earth, the only force acting on the
particle is its weight (mg) i.e. the gravitational pull of the earth. Hence acceleration for this type
of motion will always be a = -g i.e. a= - 9.8 m/s2 (-ve sign, because the force and
acceleration are directed downwards, If we select upward direction as positive).

Note :

 If acceleration is in same direction as velocity, then speed of the particle increases.


 If acceleration is in opposite direction to the velocity then speed decreases i.e. the particle slows
down. This situation is known as retardation.

Solved Examples

Example 7. A particle moving rectilinearly with constant acceleration is having initial velocity of 10 m/s.
After some time, its velocity becomes 30 m/s. Find out velocity of the particle at the
mid point of its path?

Solution: Let the total distance be 2x.

∴ distance upto midpoint = x

Let the velocity at the mid point be v

and acceleration be a.

From equations of motion

v2 = 102 + 2ax _____(1)

302 = v2 + 2ax _____(2)

(2) – (1) gives

v2 – 302 = 102 – v2

 v2 = 500  v = 10 5 m/s
Example 8. Mr. Sharma brakes his car with constant acceleration from a velocity of 25 m/s to
15 m/s over a distance of 200 m.

(a) How much time elapses during this interval?


(b) What is the acceleration?
(c) If he has to continue braking with the same constant acceleration, how much
longer would it take for him to stop and how much additional distance would he cover?
Solution: (a) We select positive direction for our coordinate system to be the direction of the
Velocity and choose the origin so that xi = 0 when the braking begins.
Then the initial velocity is ux = +25 m/s t = 0, and the final velocity and
position are vx = +15m/s and x = 200 m at time t.
Since the acceleration is constant, the average velocity in the interval can be
found from the average of the initial and final velocities.
1 1
∴ vav,x = 2(ux + vx) = 2 (15 + 25) = 20 m/s.
The average velocity can also be expressed as
∆𝑥
vav,x = ∆𝑡 . With ∆𝑥 = 200 m
and ∆𝑡 = t – 0, we can solve for t:
∆𝑥 200
t = 𝑣 = 20 = 10 s.
𝑎𝑣.𝑥
(b) We can now find the acceleration using vx = ux + axt
𝑣𝑥 −𝑢𝑥 15−25
ax = = = -1 m/s2
𝑡 10

The acceleration is negative, which means that the positive velocity is becoming smaller
as brakes are applied (as expected).

(c) Now with known acceleration, we can find the total time for the car to go
from velocity ux = 25 m/s to vx = 0. Solving for t, we find
𝑣 −𝑢 0−25
t = 𝑥𝑎 𝑥 = −1 = 25 s.
𝑥
The total distance covered is
1
x = xi + uxt + 2 axt2
1
= 0 + (25) (25) + 2 (-1) (25)2
= 625 – 312.5
= 312.5 m.
Additional distance covered
= 312.5 – 200
= 112.5 m
Example 9. A police inspector in a jeep is chasing a pickpocket an a straight road. The jeep
is going at its maximum speed v (assumed uniform). The pickpocket rides on the
motorcycle of a waiting friend when the jeep is at a distance d away, and the motorcycle starts
with a constant acceleration a. Show that the pick pocket will be caught if v ≥ √2𝑎𝑑.
Solution: Suppose the pickpocket is caught at a time t after motorcycle starts.
The distance travelled by the motorcycle during this interval is
1
S = 2 at2 ____ (1)
During this interval the jeep travels a distance
s + d = vt _____(2)
By (1) and (2)
1 2
at + d = vt
2
𝑣± √𝑣 2 −2𝑎𝑑
or, t= 𝑎
The pickpocket will be caught it t is real and positive.
This will be possible if
v2 ≥ 2ad or, v ≥ √2𝑎𝑑
Example10. A man is standing 40 m behind the bus. Bus starts with 1 m/sec2 constant acceleration
and also at the same instant the man starts moving with constant speed 9 m/s. Find the time taken
by man to catch the bus

Solution : Let after time ‘t’ man will catch the bus
For bus
1 1
x = x0 + ut + 2 at2 , x = 40 + 0(t) + 2 (1) t2
𝑡2
x = 40 + 2 ................(i)
For man , x = 9t ................(ii)
From (i) & (ii)
𝑡2
40 + 2 = 9t or t = 8 s or t = 10 s.
Example 11. A particle is dropped from height 100 m and another particle is projected vertically
up with velocity 50 m/s from the ground along the same line. Find out the position where two
particle will meet? (tame g = 10 m/s2)
Solution : Let the upward direction is positive.
Let the particles meet at a distance y from the ground.
For particle A,
y0 = +100 m
u = 0 m/s
a = - 10 m/s2
1 1
y = 100 + 0(t) = 2 × 10 × t2 [y = y0 + ut + 2 at2]
= 100 = 5t2 --------(1)

For particle B.
u = + 50 m/s
a = - 10 m/s2
1
y = 50(t) - 2 × 10 × t2
= 50t – 5t2 ------(2)
According to the problem;
50 t – 5t2 = 100 – 5t2
t = 2 sec
Putting t = 2 sec in eqn. (1),
y = 100 – 20 = 80 m

Hence, the particles will meet at a height 80 m above the ground.

Example 12. A particle is dropped from a tower. It is found that it travels 45 m in the last second of its
journey. Find out the height of the tower? (take g = 10 m/s2)
Solution :
Let the total time of journey be n seconds.
𝑎 10
Using; Sn = u + 2 (2n – 1)  45 = 0 + 2 (2n – 1)

n = 5 sec
1 1
Height of tower ; h = 2 gt2 = 2 × 10 × 5t2 = 125 m

7. REACTION TIME
When a situation demands our immediate action. It takes some time before we really respond.
Reaction time is the time a person takes to observe, think and act.

Solved Examples

Example 13. A stone is dropped from a balloon going up with a uniform velocity of 5 m/s. If the balloon
was
M high when the stone was dropped, find its height when the stone hits the ground.
Take g = m/s2.
1
Solution: S = ut + 2 at2
1
- 60 = 5(t) + 2 (-10) t2
- 60 = 5t – 5t2
5t2 – 5t – 60 = 0
t2 – t – 12 = 0
t2 – 4t + 3t – 12 = 0
(t – 4) (t + 3) = 0
∴ t=4
Height of balloon from ground at this instant
= 60 + 4 × 5 = 80 m
Example 14. A balloon is rising with constant acceleration 2 m/sec2. Two stones are released from
the balloon at the interval of 2 sec. Find out the distance between the two stones 1 sec after the
release of second stone.
Solution: Acceleration of balloon = 2 m/sec2
Let at t = 0. y = 0 when the first stone is released.
1
By the question, y1 = 0 t1 + 2 gt12
(taking vertical upward as – ve and downward as + ve)
9
∴ Position of Ist stone = g
2
(1 second after release of second stone will be the 3rd second for the 1st stone)
1
For second stone y2 = it2 + 2 gt22
U = 0 + at = -2 × 2 = - 4 m/s (taking vertical upwards as – ve and downward as + ve)
1
∴ y2 = -4 × 1 + 2 g × (1)2 (t2 = 1 second)
nd
The 2 stone is released after 2 second
1 1
∴ y = - 2 at2 = - 2 × 2 × 4 = -4
1
So, Position of second stone from the origin = -4 + 2 g – 4
1 1
Distance between two stones = 2 g × 9 - 2 g × 1 + 8 = 48 m.
Note:
 As the particle is detached from the balloon it is having the same velocity as that of
balloon, but its acceleration is only due to gravity and is equal to g.
8. STRAIGHT LINE-EQUATION, GRAPH, SLOPE (+ve, - ve, zero slope).
If 𝜃 is the angle at which a straight line is inclined to the positive direction of x – axis,
& 00 ≤ 𝜃< 1800, 𝜃 ≠ 900, then the slope of the line, denoted by m, is defined by m = tan 𝜃.
If 𝜃 is 900, m does not exist, but the line is parallel to the y-axis. If 𝜃 = 0, then m = 0 &
the line is parallel to the x-axis.
Slope – intercept form : y = mx + c is the equation of a straight line whose slope is m &
which makes an intercept c on the y - axis
𝑑𝑦
m = slope = tan 𝜃 = 𝑑𝑥
PARABOLIC CURVE-EQUATION, GRAPH(VARIOUS SITUATIONS UP, DOWN,
LEFT, RIGHT WITH CONDITIONS)

Where k is a positive constant.

Equation of parabola:
Case (i): y = ax2 + bx + c
For a > 0
The nature of the parabola will be like that of the of nature x2 = ky
Minimum value of y exists at the vertex of the parabola.
−𝐷
ymin = 4𝑎 D = b2 – 4ac
−𝑏 𝐷
Coordinates of vertex = ( 2𝑎 , 4𝑎)
Case (ii) : a < 0
The nature of the parabola will be like that of the nature of x2 = -ky
Maximum value of y exists at the vertex of parabola.
𝐷
ymin = 4𝑎 D = b2 – 4ac

10. GRAPHS IN UNIFORMLY ACCELERATED MOTION ( a ≠ 0)

 x is a quadratic polynomial in terms of t. Hence x – t graph is a parabola.

x-t graph

 v is a linear polynomial in terms of t. Hence v-t graph is a straight line of slope a.


 a – t graph is a horizontal line because a is constant.

INTERPRETATION OF SOME MORE GRAPHS

Position vs Time graph

(i) Zero Velocity

As position of particle is fixed at all the time.

so the body is at rest.


𝑑𝑥
Slope; = tan𝜃 = tan 00 = 0
𝑑𝑡

Velocity of particle is zero

𝑑𝑥
(ii) Here tan𝜃 is constant tan𝜃 = 𝑑𝑡

𝑑𝑥
∴ is constant.
𝑑𝑡

∴ velocity of particle is constant.

(iii) Non uniform velocity (increasing with time)

In this case;

As time is increasing, 𝜃 is also increasing.


𝑑𝑥
∴ = tan𝜃 is also increasing
𝑑𝑡

Hence, velocity of particle is increasing

(iv) Non uniform velocity (decreasing with time)

In this case;

As time increases, 𝜃 decreases.


𝑑𝑥
∴ = tan𝜃 is also decreasing
𝑑𝑡

Hence, velocity of particle is decreasing.


Velocity vs time graph

(i) Zero acceleration

Velocity is constant.

tan𝜃 = 0
𝑑𝑣
∴ =0
𝑑𝑡

Hence, acceleration is zero.

(ii) Uniform acceleration

tan𝜃 is constant.
𝑑𝑣
∴ = constant
𝑑𝑡

Hence, it shows constant acceleration.

(iii) Uniform retardation

Since 𝜃 > 900

∴ tan𝜃 os constant and negative.


𝑑𝑣
∴ = negative constant
𝑑𝑡

Hence, it shows constant retardation.

Acceleration vs time graph

(i) Constant acceleration

tan𝜃 = 0
𝑑𝑎
∴ =0
𝑑𝑡

Hence, acceleration is constant.

(ii) Uniformly Increasing acceleration

𝜃 is constant.

00 < 𝜃 < 900  tan 𝜃 > 0


𝑑𝑎
∴ = tan 𝜃 = constant > 0
𝑑𝑡

Hence, acceleration is uniformly increasing with time.

(iii) Uniformly decreasing acceleration

Since 𝜃 > 900


∴ tan 𝜃 is constant and negative
𝑑𝑎
∴ = negative constant
𝑑𝑡

Hence, acceleration is uniformly decreasing with time.

Solved Examples
Example 15.

The displacement vs time graph of a particle moving along


a straight line is shown in the figure. Draw velocity vs time
and acceleration vs time graph.
Solution: x = 4t2
𝑑𝑥
v= = 8t
𝑑𝑡

Hence, velocity-time graph is a straight line

having slope i.e. tan 𝜃 = 8.


𝑑𝑣
a= =8
𝑑𝑡

Hence, acceleration is constant throughtout and is equal to 8.

Example 16. At the height of 100 m, a particle A is thrown up with


V = 10 m/s, B particle is thrown down with V = 10 m/s
and C particle released with V = 0 m/s. Draw graphs of
each particle.
(i) Displacement – time

(ii) Speed-time

(iii) Velocity – time

(i) Acceleration-time

Solution: For particle A :

(i) Displacement vs time graph is


1
y = ut + 2 at2
u = + 10 m/sec2
1
y = 10t - 2 × 10 t2
= 10 t – 5t2
𝑑𝑦
v = 𝑑𝑡 = 10 – 10 t = 0
t = 1 ; hence, velocity is zero at t = 1
10 t – 5 t2 = -100
t2 – 2t – 20 = 0
t = 5.5 sec.
i.e. particle travels up till 5.5 seconds.
(ii) Speed vs time graph :

Particle has constant acceleration = g ↓ throughout the motion, so v-t curve will be straight line.

when moving up, v = u + at

0 = 10 – 10 t or t = 1 is the time at which speed is zero.

there after speed increases at constant rate of 10 m/s2

Resulting Graph is : (speed is always positive).

This shows that particle travels till a time of

1+ 21 seconds

(iii) Velocity vs time graph :


V = u + at
V = 10 – 10t ; this shows that velocity
becomes zero at t = 1 sec
and thereafter the velocity is negative with slope g.

(iv) Acceleration vs time graph:


Throughout the motion particle
has constant acceleration = -10 m/s2

For particle B :
1
u = - 10 m/s. y = - 10 t - 2 (10) t2 = - 10t – 5t2

(i) Displacement time graph :

y = 10t – 5t2
𝑑𝑡
= -10t – 5t2 = -10 – 10t
𝑑𝑦

this shows that slope becomes more negative with time.

(ii) Speed time graph :


𝑑𝑦
= -10t – 5t2 = -10 – 10t
𝑑𝑡

hence, speed is directly

proportional to time with


slope of 10. Initial speed = 10 m/s

(iii) Velocity time graph :


𝑑𝑦
= -10t – 5t2 = -10 – 10t
𝑑𝑡

hence, velocity is directly

proportional to time with

slope of -10. Initial velocity = -10 m/s

(iv) Acceleration vs time graph:

throughout the motion

particle has constant acceleration = -10 m/s2.


𝑑𝑦
a= = -10
𝑑𝑡

For particle C :

(i) Displacement time graph :


1
u = 0 , y = - 2 × 10t2 = -5t2
this shows that slope becomes
more negative with time.

(ii) Speed vs time graph :


𝑑𝑦
v = 𝑑𝑡 = - 10t
hence, speed is directly
proportional to time with
slope of 10.

(iii) Velocity time graph:

V = u + at

V = - 10 t

hence, velocity is directly

proportional to time with

slope of - 10.

(iv) Acceleration vs time graph :

throughout the motion,

particle has constant

acceleration = - 10 m/s2
12. DISPLACEMENT FROM v – t GRAPH & CHANGE IN VELOCITY FROM a – t GRAPH

Displacement = ∆x = area under v-t graph.

Since a negative velocity causes a negative displacement, areas below


the time axis are taken negative. In similar way, can see that ∆ v = a ∆t
leads to the conclusion that area under a – t graph gives the change in
velocity ∆v during that interval.

Solved Example
Example 17. Describe the motion shown by the following velocity-time graphs.

Solution : (a) During Interval AB: velocity is +ve so the particle is moving in +ve direction,
but it is slowing down as acceleration (slope of v-t curve) is negative. During
interval BC:particle remains at rest as velocity is zero. Acceleration is also zero.
During interval CD : velocity is –ve so the particle is moving in –ve direction and is
speeding up as acceleration is also negative.

(b) During interval AB: particle is moving in +ve direction with constant velocity and
acceleration is zero. During interval BC : particle is moving in +ve direction as
velocity is +ve, but it slows down until it comes to rest as acceleration is negative.
During interval CD: velocity is –ve so the particle is moving in –ve direction and is
speeding up as acceleration is also negative.

Important Points to Remember

 For uniformly accelerated motion (a ≠0) , x-t graph is a parabola (opening upwards
if a > 0 and opening downwards if a< 0). The slope of tangent at any point of the parabola
gives the velocity at that instant.
 For uniformly accelerated motion (a ≠0) , v-t graph is a straight line whose slope gives the
acceleration of the particle.
 In general, the slope of tangent in x-t graph is velocity and the slope of tangent in v-t graph is
the acceleration
 The area under a-t graph given the change in velocity.
 The area between the v-t graph gives the distance travelled by the particle, if we take all areas
as positive.
 Area under v-t graph gives displacement, if areas below the t-axis are taken negative.
Solved Example
Example 18. For a particle moving along x-axis, velocity-time
graph is as shown in figure. Find the distance travelled
and displacement of the particle?
Solution:

Distance travelled = Area under v-t graph (taking all areas as +ve)

Distance travelled=Area of trapezium + Area of triangle


1 1
= 2 (2 + 6)×8 + 2 × 4 × 5

= 32 + 10

= 42 m

Displacement = Area under v –t graph (taking areas below time axis as – ve.)

Displacement = Area of trapezium – Area of triangle


1 1
= 2 (2 + 6)×8 - 2 × 4 × 5

= 32 - 10

= 22m

Hence, distance travelled = 42 m and displacement = 22 m.

MOTION WITH NON-UNIFORM ACCELERATION

(USE OF DEFINITE INTEGRALS)


tf

∆x =  v (t )
ti

The expression on the right hand side is called the definite integral of v(t) between t = t1
and t = tf. Similarly change in velocity
tf

∆v = vf – vi =  a(t ) dt
ti

Solving Problems which involves Non uniform Acceleration


(i) Acceleration depending on velocity v or time t
𝑑𝑣
By definition of acceleration, we have a = If a is in terms of t,
𝑑𝑡

v (t ) t v t
dv

v ( t0 )
dv   a(t )dt . If a is in terms of v,
t0
v a(v)  0 dt
0
On integrating, we get a relation between v and t, and then
x t
using  dx   v(t ) dt, x and t can also be related.
x0 0

(ii) Acceleration depending on velocity v or position x


𝑑𝑣 𝑑𝑣 𝑑𝑥 𝑑𝑥 𝑑𝑣
a = 𝑑𝑡  a = 𝑑𝑥  a=
𝑑𝑡 𝑑𝑡 𝑑𝑥

𝑑𝑣
 a = v 𝑑𝑥

This is another important expression for acceleration.

If a is in terms of x,
v x

 vdv   a(x)dx
v0 x0

v x
vdv
If a is in terms of v, 
a(v) x0
 dx
v0

On integrating, we get a relation between x and v


x t
dx
Using 
v( x) 0
 dt , we can relate x and t.
x0

Solved Examples
Example 19. An object starts from rest at t= 0 and accelerates at a rate given by a = 6t. What

is

(a) its velocity and

(b) its displacement at any time t?

Solution: As acceleration is given as a function of time.


v (t ) t
 
v ( t0 )
dv   a(t )dt
t0

Here t0 = 0 and v(t0) = 0


t
t t 
2
t2
 v(t) =  v(t )dt = 6  = 6( .0) = 3t2
2
t0
 
2 0

So, v(t) = 3t2


t
As x   v(t )dt
t0
t
t t 3
 t3 
  3t dt = 3  3   0  = t3
2
c =
3 
t0
 
3 0

Hence, velocity v(t) = 3t2 and displacement x =t3.

Example 20. For a particle moving alongv + x-axis, acceleration is given as a = x. Find the
position as a function of time ? Given that at t = 0, x = 1 v = 1.

𝑣𝑑𝑣 𝑣2 𝑥2
Solution: a=x  𝑑𝑥
=x  2
= 2
+C

t = 0, x = 1 and v = 1

C = 0  v2 = x2

v= x but given that x = 1 when v = 1


𝑑𝑥 𝑑𝑥
 v=x  𝑑𝑡
=x  𝑥
= dt

nx = t + C  0=0+C  nx = 1

Example 21. For particle moving along x-axis, acceleration is given as a = v. Find the position as a
function of time?

𝑑𝑣 dv
Solution: a=v  𝑑𝑡
=v   v 
 dt

nv = t + c  0 =0+c
𝑑𝑥
v = et  𝑑𝑡
= et  0=1+c

x = et - 1

miscellaneous Solved Problems


3 1
Problem 1. A particle covers 4 of total distance with speed v1 and next 4 with v2. Find the

average speed of the particle?


4𝑣1 𝑣2
Answer: 𝑣1 +3𝑣2

Solution: Let the total distance be s


𝑇𝑜𝑡𝑎𝑙 𝑑𝑖𝑠𝑡𝑎𝑛𝑐𝑒
Average speed (<v>) = 𝑇𝑜𝑡𝑎𝑙 𝑡𝑖𝑚𝑒 𝑡𝑎𝑘𝑒𝑛

s 1 4v1v2
<v> =  
3

s 3

1 v1  3v2
4v1 4v2 4v1 4v2

Problem 2. A car is moving with speed 60 Km/h and a bird


is moving with speed 90 km/h along the same
direction as shown in figure. Find the distance
travelled by the bird till the time car reaches the. tree?
Answer: 360 m
240m 0.24
Solution: Time taken by a car to reaches the tree (t) = = hr
60km / hr 60

Now, the distance travelled by the bird during this time interval (s)

0.24
= 90  hr  0.12  3 km = 360 m.
60

Problem 3 The position of a particle moving on X – axis is given by

x = At3 + Bt2 + Ct + D

The numerical values of A, B, C, D are 1, 4 -2 and 5 respectively and Sl units are used Find (a)
the dimensions of A, B, C and D, (b) the velocity of the particle at t = 4 s, (c) the acceleration
of the particle at t = 4s, (d) the average velocity during the interval t= 0 to t = 4s, (e) the average
acceleration during the interval t = 0 to t = 4 s.

Answer: (a) [A] = [LT-3], [B] = [LT-2], [C] = [LT-1], and [D] = [L]

(b) 78 m/s ; (c) 32 m/s2 ; (d) 30 m/s ; (e) 20 m/s2

Solution: As x = At3 + Dt2 + Ct + D

(a) Dimensions of A, B, C and D,

[At3] = [x] (by principle of homogeneity)

[A] = [LT-3]

Similarly, [B] = [LT-2], [C] = [LT-1] and [D] = [L] ;


𝑑𝑥
(b) As v = = 3At2 + 2Bt + C
𝑑𝑡

velocity at t = 4 sec.

V = 3(1) (4)2 + 2(4) (4) – 2 = 78 m/s


𝑑𝑣
(c) Acceleration (a) = 𝑑𝑡 = 6 At + 2B ; a = 32 m/s2

(d) average velocity as x = At3 + Bt2+ Ct = D

position at t = 0, is x = D = 5m.

Position at t = 4 sec is (1) (64) + 4(16) – (2) (4) + 5 = 125 m

Thus the displacement during 0 to 4 sec. is 125 -5 = 120 m

 < v > = 120 / 4 = 30 m/s

(e) v = 3At2 + 20 t + C, velocity at t= 0 is c = -2 m/s

v2  v1
Velocity at t = 4 sec is 78 m/s  < a > = = 20 m/s2
t2  t1
Problem 4. For a particle moving along x – axis, velocity is given as function of time as v = 2t2 + sin t.
At t = 0, particle is at origin. Find the position as a function of time?
𝑑𝑥
Solution : v = 2t2 + sin t  𝑑𝑡
= 2t2 + sin t
x t
2
 dx =  (2t  sin t ) dt = x = t3 – cos t + 1 Ans.
2
3
0 0

Problem 5. A car decelerates from a speed of 20 m/s to rest in a distance of 100 m. What was its
acceleration, assumed constant?
Solution: v = 0 u = 20 m/s s = 100 m  as v2 = u2 + 2 as
0 = 400 + 2a × 100  a = -2 m/s
2
∴ acceleration = 2 m/s Ans.
Problem 6. A 150 m long train accelerates uniformly from rest. If the front of the train passes a
railway worker 50 m away from the station at a speed 25 m/s, what will be the speed
of the back part of the train as it passes the worker?
Solution: v2 = u2 + 2as
25 × 25 = 0 + 100 a
25
a = 4 m/s2
Now, for time taken by the back end of the train to pass the worker
25
we have v’2 = v2 + 2al = (25)2 + 2 × 48 × 150
v’2 = 25 × 25 × 4
v’ = 50 m/s . Ans
Problem 7. A particle is thrown vertically with velocity 20 m/s. Find (a) the distance travelled by
the particle in first 3 seconds, (b) displacement of the particle in 3 seconds.
Answer: 25m, 15m
Solution: Highest point say B
Vb = 0
v = u + gt
0 = 20 – 10 t
t = 2 sec.
∴ distance travel in first 2 seconds.
s = s(t = 0 to 2sec) + s (2sec. to 3sec.)
s = [ut + 1/2 at2]t=0 to t = 2s + [ut + 1/2 at2]t=2 to t = 3s
s = 20 × 2 – 1/2 × 10 × 4 + 1/2 × 10 ×12
= (40 – 20) + 5 = 25 m.
and displacement = 20 – 5 = 15 m.

Problem 8. A car accelerates from rest at a constant rate 𝛼 for some time after which it
decelerates at a constant rate 𝛽 to come to rest. If the total time elapsed is t. Find the maximum
velocity acquired by the car.
Solution: t = t1 + t 2
𝑉
slope of OA curve = tan𝜃 = 𝛼 = 𝑚𝑎𝑥 𝑡 2
𝑉𝑚𝑎𝑥
slope of AB curve = 𝛽= 𝑡1
t = t1 + t2
𝑉 𝑉 𝛼𝛽
 t = 𝑚𝑎𝑥𝛼
+ 𝑚𝑎𝑥
𝛽
 vmax = (𝛼+𝛽) t
Problem 9. In the above question find total distance travelled by the car in time ‘t’

𝛼𝛽 𝑣𝑚𝑎𝑥 𝛽𝑡 𝑣𝑚𝑎𝑥 𝛼𝑡
Solution : vmax = (𝛼+𝛽) t  t1 = 𝛼
= (𝛼+𝛽)  t2 = = 𝛽
= (𝛼+𝛽)

∴ Total distance travelled by the car in time ‘t’


1 1 1 𝛼𝛽 2 𝑡 2 𝛼2 𝛽𝑡 2 1 𝛽𝛼2 𝑡 2
= 2 𝛼𝑡112 + vmax t2 - 2 𝛽𝑡22 = 2 (𝛼+𝛽)2 + (𝛼+𝛽)2 - 2 (𝛼+𝛽)2
1 𝛼𝛽𝑡 2 𝛼𝛽𝑡 2
Area under graph (directly) = 2 (𝛼+𝛽) = 2(𝛼+𝛽) Ans.
Problem 10. The displacement vs time graph of a particle moving along
a straight line is shown in the figure. Draw velocity vs time
and acceleration vs time graph.

Upwards direction is taken as positive,


downwards direction is taken as negative.
Solution : (a) The equation of motion is : x = -8t2
𝑑𝑥
∴ v = 𝑑𝑣 = -16 t ; this shows that velocity is directly proportional to time and slope
of velocity-time curve is negative i.e. – 16.
Hence, resulting graph is (i)
𝑑𝑣
(b) Acceleration of particle is : a = 𝑑𝑡 = -16
This shows that acceleration is constant but negative.
Resulting graph is (ii)

Problem 11. Draw displacement-time and acceleration-time graph for the given velocity-time graph.

Solution: Part AB: v–t curve shows constant slope


i.e. constant acceleration or Velocity increases at constant rate with time.
Hence, s-t curve will show constant increase in slope and a-t curve will be a straight line.
Part BC : v – t curve shows zero slope i.e. constant velocity. So, s-t curve will show
constant slope and acceleration will be zero.
Part CD : v-t curve shows negative slope i.e. velocity is decreasing with time or
acceleration is negative.
Hence, s-t curve will show decrease in slope becoming zero in the end. and a-t curve
will be a straight line with
negative intercept on y-axis.

RESULTING GRAPHS ARE:


Problem 12. For a particle moving along x-axis, following graphs are given. Find the distance
travelled by the particle in 10 s in each case?

Solution: (a)Distance area under the v – t curve


∴ distance = 10 × 10 = 100 m Ans.
(b) Area under v – t curve
1
∴ distance = 2 ×10 × 10 = 50 m Ans.
Problem 13. For a particle moving along x-axis, acceleration is given as a = 2v2, If the speed
of the particle is v0 at x = 0, find speed as a function of x.
𝑑𝑣 𝑑𝑣 𝑑𝑥
Solution: a = 2v2  or 𝑑𝑡
= 2v2 or 𝑑𝑥 × 𝑑𝑡 = 2v2
𝑑𝑣 𝑑𝑣
v 𝑑𝑥 = 2v2  𝑑𝑥
= 2v
v x
dv
v v = 0 2dx  [ nv]vv0  [2 x]0x
0
𝑣
n𝑣 = 2x  v = v0e2x Ans.
0

EXERCISE-1

1. A person travels along a straight road for half the distance with velocity V1 and the remaining half
distance with velocity V2 the average velocity is given by
𝑣 2 𝑣1 +𝑣2 2𝑣1 𝑣2
(a)v1v2 (b)(𝑣2 ) (c) (d) 𝑣
1 2 1 +𝑣2
2. If a cyclist takes one minute to complete half revolution on a circular path 120 m radius. What is the
average velocity?
(a) 1 m/s (b) 2 m/s (c) 3 m/s (d) 4 m/s
3. A car travelling with a speed 126 KMPH along a straight line comes to rest after travelling a
distance 245 m. The time taken by the car to come to rest, in second it
(a) 11 (b) 12 (c) 16 (d) 14
4. A drunkard is walking along a straight road. He takes 5 steps forward and 3 steps backward and
so on. Each step is 1m long and takes 1s. There is a pit on the road 11m away from the starting point.
The drunkard will fall into the pit after
(a) 21 s (b) 29s (c) 31s (d) 37 s
5. If a car covers 2/5th of the total distance with v1 speed and 3/5th distance with v2 then average speed is
1 𝑣 +𝑣 2𝑣1 𝑣2 5𝑣1 𝑣2
(a) 2 √𝑣1 𝑣2 (b) 1 2 2 (c) 𝑣 +𝑣 (d) 3𝑣 +2𝑣
1 2 1 2
6. A motor vehicle travelled the first third of a distance s at a speed of
v1 = 10 kmph, the second third at a speed of v2 = 20 kmph and the last third at a speed of v3 = 60 kmph.
Determine the mean speed of the vehicle over the entire distance s.
(a) 15 kmph (b) 12 kmph (c) 10 kmph (d) 18 kmph
7. A motorist drives north for 35.0 minutes at 85.0 km/h and then stops for 15.0 minutes. He next
continues north, travelling 130 km in 2.00 hours. What is his total displacement
(a) 85 km (b) 179.6 km (c) 20 km (d) 140 km
8. A person walks along a straight road from his house to a market 2.5kms away with a speed of 5 km/hr
and instantly turns back and reaches his house with a speed of 7.5 kms/hr. The average speed of the
person during the time interval 0 to 50 minutes is (in m/sec)
2 5 5 1
(a) 4 (b) (c) (d)
3 3 6 3
9. A particle moving with a constant acceleration describes in the last second of its motion 9/25th of the
whole distance. If it starts from rest, how long is the particle in motion and through what distance
does it move if it describes 6 cm in the first sec.?
(a) 5 s; 150 cm (b) 10 s ; 150 c (c) 15 s ; 100 cm (d) None
10. A car moving with constant acceleration covers the distance between two points 180 m apart in 6 sec.
Its speed as it passes the second point is 45 m/s. What is its acceleration and its speed at the first point
(a) -5 m/s2 ; 15 m/s (b) -15 m/s2 ; 5 m/s (c) -5 m/s2 ; 15 m/s (d) 5 m/s2 ; 15 m/s
11. A car moving with a speed of 50km/hr can be stopped by brakes after at least 6 m. If the same car is
moving at a speed of 100 km/hr the minimum stopping distance is
(a) 12 m (b) 18 m (c) 24 m (d) 6 m
12. A particle starts moving from rest with uniform acceleration. It travels a distance x in the first 2 sec
and a distance y in the next 2 sec. Then
(a) y = k (b) y = 2x (c) y = 3x (d) y = 4x
13. The reaction time for an automobile driver is 0.7 sec. If the automobile can be decelerated at 5 m/s2
calculate the total distance travelled in coming to stop from an initial velocity of 8.33 m/s
after a signal is observed.
(a) 12.77 m (b) 14.82 m (c) 16.83 m (d) 19.65 m
14. If the particle is moving along a straight line given by the relation x = 2 – 3t + 4t3 where s is in cms.,
and t in sec. Its average velocity during the third sec is
(a) 73 cm/s (b) 80 cm/s (c) 85 cm/s (d) 90 cm/s
15. A bullet fired into a fixed target loses half of its velocity in penetrating 15 cm. How much further it will
penetrate before coming to rest?
(a) 5 cm (b) 15 cm (c) 7.5 cm (d) 10 cm
16. For a body travelling with uniform acceleration, its final velocity is v = √180 − 7𝑥, where x is the
distance travelled by the body. Then the acceleration is
(a) -8 m/s2 (b) -3.5 m/s2 (c) -7 m/s2 (d) 180 m/s2
2
17. If the velocity of a particle is v = At + Bt , where A and B constants, then the distance travelled
by it between 1s and 2s is.
3 3 7 𝐴 𝐵
(a) A + 4B (b) 3 A + 7B (c) A + B (d) +
2 2 3 2 3
18. The acceleration of a particle is increasing linearly with time t as bt. The particle starts from origin
with an initial velocity v0. The distance travelled by the particle in time t will be
1 1 1 1
(a) v0t + 3 bt2 (b) v0t + 2 bt2 (c) v0t + 6 bt3 (d) v0t + 3 bt3
19. A particle moves along a straight line such that its displacement at any time t is
given by s = (t3 – 6t2 + 3t + 4) metres. The velocity when the acceleration is zero is
(a) 3 m/s (b) 42 m/s (c) -9 m/s (d) -15 m/s
2
20. A bus starts from rest with a constant acceleration of 5 m/s . At the same time a car travelling with a
constant velocity 50 m/s overtakes and passes the bus. How fast is the bus travelling when they are side
by side?
(a) 10 m/s (b) 50 m/s (c) 100 m/s (d) none
21. A body falls from 80 m. Its time of descent is [g =10 ms-2]
(a) 3 s (b) 4 s (c) 5 s (d) 6 s
22. Two bodies whose masses are in the ratio 2 : 1 are dropped simultaneously at two places A and B
where the accelerations due to gravity are gA and gB respectively. If they reach the ground
simultaneously, the ratio of the heights from which they are dropped is
(a) gA : gB (b)2 gA : gB (c) gA : 2gB (d) √𝑔𝑎 : √𝑔𝑏
23. A body falls for 5 s from rest. If the acceleration due to gravity of earth ceases to act, the distance it
travels in the next 3 s is
(a) 73.5 (b) 294 m (c) 147 m (d) 49 m
7ℎ
24. A body freely falling from a height h describes 16 in the last second of its fall. The height h is
(g = 10 ms-2)
(a) 80 m (b) 45 m (c) 160 m (d) 40 m
25. A body released from the top of a tower of height h takes T seconds to reach the ground.
The position of the body at T/4 seconds is
ℎ ℎ
(a) at 16 from the ground (b) at 14 from the top of the tower
15ℎ 3ℎ
(c) at from the ground (d) at from the top of the tower
16 16

26. A stone falls freely under gravity. It covers distances h1, h2 and h3 in the first 5 seconds,
the next 5 seconds and the next 5 seconds respectively. The relation between h1, h2 and h3 is
ℎ ℎ
(a) h2 = 3h1 and h3 = 3h2(b) h1 = h2 = h3 (c) h1 = 2h2 = 3h3 (d) h1= 32 = 53
27. The ratio of times taken by freely falling body to cover first metre, second metre,.. is
(a) √1 : √2 : √3 (b) √1 : √2 - √1 : √3 - √2
(c) √2 : √4 : √8 (d) 2 : 3 : 4
28. A body is droped from a height 122.5 m. If its stopped after 3 seconds and again released the
further time of descent is (g = 9.8 m/s2)
(a) 2 s (b) 3 s (c) 4 s (d) 5 s
29. A freely falling body travels --- of total distance in 5th second
(a) 8% (b) 12% (c) 25% (d) 36%
30. If the distance travelled by a freely falling body in the last second of its journey is equal
to the distance travelled in the first 2s, the time of descent of the body is
(a) 5s (b) 1.5s (c) 2.5s (d) 3s
31. A ball dropped on to the floor from a height of 10 m rebounds to a height of 2.5 m. If the
ball is in contact with the floor for 0.02s, its average acceleration during contact is
(a) 2100ms-2 (b) 1050ms-2 (c) 4200ms-2 (d) 9.8ms-2

EXERCISE-2
1. A particle traversed along a straight line for first half time with velocity V0. For the remaining
part, half of the distance is traversed with velocity V1 and other half distance with velocity V2. Find the
mean velocity of the pariticle for the total journey.
2𝑣0 (𝑣1 +𝑣2 ) 𝑣0 (𝑣1 +𝑣2 )+2𝑣1 𝑣2 𝑣1 +𝑣2 𝑣0 +2𝑣1 𝑣2 )
(a) (b) (c) (d)
𝑣1 +𝑣2 +2𝑣0 2(𝑣1 +𝑣2 ) 2 2(𝑣1 +𝑣2 )
2. Two cars 1 & 2 starting from rest are moving with speeds v1 andv2 m/s (v1 > v2 ). Car 2 is
ahead of car ‘1’ by ‘S’ metres when the driver of car ‘1’ sees car ‘2’. What minimum retardation
should be given to car ‘1’ avoid collision
1v −v
2 v +v (v1 +v2) 2 (v1 −v2) 2
(a) (b) 1 S 2 (c) 2S (d) 2S
S
3. A particle moving with uniform retardation covers distances 18 m. 14 m and 10 m in successive
seconds. It comes to rest after travelling a further distance of
(a) 50 m (b) 8 m (c) 12m (d) 42m
4. The position x of a particle varies with time t as x = at2 – bt3. The acceleration of the particle
will be zero at time t equal to
𝑎 2𝑎 𝑎
(a) (b) 3𝑏 (c) 3𝑏 (d) Zero
𝑏
5. The relation 3t = √3𝑥 + 6 describes the position of a particle in one direction where x is in
metres and t in sec. The displacement, when velocity is zero, is
(a) 24 metres (b) 12 metres (c) 5 metres (d) Zero
6. An electron starting from rest has a velocity that increase linearly with the time that is v = kt,
where k = 2 m/sec2. The distance travelled in the first 3seconds will be
(a) 9m (b) 16m (c) 27m (d) 36m
7. A car accelerates from rest at a constant rate 𝛼 for some time, after which it decelerates at a
constant rate 𝛽 and comes to rest. If the total time elapsed is t, then the maximum velocity acquired by
the car is
𝛼2 +𝛽 2 𝛼2 −𝛽 2 𝛼𝛽𝑡 (𝛼+𝛽)𝑡
(a) ( )t (b) ( )t (c) 𝛼+𝛽 (d)
𝛼𝛽 𝛼𝛽 𝛼𝛽
8. Two towns A and B are connected by a regular bus service with a bus leaving in either direction
every T minutes. A man cycling with a speed of 20 km h-1 in the direction A to B notices that a bus
goes past him every 18 min in the direction of his motion, and every 6 min in the opposite direction.
The period T of the bus service and speed of buses on the road are
(a) 9 min, 40 Kmph (b) 12 min, 10 Kmph
(c) 12 min, 40 Kmph (d) 9 min, 40 Kmph
9. Two cars start in a race with velocities u1 and u2 and travel in a straight line with acceleration ‘a’ and b.
If both reach the finish line at the same time, the range of the race is
2(𝑢1 −𝑢2 ) 2(𝑢1 −𝑢2 )
(a) (𝑢1 𝛽 − 𝑢2 𝛼 (b) (𝑢1 𝛼 − 𝑢2 𝛽)
(𝛽− 𝛼)2 𝛽+ 𝛼
2(𝑢1 −𝑢2 )2 2𝑢1 𝑢2
(c) (d)
(𝛽− 𝛼)2 𝛽𝛼
𝑎
10. The displacement x of a particle varies with time according to the relation x = 𝑏 (1 – e-bt) . Then
(a) At t = 1/b, the displacement of the particle in nearly (1/3) (a/b)
(b) The velocity and acceleration of the particle at t = 0 are a and –ab respectively
(c) Both 1 and 2
(d) At t = 1/b, the displacement of the particle is (3/5) (a/b)
𝑡
11. A particle moving along x-axis has acceleration f, at time t, given by f = f0(1 − 𝑇). Where f0 and
T are constants. The particle at t = 0 has zero velocity. In the time interval between t = 0 and the instant
when f = 0, the particle’s velocity (vx) is
1 1
(a) 2 f0 T2 (b) f0 T2 (c) 2 f0 T (d) f0 T
12. A train takes t sec to perform a journey, if travel for t/n sec with uniform acceleration then
𝑛−3
for ( 𝑛 )t sec with uniform speed v and finally it comes to rest with uniform retardaction.
Then average speed of train is
𝑣 𝑣 𝑣 𝑣
(a) (3n -2 ) 2𝑛 (b) (2n -3) 2𝑛 (c) (3n -2 ) 3𝑛 (d) (2n -3) 3𝑛
13. A particle moving along straight line has velocity v = 𝜇𝑠 2 where s is the displacement. If s = s0 then
which of the following graph best represent s versus t

14. On a long horizontally moving belt, a child runs to and fro with a speed 9 km h-1(with respect to
the belt) between his father and mother located 50 m apart. The belt moves with speed of 4 km h-1.
For an observer on a stationary platform, and speed of the child running in the direction of motion
of the belt is
(a) 4 km h-1 (b) 5 km h-1 (c) 9 km h-1 (d) 13 km h-1
15. The distance travelled by a body moving along a line in time t is proportional to t3.
The acceleration – time (a, t) graph for the motion of the body will be

16. A body P is thrown vertically up with velocity 30 ms-1 and another body Q is thrown up
along the same vertically line with the same velocity but 1 second later from the ground.
When they meet
(g = 10 ms-2)
(a) P travels for 2.5s (b) Q travels for 3.5s
(c) P travels for 3.5s (d) Q travels for 1s
17. A particle is projected vertically up and another is let fall to meet at the same instant.
If they have velocities equal in magnitude when they meet, the distance travelled by them are
in the ratio of
(a) 1 : 1 (b) 1 : 2 (c) 3 : 1 (d) 2 : 3
18. A stone is dropped from a height of 10cm above the top of a window 80 cm high. The time
taken by the stone to cross the window is
(g = 9.8 ms-2)
1 3 2 4
(a) 7 s (b) 7 s (c) 7 s (d) 7 s
19. A body initially at rest is moving with uniform acceleration a. Its velocity
after n seconds is v. The displacement of the body in last 2 s is
2𝑣(𝑛−1) 𝑣(𝑛−1) 𝑣(𝑛+1) 2𝑣(𝑛+1)
(a) 𝑛 (b) 𝑛 (c) 𝑛 (d) 𝑛
20. Two balls are projected simultaneously with the same speed from the top of a tower – one
upwards and the other downwards. If they reach the ground in 6s and 2s, the height of the tower is
(g = 10 ms-2)
(a) 120m (b) 60m (c) 80m (d) 30 m
21. For the velocity – time graph shown in figure below the distance covered by the body in last
two seconds of its motion is what fraction of the total distance covered by it in all the total distance
covered by it in all the seven seconds
1
(a) 2
1
(b) 4
1
(c) 3
2
(d) 3
22. A particle starts from rest. Its acceleration (a) versus time(t) us as shown
in the figure. The maximum speed of the particle will be

(a) 110m/s
(b) 55 m/s
(c) 550m/s
(d) 660m/s
23. Velocity – time (v-t) graph for a moving object is shown in the figure. Total displacement
of the object during the time interval when there is non-zero acceleration and retardation is

(a) 60m
(b) 50m
(c) 30m
(d) 40m
24. A ball is thrown vertically upwards from the top of a tower. Velocity at a point ‘h’ m vertically
below the point of projection is twice the downward velocity at a point ‘h’ m vertically above the point
of projection. The maximum height reached by the ball above the top of the tower is
4 5ℎ
(a) 2h (b) 3 h (c) 3h (d) 3
25. When a stone falling from the top of vertical tower has fallen a distance x m, another is let fall from
a point y m below the top. If the fall from rest and reach the ground together, then the height of the
tower is
(𝑥+𝑦)2 4(𝑥+𝑦)2 4𝑥
(a) m (b) m (c) m (d) 4x(x + y)2m
4𝑥 𝑥 (𝑥+𝑦)2
26. A body is thrown vertically upward from a point ‘A’ 125 m above the ground. It goes up
to a maximum height of 250 m above the ground and passes through ‘A’ on its downward journey. The
velocity of the body when it is at a height of 70 m above the ground is
(g = 10 ms-2)
(a) 50ms-1 (b) 60ms-1 (c) 80ms-1 (d

EXERCISE -1 (ANSWER KEY)

(1) d (2) d (3) d (4) b (5) d


(6) d (7) b (8) b (9) a (10) d
(11) c (12) c (13) a (14) a (15) a
(16) b (17) c (18) c (19) c (20) c
(21) b (22) a (23) c (24) a (25) c
(26) d (27) b (28) c (29) d (30) c (31) b

EXERCISE - 2 (ANSWER KEY)

(1) b (2) d (3) b (4) c (5) d


(6) a (7) c (8) a (9) a (10) b
(11) c (12) b (13) c (14) d (15) a
(16) c (17) c (18) c (19) a (20) b
(21) b (22) b (23) b (24) d (25) a (26) b

You might also like